Contar e Inducir

´ UNIVERSIDAD PEDAGOGICA NACIONAL FACULTAD DE CIENCIA Y TECNOLOGIA ´ DEPARTAMENTO DE MATEMATICAS ´ ACTIVIDADES MATEMATI

Views 56 Downloads 9 File size 1MB

Report DMCA / Copyright

DOWNLOAD FILE

Recommend stories

Citation preview

´ UNIVERSIDAD PEDAGOGICA NACIONAL FACULTAD DE CIENCIA Y TECNOLOGIA ´ DEPARTAMENTO DE MATEMATICAS

´ ACTIVIDADES MATEMATICAS PARA EL DESARROLLO DE ´ PROCESOS LOGICOS CONTAR E INDUCIR Carlos Julio Luque Arias Lyda Constanza Mora Mendieta Jorge Edgar P´aez Orteg´on Profesores Departamento de Matem´aticas Universidad Pedag´ogica Nacional

Bogot´ a, D.C, 2002

´ ´ ACTIVIDADES MATEMATICAS PARA EL DESARROLLO DE PROCESOS LOGICOS

ii

´INDICE GENERAL

´ INTRODUCCION

1

´ 1. UN SISTEMA DE NUMEROS PARA CONTAR Introducci´on . . . . . . . . . . . . . . . . . . . . . . . . . 1.1. S´ımbolos y reglas del sistema . . . . . . . . . . . . 1.2. Operaciones . . . . . . . . . . . . . . . . . . . . . . 1.2.1. Adici´on . . . . . . . . . . . . . . . . . . . . 1.2.2. Sustracci´on . . . . . . . . . . . . . . . . . . 1.2.3. Multiplicaci´on . . . . . . . . . . . . . . . . . 1.2.4. Divisi´on . . . . . . . . . . . . . . . . . . . . 1.2.5. Potenciaci´on . . . . . . . . . . . . . . . . . . 1.3. El problema de los n´ umeros grandes . . . . . . . . . 1.3.1. Notaci´on multiplicativa . . . . . . . . . . . . 1.3.2. Notaci´on exponencial . . . . . . . . . . . . . 2. REPRESENTACIONES POSICIONALES 2.1. Una idea genial: usar la posici´on . . . . . . 2.2. Operaciones . . . . . . . . . . . . . . . . . 2.2.1. Adici´on . . . . . . . . . . . . . . . 2.2.2. Sustracci´on . . . . . . . . . . . . . 2.2.3. Multiplicaci´on . . . . . . . . . . . . 2.2.4. Divisi´on . . . . . . . . . . . . . . . 2.3. Ecuaciones con dos variables . . . . . . . . iii

. . . . . . . . . . .

. . . . . . . . . . .

. . . . . . . . . . .

13 13 14 18 18 19 20 24 29 30 30 31

´ DE NUMEROS . . . . . . . . . . . . . . . . . . . . . . . . . . . . . . . . . . . . . . . . . . . . . . . . . . . . . . . . . . . . . . . . . . . . . . . . . . . . .

. . . . . . .

. . . . . . .

37 37 46 46 50 57 66 76

. . . . . . . . . . .

. . . . . . . . . . .

. . . . . . . . . . .

. . . . . . . . . . .

. . . . . . . . . . .

´ ´ ACTIVIDADES MATEMATICAS PARA EL DESARROLLO DE PROCESOS LOGICOS

2.3.1. Representaci´on gr´afica de las soluciones . . . . . . . . . . . 2.4. Cambios de base . . . . . . . . . . . . . . . . . . . . . . . . . . . 2.5. Un problema l´ogico: la divisi´on por 0 . . . . . . . . . . . . . . . .

78 82 84

´ 3. LAS OPERACIONES SUPERIORES DE LA MATEMATICA 3.1. Potenciaci´on . . . . . . . . . . . . . . . . . . . . . . . . . . . . . . 3.1.1. Propiedades de la potenciaci´on . . . . . . . . . . . . . . . 3.2. Radicaci´on . . . . . . . . . . . . . . . . . . . . . . . . . . . . . . . 3.2.1. Propiedades de la radicaci´on . . . . . . . . . . . . . . . . . 3.2.2. C´alculo de ra´ıces . . . . . . . . . . . . . . . . . . . . . . . 3.3. Logaritmaci´on . . . . . . . . . . . . . . . . . . . . . . . . . . . . . 3.3.1. Propiedades de la logaritmaci´on . . . . . . . . . . . . . . . 3.4. Una aplicaci´on de la potenciaci´on: la escritura de n´ umeros grandes 3.5. Aplicaciones de bases diferentes a la base diez . . . . . . . . . . .

87 87 88 92 93 95 109 111 114 117

´ 4. SISTEMAS NUMERICOS EN ALGUNAS CULTURAS Introducci´on . . . . . . . . . . . . . . . . . . . . . . . . . . . . . 4.1. Los primitivos . . . . . . . . . . . . . . . . . . . . . . . . . 4.2. El sistema de los romanos . . . . . . . . . . . . . . . . . . 4.3. El sistema num´erico de los mayas . . . . . . . . . . . . . . 4.4. El sistema de numeraci´on inca . . . . . . . . . . . . . . . . 4.4.1. La representaci´on de los n´ umeros en el quipu . . . . 4.4.2. La Yupana, el “´abaco precolombino” . . . . . . . . 4.5. El sistema num´erico de Babilonia . . . . . . . . . . . . . . 4.5.1. Las operaciones aritm´eticas . . . . . . . . . . . . . 4.6. El sistema egipcio . . . . . . . . . . . . . . . . . . . . . . . 4.6.1. Las Operaciones . . . . . . . . . . . . . . . . . . . .

119 119 120 121 123 127 128 129 133 137 140 142

. . . . . . . . . . .

. . . . . . . . . . .

´ 5. LOS NUMEROS NATURALES EN OTRAS RAMAS DE ´ MATEMATICA Introducci´on: . . . . . . . . . . . . . . . . . . . . . . . . . . . . . . 5.1. Fen´omenos peri´odicos relacionados . . . . . . . . . . . . . . . 5.2. Los datos y las hip´otesis en un problema . . . . . . . . . . . . 5.3. Algunas veces no se puede . . . . . . . . . . . . . . . . . . . . 5.4. Regularidades y secuencias . . . . . . . . . . . . . . . . . . . . 5.4.1. Cuadrados m´agicos . . . . . . . . . . . . . . . . . . . . 5.4.2. Observando secuencias . . . . . . . . . . . . . . . . . . 5.5. Sumando sumas . . . . . . . . . . . . . . . . . . . . . . . . . . iv

. . . . . . . . . . .

. . . . . . . . . . .

LA 145 . . 145 . . 146 . . 149 . . 150 . . 151 . . 151 . . 152 . . 154

´INDICE GENERAL

5.6. ¿Cu´al es el n´ umero m´as grande? . . . . . . . 5.7. Contar en geometr´ıa . . . . . . . . . . . . . 5.8. Contar en topolog´ıa . . . . . . . . . . . . . . 5.9. Contar en teor´ıa de grafos. . . . . . . . . . . 5.10. Contar en teor´ıa de probabilidades . . . . . 5.11. Contar en teor´ıa de conjuntos . . . . . . . . 5.11.1. Subconjuntos y el conjunto de partes 5.11.2. El producto cartesiano de conjuntos . 5.11.3. Relaciones . . . . . . . . . . . . . . . 5.11.4. Funciones . . . . . . . . . . . . . . . ´ EN MATEMATICAS ´ 6. LA INDUCCION Introducci´on . . . . . . . . . . . . . . . . . . . 6.1. Los n´ umeros y el espacio . . . . . . . . . 6.1.1. N´ umeros pares e impares . . . . . 6.1.2. Los n´ umeros triangulares . . . . . 6.1.3. Los n´ umeros cuadrados . . . . . . 6.1.4. Los n´ umeros poligonales . . . . . 6.1.5. N´ umeros c´ ubicos . . . . . . . . .

. . . . . . .

. . . . . . .

. . . . . . . . . . . . . . . . .

. . . . . . . . . . . . . . . . .

. . . . . . . . . . . . . . . . .

. . . . . . . . . . . . . . . . .

. . . . . . . . . . . . . . . . .

. . . . . . . . . . . . . . . . .

´ ´ MATEMATICA ´ 7. EL METODO DE INDUCCION Introducci´on . . . . . . . . . . . . . . . . . . . . . . . . . . . 7.1. ¿Qu´e significa infinito? . . . . . . . . . . . . . . . . . . 7.2. El m´etodo de demostraci´on por Inducci´on Matem´atica 7.3. Definiciones por recurrencia . . . . . . . . . . . . . . . 7.4. Presentaci´on axiom´atica de los n´ umeros naturales . . . 7.4.1. La adici´on de n´ umeros naturales . . . . . . . . . 7.4.2. La multiplicaci´on de N´ umeros Naturales . . . . 7.5. El orden en los n´ umeros naturales . . . . . . . . . . . . BIBLIOGRAF´IA . . . . . . . . . . . . . . . . . . . . . . . .

v

. . . . . . . . . . . . . . . . . . . . . . . . . .

. . . . . . . . . . . . . . . . . . . . . . . . . .

. . . . . . . . . . . . . . . . . . . . . . . . . .

. . . . . . . . . . . . . . . . . . . . . . . . . .

. . . . . . . . . .

. . . . . . . . . .

156 158 161 164 170 173 173 174 175 176

. . . . . . .

179 . 179 . 180 . 180 . 182 . 187 . 193 . 195

. . . . . . . . .

203 . 203 . 206 . 211 . 223 . 226 . 227 . 231 . 234 . 239

´ ´ ACTIVIDADES MATEMATICAS PARA EL DESARROLLO DE PROCESOS LOGICOS

vi

´ INTRODUCCION

libro es el producto de la investigaci´on: “Actividades para el desarrollo E ste del pensamiento l´ ogico: El proceso de Contar”, la cual fue desarrollada en la Universidad Pedag´ogica Nacional durante los a˜ nos 1999 y 2000; uno de los prop´ositos de ella fue establecer lineamientos curriculares que sirvieran de base para el desarrollo del a´rea de a´lgebra en el primer semestre del nuevo proyecto curricular de Licenciatura en matem´aticas. Tuvo su origen al percibir que los estudiantes que ingresan al primer semestre de la Licenciatura en matem´aticas de la Universidad Pedag´ogica Nacional, tienen serias dificultades en la utilizaci´on de los n´ umeros naturales en contextos no triviales; de all´ı surgi´o la necesidad de proponer una secuencia de actividades matem´aticas en las que se enfatizaran los procesos requeridos para el conteo. Supusimos que las limitaciones en la construcci´on, expresi´on y comunicaci´on de ideas matem´aticas se deben a la falta de actividades matem´aticas que desarrollen procesos de creaci´on, discusi´on, proposici´on de algoritmos, manejo de teor´ıas, formulaci´on de conjeturas, formulaci´on y demostraci´on de teoremas y, en general, actividades caracter´ısticas del trabajo matem´atico, que son muy escasas en la educaci´on b´asica y media. Esta problem´atica no es exclusiva de la Universidad Pedag´ogica sino com´ un a muchas Instituciones de educaci´on b´asica, media y universitaria, lo que justifica su divulgaci´on. Dise˜ namos un conjunto de siete actividades, que se describen en cada uno de los cap´ıtulos del libro, con el prop´osito de mostrar ambientes acad´emicos de 1

´ ´ ACTIVIDADES MATEMATICAS PARA EL DESARROLLO DE PROCESOS LOGICOS

trabajo matem´atico; es decir, ambientes en los cuales el estudiante est´e en condiciones de crear conocimiento matem´atico nuevo para ´el, as´ı este conocimiento ya forme parte de la cultura matem´atica existente1. Para muchas personas, el estudio de las matem´aticas es un asunto dif´ıcil y a´rido, al que muy pocas personas privilegiadas y due˜ nas de un coeficiente intelectual particularmente elevado tienen acceso. Parece una clase particular de culto, con c´odigos secretos, algoritmos intrincados, s´olo para iniciados, como en los tiempos pitag´oricos donde, quien no conoc´ıa los ritos, no era admitido en el grupo. Parad´ojicamente, para algunos matem´aticos brillantes, como Bertrand Russell, es un campo donde no sabemos de qu´e hablamos, ni si lo que decimos es verdad; para otros, es un para´ıso de mundos posibles donde cada quien elige, seg´ un sus gustos particulares, entre engendros geom´etricos o topol´ogicos extravagantes, construcciones algebraicas portentosas, como los campos de n´ umeros algebraicos, y si no le gusta lo que est´a hecho, o es por ´el conocido, construye su propio mundo con el u ´nico requisito de que sea l´ogicamente coherente. Para casi todos los que de alguna manera hacen matem´aticas, es un placer, un arte, un disfrute, una fuente celestial de placeres que s´olo se encuentra aqu´ı, y quien ha descubierto y demostrado un teorema sabe de qu´e se trata. Existe la impresi´on de que en el ejercicio de las matem´aticas s´olo hay cabida para la deducci´on (el uso estricto de unas ciertas reglas l´ogicas) y no hay espacio para la intuici´on y la creaci´on; nada m´as lejos de la realidad. El matem´atico, frente a un problema, ensaya, yerra, vuelve a atacar con otro esquema y en general necesita una gran imaginaci´on y creatividad. Esta impresi´on se debe probablemente a que, en su manera de comunicarse, los matem´aticos utilizan un esquema inverso del utilizado en el descubrimiento; presentan primero las conclusiones y luego la manera de llegar a ellas, como si los conocimientos matem´aticos surgieran por generaci´on espont´anea. La presentaci´on habitual de los teoremas es lineal, precisa y sin divagaciones, pero rara vez se muestra la manera como se lleg´o a la idea; se ocultan los errores, no se 1

Asumimos, como hip´otesis, que es posible la actividad de creaci´on matem´atica (entendemos como actividad matem´atica la que desarrollan los matem´aticos en su labor diaria de crear y demostrar teoremas, proponer y resolver problemas) en los estudiantes de la Licenciatura; esta hip´ otesis fue validada en la investigaci´ on.

2

´ INTRODUCCION

muestran todos los intentos fallidos, s´olo el resultado final de mucho tiempo de incertidumbres, de b´ usqueda del camino correcto, de muchas noches de sobresalto, de levantarse semidormido a escribir la soluci´on que le vino durante el sue˜ no; porque ellos se llevan los problemas a la cama: cuando un matem´atico tiene un problema, parece ido, pero ah´ı est´a, en su cabeza, el problema bailando. Ninguna otra disciplina posee, como las matem´aticas, en grado tan profundo y tan preciso, el factor de la abstracci´on, entendida como la actividad intelectual que consiste en considerar un aspecto de la realidad y aislarlo de todo con la u ´nica finalidad de conocerla mejor. Las matem´aticas son un excepcional ejercicio para la mente; podr´ıamos llamar como una “gimnasia del cerebro”, como la define Bertrand Rusell, uno de los principales cient´ıficos que trabajaron en la modernizaci´on de la matem´atica. El trabajo de un matem´atico consiste en resolver problemas, descubrir y demostrar teoremas y enlazarlos en teor´ıas. Para hacer esto no se requieren secretos de magia bien guardados, ni actividades intelectuales extra˜ nas, por el contrario, el razonamiento matem´atico es s´olo una especializaci´on del razonamiento general, el matem´atico, para resolver sus problemas: 1. Recuerda (reconoce). 2. Relaciona (asocia). 3. Compara (mide) hace analog´ıas. 4. Justifica pasos y secuencias. 5. Clasifica (hace equivalencias). 6. Ordena. 7. Agrupa, compone (hace s´ıntesis). 8. Analiza (separa). 9. Invierte procesos. 10.

Demuestra (infiere).

11.

Genera (crea) 3

´ ´ ACTIVIDADES MATEMATICAS PARA EL DESARROLLO DE PROCESOS LOGICOS

12. Generaliza (reconoce regularidad). 13. Abstrae. 14. Aplica. 15. Eval´ ua. 16. Usa s´ımbolos. 17. Sustituye (traduce). 18. Aplica f´ormulas. 19. Expresa regularidades en t´erminos abstractos. 20. Tantea (pone a prueba ideas). 21. Planea cursos de acci´on. 22. Interpreta geom´etricamente. 23. Cuenta (calcula). 24. Transforma. 25. Interpola y extrapola. Y en forma m´as espec´ıfica, su tarea consiste en: 1. Proponer y discutir s´ımbolos, reglas y algoritmos. 2. Proponer y discutir definiciones. 3. Formular y describir relaciones. 4. Formular y argumentar proposiciones matem´aticas. 5. Establecer conjeturas. 6. Codificar y decodificar informaciones matem´aticas. 7. Formular y manejar teor´ıas matem´aticas. 8. Construir nuevas teor´ıas a partir de las ya establecidas. 4

´ INTRODUCCION

Si observamos con atenci´on, la mayor´ıa de estos procesos l´ogicos son comunes a cualquier otra actividad intelectual y acad´emica. Cabe entonces la pregunta, ¿c´omo es posible que haya gente que no entienda o que no le gusten la matem´aticas, si su trabajo s´olo invoca las reglas de la l´ogica aceptada por toda inteligencia normal y sus pruebas se fundan en principios comunes a todos los hombres?, ¿por qu´e tantos reacios a las matem´aticas? La respuesta debemos buscarla entonces en los primeros contactos que las personas tienen con las matem´aticas; es decir, en su aprendizaje. En las clases de matem´aticas tradicionales, nos encontramos con un profesor erudito que expone secuencialmente algunos resultados, m´etodos y algoritmos que el estudiante debe aprender y repetir posteriormente en un examen. Por la manera de presentaci´on pareciera que el profesor domina el tema y la participaci´on del estudiante se limita, en el mejor de los casos, a resolver ejercicios de manera mec´anica, sin mucha comprensi´on, y esta actividad definitivamente es muy aburrida. Nosotros consideramos el aprendizaje como un proceso constructivo y din´amico, en el cual el sujeto es el responsable directo (de su aprendizaje), pues ´el es quien construye su propio conocimiento a trav´es del papel activo que debe asumir como protagonista de los procesos de exploraci´on, an´alisis, s´ıntesis, generalizaci´on y formulaci´on de los contenidos matem´aticos (Moreno L. y Waldegg G., 1988). Consideramos necesario estimular al estudiante para que sea agente activo de su aprendizaje, y las actividades de descubrimiento contribuyen a tal fin, pues conllevan que ´el aprecie las matem´aticas como un proceso y no como un producto acabado, lo cual genera un ambiente propicio que favorece los procesos de transferencia de los conocimientos matem´aticos a diferentes esferas del pensamiento humano. Conducen tambi´en a que los estudiantes se familiaricen con los tipos particulares del pensamiento y del proceder de la matem´atica, lo ayudan a adquirir pr´actica en descubrir respuestas por s´ı mismos, a aprender c´omo producir y no a reproducir respuestas y conocimientos; adem´as, permiten que el estudiante, en lugar de almacenar informaci´on y evocarla s´olo en situaciones particulares, tenga la posibilidad de utilizarla para enfrentar otros problemas. Enfatizamos en que, para comprender y aprehender el conocimiento matem´atico, se requiere “hacer matem´aticas”. Por eso, en las actividades did´acticas que proponemos tenemos en cuenta que “el trabajo intelectual de los alumnos debe ser 5

´ ´ ACTIVIDADES MATEMATICAS PARA EL DESARROLLO DE PROCESOS LOGICOS

en muchas situaciones, comparable con el de los propios matem´aticos”(Godino C., Batanero V. y Navarro, 1995). Se pretende que los estudiantes tengan la oportunidad de investigar sobre los problemas, formular, probar, construir modelos, lenguajes, conceptos, teor´ıas, intercambiar sus ideas con otros (Lineamientos curriculares, MEN, 1998), reconocer los tratamientos propios de la cultura matem´atica, adoptar ideas que sean u ´tiles, descartar propuestas, criticar resultados, proponer procedimientos, formular hip´otesis, etc. (Santos L., 1993). En suma, tratamos de despertar en el aprendiz pasi´on y compromiso con la construcci´on del saber matem´atico. Desde esta perspectiva, el maestro tambi´en cumple un papel central; debe ser, ante todo, un mediador entre el alumno y el conocimiento. Su papel es mucho m´as activo y comprometido en la medida que debe ofrecer los elementos necesarios para promover la actividad cognitiva a partir del conocimiento responsable de los objetos de estudio y a la vez fomentar la interacci´on con sus alumnos, sin descuidar el bagaje de experiencias que, con relaci´on al conocimiento matem´atico, ellos ya poseen. Las actividades pretenden la estimulaci´on de los dos tipos de pensamiento que conforman el pensamiento integral, en el sentido en que lo define Santos L.: el pensamiento convergente, mediante el cual se establecen procedimientos y f´ormulas ya establecidas, y el pensamiento divergente, mediante el que se estimula la propuesta y el examen de situaciones abiertas y la animaci´on a la exploraci´on de nuevas posibilidades de pensar las diversas situaciones problema. Las actividades did´acticas dise˜ nadas proponen ambientes de discusi´on, confrontaci´on, concertaci´on y di´alogo, dando posibilidades al estudiante para que exprese sus opiniones, construya hip´otesis y las confronte con las de sus compa˜ neros, las modifique y eval´ ue las propuestas de los dem´as. Teniendo en cuenta, adem´as, que el conocimiento es un proceso de car´acter social, admitimos que “requiere de la estimulaci´on de una actitud cr´ıtica y de disposici´on al di´alogo, el aprendizaje como un proceso de construcci´on colectiva de acuerdos sobre explicaciones posibles, procedimientos y an´alisis de resultados, que, adem´as de las competencias de car´acter l´ogico y t´ecnico, requiere actitudes en relaci´on con la interacci´on social que ampl´ıen las posibilidades de expresi´on, argumentaci´on y construcci´on de consensos”(Hern´andez C., UN). Este libro de actividades, presenta problemas que contribuyen a desarrollar en el estudiante las habilidades para resolver situaciones de conteo y su capacidad 6

´ INTRODUCCION

de pensamiento l´ogico. Estas actividades se dise˜ naron de manera que representen para el estudiante un desaf´ıo intelectual, con el objetivo de lograr una modificaci´on de las estructuras cognitivas previas y no simplemente una manipulaci´on mec´anica de n´ umeros y sus operaciones, o que se puedan resolver simplemente modificando los datos. Para la elaboraci´on de las situaciones propuestas se tuvo en cuenta el car´acter significativo de ´estas; es decir, se plantearon situaciones que “encuadraran en contextos o circunstancias que fueran familiares, atractivas y motivantes”(Waldegg G., 1988) para el nivel intelectual y cronol´ogico de los estudiantes de primer semestre2. El primer cap´ıtulo muestra la construcci´on de un sistema num´erico3 hecho por los estudiantes ante el problema de contar los dedos de una persona normal, sin utilizar los d´ıgitos, ni el sistema decimal conocido por todos. Se invit´o a los estudiantes a construir un sistema num´erico propio, del que se esperaba no s´olo el desarrollo de la creatividad y con ella la habilidad para detectar las dificultades y ´exitos en los m´etodos que utilizan los matem´aticos para hacer su trabajo, sino tambi´en la vivencia de los problemas que surgen en el proceso de construcci´on y en el planteo de soluciones no siempre correctas. Contrariamente a la concepci´on usual de que cada problema tiene una soluci´on correcta y esa es la que debe aprenderse, ya que lo dem´as se considera como error, planteamos la posibilidad de proponer soluciones, algunas mejores que otras, casi todas susceptibles de ser mejoradas. Logramos deshacer mitos, como el creer que los n´ umeros son s´ımbolos a los que estamos acostumbrados por nuestro aprendizaje en la escuela, o que no es posible un sistema num´erico sin el n´ umero cero. En la primera parte de esta actividad, que consiste en la proposici´on de un conjunto de s´ımbolos y reglas de escritura que nos sirvan para contar cantidades relativamente peque˜ nas, enfatizamos en los procesos de simbolizaci´on, 2

Algunas de estas actividades tambi´en han sido desarrolladas con estudiantes de secundaria, adaptando el lenguaje y los niveles de dificultad. 3 Al repetir la experiencia en otros cursos de primer semestre y de estudiantes de secundaria, los sistemas propuestos son esencialmente los mismos, salvo en los s´ımbolos escogidos y el n´ umero de ellos.

7

´ ´ ACTIVIDADES MATEMATICAS PARA EL DESARROLLO DE PROCESOS LOGICOS

codificaci´on y decodificaci´on, cr´ıtica y argumentaci´on de sistemas propuestos, consecuci´on y respeto de acuerdos. La segunda parte la dedicamos a la proposici´on, discusi´on y selecci´on de algoritmos para efectuar las operaciones b´asicas, teniendo como criterios fundamentales la comprensi´on, la sencillez y la eficiencia. En la tercera parte, nos ocupamos de hacer una analog´ıa entre la suma y la multiplicaci´on para obtener la potenciaci´on, resaltando el modo de crecimiento de los resultados, lo que nos obliga a replantear los sistemas propuestos para escribir n´ umeros grandes. En la u ´ltima parte, del primer cap´ıtulo, buscamos que los estudiantes propongan alternativas de soluci´on al problema de la representaci´on de los n´ umeros grandes, sin incluir nuevos s´ımbolos y enfatizando en el establecimiento de regularidades b´asicas que permitan simplificar los procedimientos en las operaciones. En el segundo cap´ıtulo mostramos las soluciones4que los estudiantes le dieron al problema de escribir n´ umeros grandes con el sistema creado en la actividad anterior. Inicialmente, en este cap´ıtulo, conducimos, mediante preguntas, a los estudiantes hacia la construcci´on de sistemas posicionales con base en la exploraci´on de las propuestas que se realizaron en la actividad anterior. Seguidamente, estimulamos la propuesta de algoritmos para realizar las operaciones elementales en cualquier base num´erica. Animamos a los estudiantes a la discusi´on, revisi´on y validaci´on de algoritmos formales y de algunos algoritmos no convencionales. A continuaci´on, invitamos, a los estudiantes, a establecer las propiedades de las operaciones elementales, que se cumplen independientemente de una base particular. Luego, proponemos la tarea de traducir expresiones num´ericas de una base a otra, primero haciendo tr´ansito por la base diez y luego sin hacer uso de ella. 4

Las soluciones multiplicativas y otras que no resisten la cr´ıtica, por conducir a sistemas muy dif´ıciles de manejar, son pronto desechadas por los estudiantes. De nuevo las soluciones viables var´ıan, en otros grupos donde se plante´ o el experimento, en el n´ umero de elementos que se toman como base.

8

´ INTRODUCCION

Aunque vemos la conveniencia de los sistemas posicionales sobre los no posicionales, hacemos ´enfasis en el manejo de los c´alculos aritm´eticos en cualquier base, lo que facilita una iniciaci´on al a´lgebra al observar resultados que son v´alidos en cualquier base, esto es, independientes de su representaci´on y que tan s´olo dependen de las relaciones intr´ınsecas de los n´ umeros. En el tercer cap´ıtulo mostramos las consideraciones de los estudiantes sobre las operaciones superiores de la Aritm´etica; en la primera parte de esta actividad ellos reconocen la potenciaci´on como otra operaci´on definible en los n´ umeros naturales y estudian sus propiedades. Adem´as, pretendemos que los estudiantes, usando los conocimientos que ya poseen, identifiquen la radicaci´on y la logaritmaci´on como formas de representaci´on equivalentes a la potenciaci´on. En la segunda parte, del tercer cap´ıtulo, buscamos inferir las propiedades de la potenciaci´on de la observaci´on de ejemplos, para luego advertir las caracter´ısticas que dichas propiedades adquieren en la radicaci´on y la logaritmaci´on. Finalmente, invitamos a los estudiantes a demostrar las propiedades de la radicaci´on y la logaritmaci´on por medio de traducciones a la potenciaci´on. En la tercera parte, utilizamos la estrategia de descomposici´on en factores primos como recurso para resolver problemas de radicaci´on. En el desarrollo de la secci´on, los estudiantes deben descubrir, mediante la observaci´on de listas, criterios de divisibilidad por dos y por tres en cualquier base. En la cuarta parte, de este cap´ıtulo, ilustramos la interconexi´on del razonamiento geom´etrico y el algebraico en la resoluci´on de ecuaciones cuadr´aticas, a la manera que lo expusieron los matem´aticos griegos; inicialmente proponemos ejemplos y luego invitamos a los estudiantes a copiar el m´etodo en ecuaciones que tengan formas similares. En la quinta parte invitamos al estudiante a desarrollar su capacidad de estimaci´on del cardinal de un conjunto, en casos en que el cardinal es muy grande y es necesario expresarlo en t´erminos de potencias. En el cap´ıtulo cuatro estudiamos soluciones que algunas culturas antiguas, maya, inca, babilonia y egipcia, le dieron al problema de contar para resaltar sus ventajas y desventajas en los contextos hist´oricos en que fueron creados, y para comparalas con el sistema desarrollado por los estudiantes durante el desarrollo de la primera actividad. 9

´ ´ ACTIVIDADES MATEMATICAS PARA EL DESARROLLO DE PROCESOS LOGICOS

Inicialmente, en este cap´ıtulo, pretendemos que los estudiantes despu´es de estudiar el sistema num´erico de los Mayas caractericen las dificultades para realizar operaciones en ese sistema y propongan soluciones a ellas. Luego, despu´es de estudiar el sistema inca, se anima los estudiantes a la construcci´on y utilizaci´on de instrumentos de c´alculo, como a´bacos y yupanas, en bases distintas a la base diez, y a hacer propuestas sobre las maneras de operar empleando estos instrumentos. Seguidamente, se estimula a los estudiantes a producir explicaciones de las diversas maneras de operar en las culturas babilonia y egipcia, buscando que apliquen los algoritmos para hacer c´alculos, estudien sus dificultades, sus aciertos y sus posibilidades did´acticas. El quinto cap´ıtulo, lo dedicamos a presentar situaciones y problemas relacionados con otras ramas de la matem´atica, como la geometr´ıa, la teor´ıa de n´ umeros, la topolog´ıa, la combinatoria y la teor´ıa de conjuntos. Mediante el planteamiento y soluci´on de problemas, observamos c´omo el proceso de contar est´a inmerso en casi todas las ramas de las matem´aticas, desde maneras elementales hasta formas muy sutiles. En todas las situaciones presentamos varios niveles de complejidad, para desarrollar diferentes niveles de desempe˜ no en los alumnos; inicialmente, examinamos las situaciones en casos sencillos por conteo directo, luego invitamos a los estudiantes a extender el an´alisis para casos en que se requieren manipular cantidades m´as grandes, que implican procesos extensos y dispendiosos, lo cual exige la b´ usqueda de regularidades para simplificar procesos; posteriormente, estimulamos a los estudiantes a explicitar las regularidades en una secuencia que permita resolver el problema y, por u ´ltimo, invitamos a construir y proponer una generalizaci´on del problema en cuesti´on y una manera para resolver tal situaci´on. En el cap´ıtulo sexto, abordamos el problema de encontrar regularidades en secuencias num´ericas. Partiendo de un conjunto finito de observaciones, los estudiantes deben proponer f´ormulas y describir el t´ermino general de ellas. Inicialmente, presentamos situaciones geom´etricas que son intuitivamente m´as cercanas, luego pasamos a la observaci´on y an´alisis de secuencias num´ericas m´as abstractas, utilizando para ello conceptos sencillos de la teor´ıa de n´ umeros. 10

´ INTRODUCCION

Utilizamos como herramientas: gr´aficos, listas y propiedades elementales de los n´ umeros para ejercitar la inducci´on intuitiva, sin hacer un desarrollo lineal de los contenidos matem´aticos involucrados, a los cuales nos tienen tan acostumbrados la mayor parte de los libros de texto. Pretendemos conseguir la identificaci´on de la idea de sucesi´on como la idea fundamental en el concepto de n´ umero natural. En la primera parte del u ´ltimo cap´ıtulo, pretendemos que los estudiantes eval´ uen y critiquen las f´ormulas obtenidas en la actividad anterior, mostrando con contraejemplos el peligro de generalizar f´ormulas que se verifican en un n´ umero finito de casos, pero no en todos. Luego, tratamos el m´etodo de demostraci´on por inducci´on matem´atica como recurso para decidir la validez de una proposici´on que incluya en su enunciado el conjunto de los n´ umeros naturales. Mostramos c´omo el proceso para garantizar la validez de las afirmaciones antes intuidas, se encuentra separado del proceso de validaci´on. Utilizamos el m´etodo para establecer la veracidad de las f´ormulas generales alcanzadas a trav´es del curso. Finalmente, estudiamos el m´etodo de definici´on por recurrencia y la presentaci´on axiom´atica de los n´ umeros naturales de Peano y hacemos demostraciones de sus propiedades fundamentales. Estamos seguros de que el ´exito en el aprendizaje de alg´ un aspecto de las matem´aticas y el provecho que cada persona pueda sacar de este trabajo depende de la seriedad con que asuma el compromiso de estudiar. Se puede hacer una lectura ligera y superficial; pero, si quiere entrar en el mundo de las matem´aticas, debe dedic´arsele tiempo y ganas. Los autores

11

´ ´ ACTIVIDADES MATEMATICAS PARA EL DESARROLLO DE PROCESOS LOGICOS

12

CAP´ITULO 1 ´ UN SISTEMA DE NUMEROS PARA CONTAR Introducci´ on Los seres humanos1 en sus distintas culturas y en la medida en que sus necesidades e intereses lo han exigido, han resuelto su problema de contar, invent´andose su propio sistema de n´ umeros. Nuestro primer inter´es no radica en aprender las soluciones que se han dado a este problema, sino m´as bien, desde un punto de vista pedag´ogico, queremos ver c´omo se construye un sistema num´erico; estamos interesados en el proceso matem´atico de construir. Queremos estudiar la forma de construir un sistema de n´ umeros para contar2. Para esto inventaremos s´ımbolos y reglas, haremos acuerdos, encontraremos problemas para los cuales propondremos soluciones, descubriremos procedimientos mentales que nos permitan hacer cuentas de manera m´as r´apida y m´as eficiente, mostrando c´omo es posible evitar el trabajo f´ısico directo para sustituirlo por el trabajo mental (c´alculo), y a su vez ver c´omo el c´alculo permite predecir resultados verificables con la experimentaci´on. 1

En el art´ıculo “La capacidad de los p´ ajaros para contar”, de O. Koehler, y en el de “Contar”, de Levi Leonard Conant, que aparecen en el volumen 4 de la enciclopedia Sigma, el mundo de las Matem´ aticas, Editorial Grijalbo, Barcelona, 1985, se discute la posibilidad de que algunos animales tengan tambi´en las habilidades necesarias para contar. 2 Los n´ umeros que se utilizan para contar, se conocen como n´ umeros naturales.

13

´ ´ ACTIVIDADES MATEMATICAS PARA EL DESARROLLO DE PROCESOS LOGICOS

Luego, cuando sea posible, contaremos en reversa a partir de un cierto n´ umero (la resta); contaremos por grupos de 2, de 3, de 5, etc. (la multiplicaci´on); desharemos este proceso cuando sea posible, pregunt´andonos, por ejemplo, cu´antos grupos de 5 se pueden formar con 36, cu´antos de 6, etc. (la divisi´on). En el paso siguiente y haciendo una analog´ıa con el producto, donde se adiciona varias veces la misma cantidad, multiplicaremos varias veces la misma cantidad en un proceso mental que nos permite obtener una nueva operaci´on (la potenciaci´on), la cual ser´a de mucha ayuda para resolver el problema de escribir n´ umeros grandes de manera m´as simple. Para desarrollar estas ideas, no usaremos los d´ıgitos conocidos 0, 1, 2, 3, 4, 5, 6, 7, 8, 9 con lo que nos obligamos a inventaremos nuevos s´ımbolos para representar nuestros n´ umeros, aceptamos si embargo, copiarnos de procesos conocidos. Resaltamos que este trabajo es el producto de la continua observaci´on e interacci´on de profesores y estudiantes en aulas de clase de colegios de secundaria y primeros semestres de la universidad, en donde la intenci´on no es ense˜ nar un sistema de numeraci´on, sino con base en preguntas, producir situaciones que lleven al estudiante a proponer soluciones a problemas de conteo. M´etodos y resultados han surgido luego de ensayos y errores y no nos hemos preocupado mucho por saber si lo que descubrimos en clase ya fue inventado en otra parte; en su lugar, nos dedicamos al libre ejercicio de pensar y disfrutarlo. Invitamos al lector, con insistencia, a elaborar sus propios caminos, a plantear sus propias soluciones y, sobre todo, a formular sus propias preguntas.

1.1.

S´ımbolos y reglas del sistema

Empecemos planteando un problema simple: supongamos que queremos contar el n´ umero de dedos que tiene una persona normal, con la condici´on de no utilizar los s´ımbolos usuales. Para ello hacemos3 una representaci´ on del n´ umero pintando en un papel un s´ımbolo por cada dedo, digamos, una raya vertical, horizontal; puede no ser raya, 3

Lo que presentamos aqu´ı es el resultado de las discusiones de estudiantes de bachillerato y primer semestre de la Universidad en la carrera de Licenciatura en Matem´aticas. Salvo por los s´ımbolos elegidos, los resultados son similares.

14

´ UN SISTEMA DE NUMEROS PARA CONTAR

puede ser una flor, o el auto de papi (para los gomelos), o lo que sea. Ojal´a sea f´acil de pintar, para no detenernos mucho en un asunto que no es fundamental ´ para lo que queremos hacer. Esta es s´olo una manera de hacerlo, cada cual puede inventar sus propios s´ımbolos y sus propias reglas para representar un n´ umero. Enfatizamos que el n´ umero es u ´nico, pero sus representaciones pueden ser diversas. El resultado inicial se ver´a: IIIIIIIIIIIIIIIIIIII Esta representaci´on significa que el n´ umero de dedos de una persona normal es el mismo que el de rayas en el papel. Muchas veces nos referiremos al n´ umero como su representaci´on, pero estos conceptos no deben confundirse4 . Estamos iniciando y ya empezaron los problemas: 1. La representaci´on elegida es dif´ıcil de leer. 2. Si el n´ umero de cosas que queremos contar es grande se complica a´ un m´as la escritura y la lectura. Una soluci´on al primer problema es disponer las rayas de manera que sea m´as f´acil leerlas; por ejemplo, sustituyendo: IIIII por , con lo cual nuestra cuenta queda    . ¿Tiene usted otras soluciones? ❆ ❆ ❆ ❆ ❆ ❆ ❆ ❆ ❆ ❆ ❆ ❆ ❆ ❆ ❆ ❆ ❆ ❆ Es muy importante que su papel, amable lector, sea activo, que la lectura sea s´olo un motivo de reflexi´on y de construcci´on de su propio proceso, que toda afirmaci´on sea para usted una pregunta. ❆ ❆ ❆ ❆ ❆ ❆ ❆ ❆ ❆ ❆ ❆ ❆ ❆ ❆ ❆ ❆ ❆ ❆ Sin embargo, pintar un rect´angulo es complicado y si tenemos prisa, seguramente no nos queda exactamente rect´angulo y podr´ıa parecer un c´ırculo; para evitar calamidades y confusiones mejor convengamos sustituir I I I I I por O y para simplificar de una sola vez convengamos sustituir OOOO por X, as´ı el 4

Un n´ umero es un concepto abstracto; un s´ımbolo num´erico es una forma de representar el n´ umero.

15

´ ´ ACTIVIDADES MATEMATICAS PARA EL DESARROLLO DE PROCESOS LOGICOS

n´ umero de dedos de una persona normal la podemos escribir en forma muy sint´etica como X. Ya podemos contar, no s´olo nuestros propios dedos, sino los de una cantidad razonable de personas. Nota: la representaci´on que se utiliz´o aqu´ı es arbitraria y usted puede elegir la suya. Los mayas5 , los babilonios6 , los egipcios7, los chinos y en general, cada cultura invent´o sus propios s´ımbolos y sus reglas de c´alculo; un ejercicio interesante consiste en hacer un estudio comparativo de estos sistemas, incluyendo, por supuesto, el suyo, con esto podr´a comprobar que algunas ideas que se le ocurrieron a usted, ya se le hab´ıan ocurrido a otros grandes pensadores de la antig¨ uedad; si lo mira bien esto puede aumentar su autoestima. En el cap´ıtulo 4 daremos algunas ideas para este estudio, pero es deseable que usted lo inicie por su cuenta. Ejemplo El n´ umero de d´ıas del a˜ no en el calendario8 juliano9 y en el calendario maya10 5

CHURCHILL, E. M., Contando y midiendo, Uteha, 1965, p. 30. KLEIN, M., El pensamiento matem´ atico de la antig¨ uedad a nuestros d´ıas, Alianza Universidad, 1994, p. 18-34. 7 NEWMAN, J., Sigma, El mundo de las Matem´ aticas Vol 1 , Grijalbo, 1994, p. 95 8 Si convenimos que: un a˜ no es el tiempo que dura la tierra en dar una vuelta alrededor del sol, un mes es el ciclo completo de las fases de la luna y un d´ıa es el tiempo de una vuelta de la tierra alrededor de s´ı misma, un calendario es una distribuci´ on del a˜ no en meses y d´ıas. Lamentablemente, no existe una correspondencia exacta entre los tres movimientos, es decir que un a˜ no ¡no son 12 meses! ¡ni 365 d´ıas!, el d´ıa no tiene una duraci´ on fija! (¡tantas cosas que uno se cree! ); por lo tanto, un calendario es lo que uno quiera (como casi todo ). 9 En el a˜ no 708 del calendario romano, el emperador Julio C´esar intent´ o poner de acuerdo, con la ayuda del astr´onomo Sos´ıgenes, el curso del Sol con la duraci´ on de los a˜ nos en su calendario, agregando un d´ıa cada cuatro a˜ nos (a˜ nos bisiestos ) pero este calendario ten´ıa una inexactitud de 11 minutos 14 segundos por a˜ no. En 1582 el papa Gregorio XIII decret´ o que el 5 de octubre se volviera 15 de octubre y adem´as el primer a˜ no de cada siglo fuera bisiesto en tres siglos de cada cuatro; 1700,1800,1900 no han sido bisiestos a pesar de ser divisibles por 4, pero el 2000 lo es ! 10 Existen dos tipos de calendarios mayas. El tzolk´ın y el haab. El primero era sagrado y constaba de 260 d´ıas (kines), pero como no coinciden con la translaci´ on terrestre alrededor del Sol, no era u ´til como una gu´ıa de actividades agr´ıcolas. El a˜ no civil (haab) se compon´ıa de 18 meses de 20 d´ıas cada uno y cinco d´ıas adicionales (uayeb), al final del a˜ no considerados nefastos y en ellos se guardaba ayuno y se hac´ıan sacrificios de sangre. 6

16

´ UN SISTEMA DE NUMEROS PARA CONTAR

es: XXXXXXXXXXXXXXXXXXO Podr´ıamos argumentar que existen otras posibilidades para escribir el mismo n´ umero, con los mismos signos pero de apariencia distinta, por ejemplo: XXXXXXXXXXOXXXXXXXX O algo m´as ex´otico: XXXXXXXXXXXXXXO X X X X Esto nos obliga a poner reglas, para que cada n´ umero tenga s´olo una manera de escribirse. Por ejemplo: 1. Escribir de izquierda a derecha (esto por seguir nuestras costumbres11). 2. Escribir primero las X luego las O y finalmente los I. 3. Escribir la representaci´on del n´ umero con el m´ınimo de s´ımbolos posibles. Si pensamos en escribir n´ umeros un poco m´as grandes, podr´ıa ser u ´til, inventarnos m´as s´ımbolos, por ejemplo reemplazar XXXXXXX por L o algo similar. Pero esto no es soluci´on, porque para n´ umeros a´ un m´as grandes tendr´ıamos que seguir aumentando el n´ umero de s´ımbolos, en un proceso sin fin. Esta es la raz´on de dejar s´olo tres s´ımbolos en nuestro sistema. El problema de escribir n´ umeros grandes, requiere mayor esfuerzo de parte nuestra, por lo que dejaremos esta tarea para m´as adelante. Ejercicio Invente sus propios s´ımbolos, elija sus propias reglas y construya un sistema de n´ umeros para contar, luego proponga soluciones para el problema de escribir n´ umeros grandes. 11

Los antiguos egipcios, en su sistema de jerogl´ıficos, le´ıan los n´ umeros de derecha a izquierda. En el idioma alem´ an los n´ umeros de dos cifras se escriben de izquierda a derecha pero se leen de derecha a izquierda.

17

´ ´ ACTIVIDADES MATEMATICAS PARA EL DESARROLLO DE PROCESOS LOGICOS

❆ ❆ ❆ ❆ ❆ ❆ ❆ ❆ ❆ ❆ ❆ ❆ ❆ ❆ ❆ ❆ ❆ ❆ No espere que siempre le pongan ejercicios, haga sus propias hip´otesis, intente caminar solo desde el principio; es posible que se caiga a veces, pero consu´elese, eso nos pasa a todos. ❆ ❆ ❆ ❆ ❆ ❆ ❆ ❆ ❆ ❆ ❆ ❆ ❆ ❆ ❆ ❆ ❆ ❆

1.2.

Operaciones

Una de las utilidades m´as grandes de un sistema de n´ umeros es la posibilidad de calcular, esto es, operar dos o m´as n´ umeros para obtener otro mediante procedimientos mentales, sin tener que recurrir a la experiencia directa12. Por ejemplo, si queremos conocer el n´ umero de elementos que resulta de reunir dos colecciones de objetos, de cada una de las cuales ya conocemos su n´ umero, no es necesario hacer de nuevo el conteo de la nueva colecci´on; podemos efectuar una operaci´on y con ella obtener el resultado. La m´as simple de las operaciones aritm´eticas es la adici´on

1.2.1.

Adici´ on

Para sumar dos cantidades cualesquiera, podemos colocar los s´ımbolos que representan cada una de las cantidades, una a continuaci´on de otra y reescribir el resultado de acuerdo a nuestras reglas13 ; a este procedimiento lo llamaremos umeros que adici´on 14 , al resultado de la operaci´on lo llamaremos suma y a los n´ se adicionan los llamaremos sumandos. Ejemplo Si tenemos los n´ umeros XOOOIII y XOOII podemos adicionarlos de la siguiente manera: 12

No todas las culturas que han desarrollado s´ımbolos num´ericos, los han utilizado para calcular con ellos, las culturas que lo hicieron, desarrollaron conocimientos matem´ aticos m´as avanzados, como los babilonios, los egipcios y los griegos. 13 Esta manera de adicionar es la misma que usaron los babilonios para adicionar n´ umeros menores de 60 y tambi´en la usaron los egipcios. 14 Los incas, una de las m´ as avanzadas culturas americanas, practicaban la adici´ on en tablillas llamadas Yupanas donde se colocaban granos o piedras y luego las registraban haciendo nudos en unas cuerdas de vivos colores que iban juntando en lo que llamaban quipu.

18

´ UN SISTEMA DE NUMEROS PARA CONTAR

1. Coloquemos uno a continuaci´on del otro: XOOOIIIXOOII 2. Escribamos los s´ımbolos en el orden acordado: XXOOOOOIIIII 3. Reduzcamos la cantidad de s´ımbolos: XXXOO. Luego de hacer varias adiciones nos vamos dando cuenta de ciertas cosas que pasan en todas ellas; de ciertas reglas generales que van surgiendo de la experimentaci´on y la observaci´on, por ejemplo: 1. La suma no depende del orden de los sumandos. 2. El resultado de contar un conjunto dado de objetos no depende del orden en que se cuenten. Ejercicio Encuentre otras regularidades comunes a todas las sumas.

1.2.2.

Sustracci´ on15

En muchas situaciones es necesario quitar una cantidad de otra y este procedimiento tambi´en puede llevarse a cabo sin necesidad de recurrir a los objetos que las cantidades representan. Un procedimiento que nos permita realizar esta tarea lo llamaremos Sustracci´ on. Si tenemos un n´ umero que llamamos minuendo y le queremos sustraer otro n´ umero que llamamos sustraendo, utilizamos nuestros s´ımbolos y reglas para encontrar el resultado que llamamos diferencia o resta, de la siguiente forma: 1. Eliminamos los s´ımbolos que se encuentren comunes en el minuendo y el sustraendo, hasta que no queden s´ımbolos en el sustraendo. (Es necesario que el n´ umero que representa el minuendo sea mayor que el del sustraendo; a diferencia de la adici´on, los n´ umeros que participan en la sustracci´on juegan un papel diferente). 15

En rigor, la sustracci´ on no es una operaci´ on entre n´ umeros naturales, pues no todas las sustracciones se pueden efectuar, s´ olo es posible si el minuendo es mayor o igual que el sustraendo, pero para los prop´ ositos de este trabajo, esta limitaci´on no la consideramos fundamental por el momento.

19

´ ´ ACTIVIDADES MATEMATICAS PARA EL DESARROLLO DE PROCESOS LOGICOS

2. En el caso en que no hayan s´ımbolos comunes, se sustituye en el minuendo X ´o O por su equivalencia y se procede como en el numeral 1. 3. Se aplican las reglas de escritura a los s´ımbolos que quedan y este es el resultado. Ejercicio Si se quiere sustraer (tambi´en se dice restar) del n´ umero XXXOIII el n´ umero XOOI i. Eliminamos, uno a uno, los s´ımbolos que se encuentran comunes en el minuendo y el sustraendo. El resultado de este primer paso es: XXII en el minuendo y O en el sustraendo. ii. Como a´ un tenemos s´ımbolos en el sustraendo, sustituimos una X del minuendo por OOOO, quedando XOOOOII en el minuendo y O en el sustraendo. iii. Repetimos el paso 1. y tenemos como resultado final: XOOOII. Ejercicio ¿Encuentra algunas regularidades que se cumplan para todas las sustracciones? Enum´erelas, comp´ arelas con las observadas para la adici´ on.

1.2.3.

Multiplicaci´ on

Cuando se requiere sumar varias veces la misma cantidad, es natural intentar simplificar el proceso de adicionar, en un proceso que nos evite hacer varias veces lo mismo. Este procedimiento, en el caso de que exista, lo llamaremos multiplicaci´ on; el resultado de una multiplicaci´on lo llamamos producto, los n´ umeros que se multiplican los llamamos factores. Como el prop´osito de ahora es contar por grupos que tengan la misma cantidad, lo que equivale a hacer varias adiciones de la misma cantidad; iniciaremos la discusi´on por lo m´as sencillo (este truco funciona casi siempre). Empecemos por multiplicar los s´ımbolos b´asicos de nuestro sistema, por ejemplo: 20

´ UN SISTEMA DE NUMEROS PARA CONTAR

1. La multiplicaci´on de O y I, consiste en colocar O veces I y sumar16, o sea, IIIII Esto es, repetir el s´ımbolo I tantas veces como est´e I en el primer t´ermino; el resultado es IIIII, o lo que es lo mismo, seg´ un nuestras reglas, O. En resumen O veces I es O. 2.

O veces O significa colocar IIIII veces O o sea, repetir el s´ımbolo O tantas veces como est´e I en el primer t´ermino y sumar, esto es OOOOO, por lo tanto

O veces O es OOOOO.

o sea,

XO

Razonando en forma parecida obtenemos que O veces X es XXXXX. Similarmente conseguimos los resultados siguientes: X veces I es X X veces O es XXXXX. X veces X es XXXXXXXXXXXXXXXXXXXX. Es claro que para sumar o para multiplicar se requieren por lo menos dos n´ umeros; sin embargo, muchas veces resulta u ´til hacer convenios que nos permiten escribir de manera unificada un conjunto de reglas o procedimientos sin tener que enunciar cada vez una lista de excepciones. Este es el caso para que convengamos, que aunque no estamos sumando, digamos que I vez I debe ser I. I vez O es O I vez X es X. 16

En las escuelas primaria y secundaria se suele decir que la multiplicaci´on de O y I, consiste en sumar O veces I, pero esta interpretaci´ on es incorrecta pues en ella s´olo hay IIII sumas; para efectuar una suma se necesitan dos n´ umeros.

21

´ ´ ACTIVIDADES MATEMATICAS PARA EL DESARROLLO DE PROCESOS LOGICOS

Todos estos resultados los podemos poner juntos en una tabla, que se conoce como una tabla de multiplicar. •

I

O

X

I

I

O

X

O

O

XO

XXXXX

X

X

XXXXX

XXXXXXXXXXXXXXXXXXXX

Antes de ponernos a hacer cuentas m´as complicadas, deteng´amonos un momento a pensar (este es otro viejo truco de los matem´ aticos); miremos qu´e caracter´ısticas tiene nuestro sistema, para ver si podemos hacer las cuentas m´as f´acil 1. Para simplificar la notaci´on, reemplazaremos la palabra “veces”por un s´ımbolo m´as sencillo de escribir, por ejemplo un punto • o lo que usted quiera, siempre y cuando no se le olvide y despu´es use algunas veces un punto y otras, una raya u otra cosa; una condici´on que exigen los s´ımbolos, es que una vez que usted los acepte, deben tener siempre el mismo significado, mejor dicho, seamos serios con los s´ımbolos. 2. Para multiplicar dos n´ umeros, los escribimos uno a continuaci´on del otro colocando el s´ımbolo acordado entre ellos. Es conveniente acordar que si alguno de los factores est´a conformado por m´as de un s´ımbolo ´este ser´a escrito entre par´entesis17 dado que se puede prestar para confusi´on si no lo hacemos. Por ejemplo, si escribimos X • OI, es posible interpretar esta multiplicaci´on como X • (OI) o como (X • O) I, de lo cual obtenemos resultados diferentes. 3. Al efectuar multiplicaciones notamos que: a) No importa el orden en que se multipliquen dos n´ umeros cualesquiera 18 el producto de ellos es el mismo . A este comportamiento se le conoce 17

Los par´entesis en esta secci´on s´olo indican que lo que est´ a entre ellos debe considerarse como un u ´ nico n´ umero. 18 Usaremos dos l´ıneas de la misma longitud ( = ) para indicar la igualdad de acuerdo con la costumbre; este signo fue usado por primera vez por Robert Record en Inglaterra en 1557.

22

´ UN SISTEMA DE NUMEROS PARA CONTAR

como propiedad conmutativa de la multiplicaci´on. O•I=I•O X•I=I•X O•X=X•O b) Cualquier n´ umero multiplicado con I da como resultado el mismo n´ umero; es decir, que I tiene un comportamiento especial en nuestro sistema; por esto a I se le conoce como elemento id´entico, elemento neutro o m´ odulo de la multiplicaci´on. c) Multiplicar un n´ umero por otro, equivale a escribir uno de ellos tantas veces como I est´e en la representaci´on del otro, es decir repetirlo una vez por cada I . d ) Para multiplicar cantidades que tengan varios s´ımbolos, multiplicamos cada s´ımbolo del primer factor por cada uno de los del segundo factor y luego sumamos los resultados. Esta propiedad se conoce como propiedad distributiva de la multiplicaci´ on respecto a la adici´ on. Estas observaciones nos ayudar´an cuando las cosas empiecen a complicarse. Por ejemplo, si queremos multiplicar (OI)•(XOII); es decir, saber el resultado de adicionar OI veces la cantidad XOII, procedemos as´ı: 1. Separamos el s´ımbolo OI en O y I y cada uno de ellos lo multiplicamos por XOII (OI)•(XOII)=(O •(XOII)) (I•(XOII)) 2.

A la derecha, reemplazamos la primera O por su equivalente en I (OI)•(XOII)=((IIIII)•(XOII)) (I•(XOII))

3.

Por cada I hacemos una copia de XOII (OI)•(XOII)=(XOII)(XOII)(XOII)(XOII)(XOII)(XOII) 23

´ ´ ACTIVIDADES MATEMATICAS PARA EL DESARROLLO DE PROCESOS LOGICOS

4.

Reescribimos el resultado de acuerdo con nuestras reglas (OI)•(XOII)=XXXXXXOOOOOOIIIIIIIIIIII (OI)•(XOII)=XXXXXXXXII.

El procedimiento descrito aqu´ı para multiplicar, por fortuna, no es el u ´nico, como sucede con casi todos los procedimientos y m´ etodos usados en matem´ aticas; otra manera de hacerlo que puede resultarnos m´as familiar por nuestros d´ıas de escuela, es la siguiente: XOII • OI XOII XXXXXXOOO XXXXXXXXII

Lo que hemos hecho es: 1. Multiplicar el I de OI por cada uno de los s´ımbolos de XOII y lo escribimos en el primer rengl´on. 2. Multiplicar O de OI por cada uno de los s´ımbolos de XOII de acuerdo con la tabla de multiplicar y lo escribimos en el segundo rengl´on. 3. Sumar los resultados. Notemos que en este proceso las posiciones no son importantes; adem´as, podemos empezar por cualquier s´ımbolo y hacerlo en cualquier orden. Un muchacho de escuela posiblemente apreciar´a esta informaci´on. Ejercicio Seguramente usted ya not´o que existen otros procesos para realizar este mismo c´alculo y est´a sacando sus propias conclusiones. Verifique si las observaciones que se cumplen para multiplicar los s´ımbolos b´ asicos tambi´en se tienen cuando un n´ umero tiene varios s´ımbolos. 24

´ UN SISTEMA DE NUMEROS PARA CONTAR

1.2.4.

Divisi´ on19

Nuestro prop´osito ahora es repartir una cantidad en grupos, de manera que todos los grupos queden con el mismo n´ umero de elementos; este proceso se conoce como hacer una divisi´on.

La cantidad a repartir la llamaremos dividendo, la cantidad de elementos de cada grupo la llamamos divisor, el n´ umero de grupos resultante cociente y lo que sobra residuo.

Copiando lo hecho para la multiplicaci´on, primero hagamos unas cuentas simples.

En los procesos de repartici´on hay dos tipos de problemas.

1. Repartir, por ejemplo, una cantidad X en O grupos con el mismo n´ umero de elementos.

2. Repartir la cantidad X en grupos de O elementos. Para resolver el primer problema, un camino es:

a) Escribir el dividendo utilizando solamente el s´ımbolo I. b) Colocar un I en cada grupo como en la figura 1. c) Repetir el paso b, hasta agotar las existencias o hasta que lo que sobre no alcance para asignar un I a cada grupo. 19

La divisi´ on, como la sustracci´on, tampoco es una operaci´on en el sentido de que no siempre es posible repartir una cantidad dada en un determinado n´ umero de grupos, sin que sobren elementos, pero nuestro inter´es se centra por ahora en los procedimientos, por eso omitiremos por ahora esta dificultad.

25

´ ´ ACTIVIDADES MATEMATICAS PARA EL DESARROLLO DE PROCESOS LOGICOS

el resultado de este proceso lo podemos resumir en la Figura 2.

Como se ve, en cada grupo hay IIII elementos. Lo que expresamos simb´olicamente as´ı: X O IIII

Ejercicio Plantee un procedimiento para resolver el segundo problema. 26

´ UN SISTEMA DE NUMEROS PARA CONTAR

❆ ❆ ❆ ❆ ❆ ❆ ❆ ❆ ❆ ❆ ❆ ❆ ❆ ❆ ❆ ❆ ❆ ❆ Si su l´apiz, atento lector, ha estado activo, proponerle m´as ejercicios de nuestra parte no se hace necesario. ❆ ❆ ❆ ❆ ❆ ❆ ❆ ❆ ❆ ❆ ❆ ❆ ❆ ❆ ❆ ❆ ❆ ❆ Notemos que estos procesos son poco pr´acticos cuando las cantidades son grandes, esto hace que sea necesario buscar procedimientos (tambi´en llamados algoritmos) que nos faciliten resolver el problema m´as eficientemente. Recurramos entonces a un ejemplo, si queremos dividir XXOOIII entre XO, el dividendo puede representarse en la forma XO XO III, es decir: XXOOIII= XO XO III se observa que el divisor “cabe”II veces en el dividendo y que el residuo es III. Tambi´en podr´ıamos adaptar los procedimientos que aprendimos en la escuela20 para ver si ´estos funcionan tambi´en en nuestro sistema num´erico. Continuando con nuestro ejemplo, divididamos XXOOIII entre XO copiando el procedimiento que aprendimos en la escuela para hacer divisiones, as´ı: 1. Tomamos el primer s´ımbolo de la izquierda del dividendo y del divisor. 2. Como el s´ımbolo del divisor es igual que el s´ımbolo del dividendo, buscamos en la tabla de multiplicar un s´ımbolo que multiplicado por X nos de X, este s´ımbolo es I y va hacer parte del cociente. 3. Multiplicamos I por el divisor y lo restamos del dividendo, XXOOIII XO XO I XOIII 4. Repetimos 1, 2 y 3 tomando como dividendo el residuo. 20

Curiosamente, esta es una de las primeras propuestas que surgen en las aulas al discutir el tema.

27

´ ´ ACTIVIDADES MATEMATICAS PARA EL DESARROLLO DE PROCESOS LOGICOS

XXOOIII XO XO II XOIII XO III El cociente de nuestra divisi´on es II y el residuo es III. Este proceso lo describimos en los siguientes pasos: 1. Separamos desde el extremo izquierdo del dividendo tantos s´ımbolos como haya en el divisor. 2. Si el divisor es menor o igual que el n´ umero formado en la separaci´on hecha en el paso anterior en el dividendo; es decir, si el divisor se encuentra de manera expl´ıcita o impl´ıcita en alguna representaci´on del n´ umero separa21 do , 2.1. Buscamos un n´ umero que multiplicado por el divisor quede lo m´as pr´oximo posible, pero sin que sea mayor que el n´ umero separado, este n´ umero va a ser parte del cociente. 2.2. Multiplicamos el n´ umero que encontramos por el divisor y el resultado lo restamos del n´ umero separado del dividendo. 2.3. Tomamos el resultado de esta resta (residuo) como nuevo dividendo y repetimos el proceso hasta que el residuo sea menor que el divisor. Aqu´ı termina el proceso. 3. Si el n´ umero formado en el dividendo, en la separaci´on inicial del paso 2, es menor que el divisor agregamos el s´ımbolo siguiente (si existe) en el dividendo y continuamos en el paso 2. Si no existe, el proceso termina o la divisi´on no se puede efectuar.

Ejercicio Proponga su propio procedimiento para dividir. 21

Por ejemplo I es menor que O porque I est´ a en la representaci´on IIIII de O, O es menor que X porque O est´a en la representaci´on OOOO de X.

28

´ UN SISTEMA DE NUMEROS PARA CONTAR

1.2.5.

Potenciaci´ on

En esta secci´on vamos a usar un proceso mental, muy utilizado en la construcci´on de nuevas teor´ıas matem´aticas a partir de otras previamente establecidas; consiste en copiar un procedimiento o concepto, modificando alguno de sus componentes. Vimos c´omo la multiplicaci´on se puede construir con la ayuda de la adici´on repitiendo un sumando alg´ un n´ umero de veces; an´alogamente podemos repetir un factor para hacer una multiplicaci´on de ´el por s´ı mismo un n´ umero de veces y con ello obtener otra operaci´on; esta nueva operaci´on se llama potenciaci´on. Como ´esta es una nueva operaci´on debemos escoger una manera adecuada de escribirla para no confundirnos; es decir, una notaci´ on (este tambi´ en es un paso importante cuando estamos inventando); como de costumbre la simbolog´ıa y la notaci´on podemos escogerlas tan ex´oticas como queramos, pero ser´ıa mejor que si estamos expresando ideas ya conocidas, usemos la notaci´on usual. Escribiremos OII para expresar O • O OIII = O • O • O OO = O• O • O • O • O No hay problema en escribir O • O • O puesto que de cualquier forma que realicemos los productos el resultado es el mismo; es decir, (O • O) • O = O • (O • O) donde los par´entesis indican qu´e debe hacerse primero. En general colocamos una cantidad, que llamaremos exponente, en la parte superior derecha de otra, que llamamos base, para expresar que se repite la base por s´ı misma tantas veces como lo diga el exponente y luego se multiplica. Convenimos adem´as que OI significa O; es bueno notar que en este caso no hay multiplicaciones22. 22

Como en cualquier otra operaci´ on, se necesitan dos n´ umeros para hacer una multiplicaci´ on.

29

´ ´ ACTIVIDADES MATEMATICAS PARA EL DESARROLLO DE PROCESOS LOGICOS

❆ ❆ ❆ ❆ ❆ ❆ ❆ ❆ ❆ ❆ ❆ ❆ ❆ ❆ ❆ ❆ ❆ ❆ Es hora de soltar la mano; aseg´ urese de hacer tantas operaciones como sea necesario para que el procedimiento llegue a ser autom´atico en su cabeza y dejemos la inteligencia para dedicarla a cosas mejores. ❆ ❆ ❆ ❆ ❆ ❆ ❆ ❆ ❆ ❆ ❆ ❆ ❆ ❆ ❆ ❆ ❆ ❆

1.3.

El problema de los n´ umeros grandes

Si en nuestro sistema num´erico queremos escribir un n´ umero grande, la cantidad de X que habr´ıa que escribir ser´ıa tan tediosa como representarlo s´olo con los s´ımbolos I. Hemos descartado ya la posibilidad de hacer nuevos reemplazos y agregar cada vez m´as s´ımbolos para representar un n´ umero, por cuanto esto no resuelve para nada el problema y s´ı nos llenar´ıa de una cantidad indistinguible de s´ımbolos, que dificultar´ıa su manejo para cualquier persona no acostumbrada a trabajar con ellos.

1.3.1.

Notaci´ on multiplicativa

Una propuesta de soluci´on para este problema es que utilicemos los resultados de las multiplicaciones para resumir s´ımbolos, escribiendo por ejemplo X • O en lugar de XXXXX. Por supuesto toca modificar las reglas de interpretaci´on de la escritura de los n´ umeros para no generar confusiones; si escribimos X • O debemos distinguir si tenemos que realizar la multiplicaci´on o dejarla indicada. Por ello, modifiquemos un poco las reglas de la escritura. Como nuestro problema consiste en un elevado n´ umero de X para indicar n´ umeros grandes, escribamos el n´ umero de veces que aparece la X, en lugar de escribir las X. Pero no requerimos escribir el s´ımbolo de multiplicaci´on, basta escribir el n´ umero de veces que aparece la X, en la parte inferior derecha de ella. Por ejemplo el n´ umero XXXXXXXXXXXXXXXXXXXXXXXXXXXOOOIIII 30

´ UN SISTEMA DE NUMEROS PARA CONTAR

Lo escribimos XXOII OOOIIII Notemos que esta soluci´on no introduce nuevos s´ımbolos al sistema y simplifica la escritura de n´ umeros grandes. Si representamos un n´ umero cualquiera con una letra, por ejemplo m, n, etc, podemos verificar ciertos comportamientos, por ejemplo: Xm Xn = Xmn n • Xm = Xm•n Xm • Xn = XXm•n Ejercicios 1. Justifique las f´ ormulas anteriores. 2. Estudie la propuesta de notaci´ on multiplicativa y proponga un conjunto de reglas para formar un sistema de representaci´ on de n´ umeros, con sus 23 operaciones b´ asicas. ¿C´ omo se comportan las potencias de Xn ? 3. Represente el n´ umero de d´ıas de un a˜ no en el calendario juliano en este sistema.

1.3.2.

Notaci´ on exponencial

Otra posibilidad es utilizar la misma idea anterior pero cambiando la operaci´on y en lugar de usar la multiplicaci´on usar la potenciaci´on. Esta propuesta est´a basada en la observaci´on de la manera como crecen los n´ umeros cuando reiteramos la operaci´on potenciaci´on, con respecto a las dem´as operaciones, veamos: Por ejemplo, tomemos como base X y expresemos sus primeras potencias: umero de hojas que tiene un libro como el a´lgebra XII es aproximadamente el n´ de Baldor. umero de kil´ometros del radio de la tierra. XIII es aproximadamente el n´ 23

Llamaremos potencia al n´ umero que resulta al efectuar una potenciaci´ on.

31

´ ´ ACTIVIDADES MATEMATICAS PARA EL DESARROLLO DE PROCESOS LOGICOS

XIIII es aproximadamente el n´ umero de pesos que hay en el salario m´ınimo de los colombianos para el a˜ no 1997. O umero de habitantes de una ciudad como Cali. X es aproximadamente el n´ XOI es aproximadamente el doble de la poblaci´on de toda Colombia y el n´ umero de cabellos que tiene una persona joven. XOOOII es aproximadamente el n´ umero de estrellas del universo conocido. umero de gotas de agua que hay en el mar. XXII es aproximadamente el n´ umero de a´tomos del universo. XXXOO es aproximadamente el n´ No discutimos aqu´ı la manera para calcular estos n´ umeros, por lo engorroso de la escritura, pero en principio los c´alculos son posibles en cualquier sistema num´erico; dejamos esta tarea para cuando tengamos otras representaciones m´as evolucionadas, donde su presentaci´on sea m´as c´omoda. Otra raz´on para escoger la potenciaci´on es que ella tambi´en tiene comportamientos que permiten simplificar algunos c´alculos, sobre todo cuando la base de los n´ umeros es la misma. Por ejemplo, para multiplicar XOI por XOI simplemente colocamos la misma base X y sumamos los exponentes, obteniendo como resultado XOOII ; esto se debe a que debemos repetir X, primero OI veces y luego OI veces, en total OOII veces para multiplicar, que es lo que expresa el u ´ltimo resultado. Como vemos, la notaci´on exponencial nos permite escribir n´ umeros grandes con pocos s´ımbolos, lo que sugiere utilizar esta notaci´on para modificar nuestro sistema de representaci´on de n´ umeros y obtener una manera m´as corta para escribir n´ umeros grandes; sin embargo, si escogemos todas las potencias de X y de O, no tenemos un criterio sencillo para decidir cu´al n´ umero entre XO o OX es m´as peque˜ no que el otro, para incluirlo seg´ un nuestras reglas en la escritura de un n´ umero m´as grande, adem´as existen muchos n´ umeros que no se pueden escribir de manera sencilla con esta notaci´on, s´olo son sencillos aquellos que sean potencias de los s´ımbolos fundamentales. umeros cuya escritura es disPor ejemplo, entre OO y OX hay muchos n´ pendiosa. Ejercicio Escriba el n´ umero anterior a OX . 32

´ UN SISTEMA DE NUMEROS PARA CONTAR

El problema de la escritura se debe a que X es IIII veces O, pero O es IIIII veces I ; es decir, que uno no se puede escribir como potencia del otro. Una salida a este problema ser´ıa omitir un s´ımbolo de nuestro sistema y en su lugar colocar, no una sino todas las potencias del otro; es decir, eliminemos de nuestro sistema inicial el s´ımbolo X (o el s´ımbolo O) y escribamos los n´ umeros solamente con I y O y sus potencias. As´ı, los primeros n´ umeros ser´ıan: I, II, III, IIII, O, OI, OII, OIII, . . ., OOOOIIII, OII, . . ., OIII, . . ., OIIII, . . . O , OO I, OO II, OO III, OO IIII, OO O, . . . O

Ejercicio Defina procedimientos para las operaciones elementales con la notaci´ on exponencial y reintente con la notaci´on multiplicativa; es posible que esta discusi´on le haya sugerido alguna idea. ❆ ❆ ❆ ❆ ❆ ❆ ❆ ❆ ❆ ❆ ❆ ❆ ❆ ❆ ❆ ❆ ❆ ❆ Suele suceder que luego de avanzar un poco, los problemas por los que hemos pasado se ven con una mirada diferente. ❆ ❆ ❆ ❆ ❆ ❆ ❆ ❆ ❆ ❆ ❆ ❆ ❆ ❆ ❆ ❆ ❆ ❆ A pesar de las dificultades, hemos logrado resolver el problema de contar, siempre y cuando los n´ umeros no sean demasiado grandes. Aunque el problema te´orico est´a resuelto, queda uno pr´actico por resolver: la escritura de los n´ umeros grandes tiene un aspecto muy poco est´etico, por lo que debemos abordarlo m´as adelante. Sistemas num´ericos, como el desarrollado en este cap´ıtulo, fueron inventados por diversas culturas, como los jonios y los romanos; estos sistemas tienen dificultades importantes cuando se utilizan para calcular, a diferencia del que hemos construido aqu´ı. Ejercicio

1. Proponga procedimientos para operar con n´ umeros romanos. 33

´ ´ ACTIVIDADES MATEMATICAS PARA EL DESARROLLO DE PROCESOS LOGICOS

2. Haga una analog´ıa con la operaci´ on multiplicaci´on para definir una nueva operaci´ on basada en la repetici´ on de la potenciaci´ on e investigue qu´e propiedades tiene. Le sugerimos, por ejemplo, definir: n

X = ((((X)X )X )...)X

donde n es el n´ umero de veces que repetimos la X. ¿Ser´a u ´til esta operaci´on para inventar otra manera de escribir n´ umeros grandes? La notaci´on multiplicativa se comporta bien con las adiciones, la exponencial con las multiplicaciones, ¿esta nueva operaci´on se comporta bien con la potenciaci´on? 3. La figura 3 es una reproducci´ on de una tablilla de la antigua Babilonia24 . Descifra lo que en ella se encuentra escrito.

24

AABOE, A., Matem´ aticas: Episodios hist´ oricos, desde Babilonia a Ptolomeo, Biblioteca de Matem´atica contempor´ anea, Random House, Ed. Norma, Cali, p. 21

34

´ UN SISTEMA DE NUMEROS PARA CONTAR

35

´ ´ ACTIVIDADES MATEMATICAS PARA EL DESARROLLO DE PROCESOS LOGICOS

36

CAP´ITULO 2 REPRESENTACIONES ´ POSICIONALES DE NUMEROS 2.1.

Una idea genial: usar la posici´ on

Al final del cap´ıtulo anterior, notamos que las potencias de un n´ umero dan resultados que crecen muy r´apido cuando el exponente aumenta, lo que sugiri´o utilizar la notaci´on exponencial para escribir n´ umeros grandes. La idea1 fue, escoger como s´ımbolos b´asicos I, O y sus potencias, convenir reglas para su escritura y estudiar procedimientos para hacer las operaciones que hemos definido en el cap´ıtulo anterior, esto es: I O =IIIII OII =O O O O O OIII =OII OIIOIIOII OII OIIII =OIIIOIIIOIII OIII , etc. De esta forma, no es necesario incluir s´ımbolos nuevos. 1

El primer vestigio de una soluci´ on con este criterio aparece en la cultura babil´ onica, donde se escogi´o como base el 60 para los problemas de matem´aticas y astronom´ıa y el 10 para cuestiones comerciales. Los egipcios agrupaban en base 10, los mayas en base 20, etc

37

´ ´ ACTIVIDADES MATEMATICAS PARA EL DESARROLLO DE PROCESOS LOGICOS

Si adoptamos las mismas reglas para la escritura de los n´ umeros que en el cap´ıtulo anterior y estos nuevos s´ımbolos, el n´ umero de dedos de una persona normal es O O O O; no parece que hayamos logrado un gran avance, pero si miramos n´ umeros m´as grandes empezamos a ver las ganancias. Por ejemplo, el n´ umero de d´ıas del a˜ no en el calendario juliano es: OIII OIII OII OII OII OII O O O aunque el n´ umero de s´ımbolos disminuy´o con respecto a nuestro antiguo sistema, no se ve a´ un muy atractivo. Pero si consideramos n´ umeros como: OOIII que sirve para representar aproximadamente el n´ umero de habitantes de una ciudad como Neiva, o OOOIII que es aproximadamente el n´ umero de habitantes actuales de toda China, en nuestro antiguo sistema de numeraci´on tendr´ıan una escritura mucho m´as aparatosa; por ejemplo, el primero tendr´ıa cinco mil ochocientos ochenta y dos veces la X, una O y IIII, lo que justifica el esfuerzo actual. Los n´ umeros en este nuevo sistema tienen apariencias como OIII OIII OIII OIII OII O O O O I Si los observamos un buen rato vemos que el s´ımbolo O se repite muchas veces, y como ya convenimos que representa grupos de IIIII, podr´ıamos no escribirlo y escribir solamente el n´ umero de veces que aparece cada s´ımbolo con su exponente; para no confundirnos los separamos con un espacio, o de alguna otra forma. Con estos nuevos convenios el anterior n´ umero lo escribimos: IIII

I

IIII

I

lo que significa que OIII aparece IIII veces, OII aparece I vez, O aparece IIII veces y I s´olo I vez ( ¿Esto no le recuerda los c´odigos de barras de los art´ıculos en los supermercados? ) 38

´ REPRESENTACIONES POSICIONALES DE NUMEROS

Ejercicio Averig¨ ue c´omo funcionan los c´odigos de barras y elabore un informe. Volvamos a lo nuestro; como vemos, podemos ahora omitir tambi´en la O, pero aparece un problema, ¿qu´e hacemos cuando no haya, por ejemplo, grupos de OIII pero s´ı de OII y de OIIII ? , deber´ıamos dejar un espacio entre la cantidad de veces que est´e OIIII y la cantidad de veces que est´e OII; pero escribiendo de prisa, no sabr´ıamos diferenciar si el espacio es doble o sencillo; mejor deteng´ amonos un poco a pensar. Estamos en una circunstancia similar a la inicial, pero con una gran ventaja, que hemos descubierto la posibilidad de usar la posici´on de los s´ımbolos como ayuda para escribir n´ umeros de una forma m´as breve. Tenemos ahora un solo s´ımbolo ( I ) para representar todos los n´ umeros, el cual aparece repetido en cada posici´on hasta una unidad menos que la cantidad que escogimos como base. Si queremos de nuevo simplificar la escritura, podemos introducir algunos nuevos s´ımbolos. Por ejemplo, convengamos escribir un s´ımbolo, digamos: ×  +   r  c 

para para para para para

cuando cuando cuando cuando cuando

no aparece el s´ımbolo b´asico I aparezca I vez aparezca II veces aparezca III veces aparezca IIII veces

y no m´as, porque en el siguiente paso aparece una unidad para la siguiente posici´on de la izquierda; as´ı, el n´ umero de d´ıas del calendario juliano es  c  r  × ¡y esto ya parece idioma chino! ❆ ❆ ❆ ❆ ❆ ❆ ❆ ❆ ❆ ❆ ❆ ❆ ❆ ❆ ❆ ❆ ❆ ❆ Una situaci´on frecuente en el estudio de las matem´aticas es que cuando avanzamos en un proceso de abstracci´on, podemos sustituir un conjunto de s´ımbolos o ideas por un nuevo s´ımbolo o una nueva idea. Conjuntos 39

´ ´ ACTIVIDADES MATEMATICAS PARA EL DESARROLLO DE PROCESOS LOGICOS

de estos a su vez pueden ser reemplazados dentro de una teor´ıa por otros cada vez m´as abstractos, hasta llegar a conceptos tan sofisticados que dar´ıan la impresi´on de que estamos hablando sobre nada u´til. ❆ ❆ ❆ ❆ ❆ ❆ ❆ ❆ ❆ ❆ ❆ ❆ ❆ ❆ ❆ ❆ ❆ ❆ Escogimos los s´ımbolos anteriores de manera caprichosa y deliberadamente complicados, s´olo con el prop´osito de insistir en que uno puede inventarse lo que se le ocurra; es mejor, por supuesto, que si hay s´ımbolos inventados para escribir algunas ideas ser´a mejor usarlos y dejar nuestra inventiva para cosas m´as productivas. Por eso, a cambio de: ×  +   r  c 

escribimos escribimos escribimos escribimos escribimos

0 1 2 3 4

y nuestro sistema queda con los s´ımbolos 0, 1, 2, 3, 4 . Este sistema tiene las siguientes caracter´ısticas fundamentales: 1. En ´el es importante la posici´on de los s´ımbolos2, pues 34 significa que el I aparece 4 veces y la O aparece 3 veces, pero 43 significa que I aparece 3 veces y O aparece 4 veces. Por esta raz´on se llaman representaciones posicionales de sistemas num´ericos. A cada uno de los s´ımbolos que forma un n´ umero lo llamaremos cifra 3. 2. El s´ımbolo O no aparece expl´ıcitamente en la escritura de los n´ umeros pero est´a impl´ıcito; se llama la base del sistema y lo podemos escoger de manera arbitraria para reemplazar tantos I como deseemos. 2

Los babilonios escrib´ıan los n´ umeros en grupos separados por espacios. Su escritura se hac´ıa de derecha a izquierda; cada grupo representaba respectivamente unidades, grupos de sesenta y grupos de tres mil seiscientos. Para hallar el n´ umero representado se efectuaban las multiplicaciones correspondientes y se adicionaban los productos resultantes. Para representar el cero al principio dejaban un espacio y luego lo hicieron con cu˜ nas oblicuas. Los mayas escrib´ıan los n´ umeros en columnas de abajo hacia arriba. El grupo inferior representaba las unidades, el siguiente grupo el n´ umero de agrupaciones de veinte y el tercer grupo, el n´ umero de agrupaciones de trescientas sesenta. Ten´ıan adem´ as un s´ımbolo para representar el cero con la apariencia de un ojo semiabierto o una concha marina vista de perfil. 3 La palabra cifra tambi´en se utiliza frecuentemente como sin´onimo de n´ umero o cantidad, pero aqu´ı s´olo la usamos para referirnos a un s´ımbolo b´ asico de un sistema num´erico.

40

´ REPRESENTACIONES POSICIONALES DE NUMEROS

3. Las cifras de las representaciones posicionales no son, como en las representaciones no posicionales, un n´ umero de veces otra de ellas; aqu´ı cada s´ımbolo representa un posible residuo de hacer grupos de O elementos. 4. Existe un s´ımbolo, el 0, para expresar una posici´on vac´ıa4 . El cero 0 de nuestra representaci´on decimal, como expresi´on de ausencia de alg´ un grupo, 5 no 876 d.C. fue inventado por los hind´ ues alrededor del a˜ El n´ umero cero en la actualidad representa mucho m´as que una posici´on vac´ıa y forma parte fundamental del sistema de los n´ umeros reales6 . Ejemplos

1. Sistema decimal ( Base 10 ) El sistema que usamos en la actualidad y que usa la mayor´ıa de los pa´ıses del mundo, es el sistema decimal7; parece natural pensar que esto sea consecuencia de que el ser humano tiene diez dedos en las manos. Los s´ımbolos b´asicos de este sistema los llamamos d´ıgitos, palabra que significa dedo, en lat´ın. Este sistema desarrollado por los hind´ ues, inclu´ıa un s´ımbolo para el cero. Los a´rabes lo dieron a conocer en Europa. Por eso, tambi´en se conoce como indoar´abigo, aunque el aspecto de sus s´ımbolos ha ido cambiando con el tiempo, como se observa en la figura 1. 4

La palabra cero proviene de la voz a´rabe ziffero, que significa lugar vac´ıo, fue inventado por los hind´ ues, pero en Am´erica los mayas utilizaron un concepto de cero, mucho antes de ser usado en Europa para expresar la ausencia de unidades o d´ıas en su sistema vigesimal. 5 Se dice que el s´ımbolo (0) es hind´ u, aunque algunos afirmaban que tuvo su origen en la griega letra “omicron”que es la inicial de la palabra “ouden”o vac´ıo. Actualmente se sabe que aunque el s´ımbolo para representar una posici´ on vac´ıa, en algunas versiones en la tabla de Ptolomeo se parece a un “omicron”, los s´ımbolos iniciales para el cero en las fracciones sexadecimales griegas eran formas redondas decoradas, adem´as, cuando se consolid´ o el sistema decimal en el Imperio Bizantino durante el siglo XV, se parti´ o de un sistema alfab´etico para numerales, quitando las u ´ltimas 18 letras y a˜ nadiendo un s´ımbolo para el cero que ten´ıa diferentes formas y que parec´ıa una h invertida o un punto. 6 SPIVAK, M., C´ alculo infinitesimal, Editorial Revert´e, Bogot´a,1974, p. 5. 7 Los egipcios fueron los primeros en desarrollar un sistema de numeraci´ on decimal.

41

´ ´ ACTIVIDADES MATEMATICAS PARA EL DESARROLLO DE PROCESOS LOGICOS

2. Sistema binario8 (Base 2) Este sistema est´a basado solamente en dos s´ımbolos: 0 y 1. Es el lenguaje ideal para describir situaciones donde se presentan u ´nicamente dos opciones: blanco y negro, verdadero y falso, o para entes que s´olo distingan entre s´ı y no; como las computadoras9 : ellas s´olo reconocen estos dos s´ımbolos; cuando pasa corriente por un circuito electr´onico, la m´aquina lo entiende como 1, si no pasa lo entiende como 0. Con estos 2 s´ımbolos basta para hacer entender a una m´aquina todo lo que ella necesita conocer para manejar el tr´ansito de una ciudad, llevar una nave espacial a Marte, calcular el estado del clima, ganarle al campe´on del mundo jugando ajedrez, etc. Para escribir un n´ umero en base 2, debemos hacer grupos de 2 elementos, y si sobra alg´ un elemento esa ser´a la cifra de las unidades, luego debemos formar grupos de dos con los grupos resultantes del paso anterior, apuntar lo que sobra y as´ı sucesivamente hasta que sea imposible formar nuevos grupos de 2. Los siguientes son los diez primeros n´ umeros escritos en sistema decimal y en sistema binario. 8

Este sistema fue descubierto por el matem´atico alem´an Wilhelm Leibnitz en el siglo XVII. La sugerencia de utilizar el sistema binario en las computadoras la hizo John Von Neumann en 1945, desde entonces las computadoras operan en sistema binario para las entradas y sistema decimal en la salida. 9

42

´ REPRESENTACIONES POSICIONALES DE NUMEROS

Base 10

Base 2

0

1

1

1

2

10

3

11

4

100

5

101

6

110

7

111

8

1000

9

1001

10

1010

3. Sistema hexadecimal (Base 16) Es un sistema cuyos s´ımbolos b´asicos son: 0, 1, 2, 3, 4, 5, 6, 7, 8, 9, A, B, C, D, E, F donde A, B, C, D, E, F representan, respectivamente, los n´ umeros 10, 11, 12, 13, 14 y 15 del sistema decimal. El sistema hexadecimal es muy usado en la programaci´on de computadoras, sobre todo en la descripci´on de localizaciones de memoria en los discos duros y flexibles. Los ordenadores o computadoras modernos son m´aquinas de c´alculo r´apido; la parte de la m´aquina que sirve para la ejecuci´on directa de operaciones se llama unidad aritm´etica. La parte que regula el trabajo de toda la m´aquina se llama dispositivo de control y la parte que almacena los resultados se llama dispositivo de memoria. La memoria es un almac´en de n´ umeros y signos convencionales que se registran el´ectrica o magn´eticamente en cinta o en disco. El car´acter grabado puede ser le´ıdo o borrado, grab´andose otro en su lugar. Los dispositivos de memoria constan de miles de celdas y cada celda, de varias decenas de elementos magn´eticos; 43

´ ´ ACTIVIDADES MATEMATICAS PARA EL DESARROLLO DE PROCESOS LOGICOS

para registrar los n´ umeros por medio del sistema de base dos, cada elemento imantado expresa el 1 y los no imantados, el 0. En computadoras la unidad de informaci´on b´asica, un 0 o un 1, se llama bit, 8 bits forman un Byte y cada car´ acter (cualquier s´ımbolo que una m´aquina de procesamiento de datos puede leer, almacenar e imprimir ) se puede representar por un Byte; es decir, un n´ umero binario de 8 cifras, lo que significa que podemos especificar 256 posibles caracteres b´asicos ( ¿Por qu´e ? ). Los 256 caracteres se pueden colocar en un cuadrado de 16 de lado; es decir, que cualquiera de ellos se puede especificar con n´ umeros hexadecimales de s´olo dos cifras. La primera cifra hexadecimal corresponde a las primeras cuatro cifras binarias de la izquierda y la segunda a las otras cuatro. En la tabla 2 se muestran algunos de ellos. Car´ acter

Configuraci´ on bits

C´ odigo hexadecimal

A

1010 0001

A1

B

1010 0010

A2

C

1010 0011

A3

D

1010 0100

A4

E

1010 0101

A5

F

1010 0110

A6

G

1010 0111

A7

H

1010 1000

A8

I

1010 1001

A9

J

1010 1010

AA

K

1010 1011

AB

L

1010 1100

AC

4. Otros sistemas (Bases 1, 3, 4, 5, etc.) Utilizando el mismo principio de construcci´on pueden hacerse sistemas de numeraci´on en cualquier base; algunas de ellas son poco usadas, otras son usadas por algunas culturas; por ejemplo, en la lengua makua del norte de Mozambique, las palabras thanu (cinco) y miloko (diez) constituyen la base del sistema de numeraci´on. As´ı, seis se dice thanu na moza (cinco m´as uno) y siete, thanu na 44

´ REPRESENTACIONES POSICIONALES DE NUMEROS

pili (cinco m´as dos). Veinte se dice miloko mili (diez veces dos) y treinta, miloko miraru (diez veces tres) En el dialecto balante de Guinea-Bissau se mezclan las bases cinco y veinte, mientras que la lengua bet´e de Cˆote d’Ivoire emplea tres bases: cinco, diez y veinte, lo que da para decir cincuenta y seis, por ejemplo, golosso-ya-kogbo-gbeplo; es decir, “veinte veces dos m´as diez (y) cinco (y) uno ”. En la tabla 3, aparecen los quince primeros n´ umeros escritos en distintas bases; observe la secuencia de formaci´ on. Para indicar la base en que un n´ umero est´a expresado se acostumbra escribirla en la parte inferior derecha de ´el, as´ı 654 8 . Si la base est´a establecida desde el principio no se escribe y si no se hace ninguna referencia a ella se supone que es base 10. N´ umero de la Base 16

10

9

8

7

6

5

4

3

2

1

1

1

1

1

1

1

1

1

1

2

2

2

2

2

2

2

2

2

10

3

3

3

3

3

3

3

3

10

11

4

4

4

4

4

4

4

10

11

100

5

5

5

5

5

5

10

11

12

101

6

6

6

6

6

10

11

12

20

110

7

7

7

7

10

11

12

13

21

111

8

8

8

10

11

12

13

20

22

1000

9

9

10

11

12

13

14

21

100

1001

A

10

11

12

13

14

20

22

101

1010

B

11

12

13

14

15

21

23

102

1011

C

12

13

14

15

20

22

30

110

1100

D

13

14

15

16

21

23

31

111

1101

E

14

15

16

20

22

24

32

112

1110

F

15

16

17

21

23

30

33

120

1111

45

´ ´ ACTIVIDADES MATEMATICAS PARA EL DESARROLLO DE PROCESOS LOGICOS

5. Sistema unario Un sistema posicional con un solo n´ umero, digamos 1, es llamado unario. En ´el cualquier n´ umero es denotado por una cadena de unos y es equivalente al sistema no posicional que construimos en el primer cap´ıtulo pero con un solo s´ımbolo.

2.2.

Operaciones

Centraremos ahora nuestra discusi´on en construir procedimientos, que como ya dijimos tambi´en llamamos algoritmos, para efectuar las operaciones fundamentales en diferentes bases. De nuevo los m´etodos no son u ´nicos, son propuestas hechas y discutidas por los alumnos en el sal´on de clase, que por supuesto pueden modificarse, que es lo deseable.

2.2.1.

Adici´ on

Continuemos en la base O = IIIII, que construimos al principio de este cap´ıtulo. Si queremos adicionar10 dos cantidades, por ejemplo 3423 y 413 debemos inicialmente adicionar las cifras de la derecha (que llamaremos de las unidades) en ambos n´ umeros para saber si logramos formar grupos de I con O elementos; en este caso 3 + 3 = IIIIII = (IIIII) I = O I, en nuestra nueva notaci´on se escribe 11, lo que significa que tenemos un residuo de 1 que retenemos para el resultado final y hemos formado 1 grupo de O elementos, que se tendr´a en cuenta para la adici´on de la siguiente cifra. Este grupo ser´a adicionado a los de las cifras que siguen a la izquierda de las unidades llamadas quinquenas, en este caso 2+1= O O O y le agregamos O del grupo que se form´o en el proceso anterior, as´ı obtenemos O O O O en nuestra notaci´on 4. 10

Usaremos los signos habituales + y - para denotar la adici´ on y la sustracci´ on respectivamente, ´estos fueron empleados por Widman, en un libro de aritm´etica publicado en Alemania en 1489. Widman los utiliz´ o para indicar un exceso o un d´eficit, como “m´as ”o como “menos ”, pero pronto empezaron a usarse como signos de las operaciones de adici´ on y sustracci´ on.

46

´ REPRESENTACIONES POSICIONALES DE NUMEROS

Continuamos adicionando las cifras siguientes de la izquierda de las quinquenas que llamaremos biquinquenas, 4 + 4 = OII OII OIIOII OIIOII OIIOII = (OIIOII OIIOII OII)OII OIIOII = OIIIOII OIIOII lo que escrito en nuestra notaci´on es 13 biquinquenas llevamos 1 y dejamos el 3 para el resultado final. umero Por u ´ltimo adicionamos la cantidad de OIII que tenemos en cada n´ 3 + 0 = OIII OIII OIII y le agregamos OIII que corresponde al grupo que se form´o en el paso anterior obtenemos OIII OIII OIII OIII , que equivale a 4 triquinquenas. Por lo tanto: 3423 + 413 = 4341 Este proceso es el conocido desde la escuela para adicionar n´ umeros naturales, s´olo que all´ı se utilizaban grupos de 10 elementos llamados decenas y grupos de 10 decenas llamados centenas, etc. Al parecer no hemos avanzado mucho, hemos llegado a los mismos resultados que lleg´o la humanidad hace cientos de a˜ nos, s´olo que ahora entendemos el por qu´e y debido a esto podemos generalizar este m´etodo a cualquier base, sabiendo que a pesar de que la notaci´on es diferente, las ideas de fondo y los resultados son los mismos. ❆ ❆ ❆ ❆ ❆ ❆ ❆ ❆ ❆ ❆ ❆ ❆ ❆ ❆ ❆ ❆ ❆ ❆ Muchas veces ser´a necesario tomar l´apiz y papel para seguir una lectura en matem´aticas. ❆ ❆ ❆ ❆ ❆ ❆ ❆ ❆ ❆ ❆ ❆ ❆ ❆ ❆ ❆ ❆ ❆ ❆ Ejemplos

1. En Base 2 el lector puede verificar que: 1111001 + 1011 = 10000100 47

´ ´ ACTIVIDADES MATEMATICAS PARA EL DESARROLLO DE PROCESOS LOGICOS

❆ ❆ ❆ ❆ ❆ ❆ ❆ ❆ ❆ ❆ ❆ ❆ ❆ ❆ ❆ ❆ ❆ Tambi´en es costumbre en matem´aticas, que el lector verifique las afirmaciones que se hacen en un texto y que muchas veces no se explicitan. ❆ ❆ ❆ ❆ ❆ ❆ ❆ ❆ ❆ ❆ ❆ ❆ ❆ ❆ ❆ ❆ ❆ 2. En Base 5 la tabla de la adici´on es: +

0

1

2

3

4

0

0

1

2

3

4

1

1

2

3

4

10

2

2

3

4

10

11

3

3

4

10

11

12

4

4

10

11

12

13

3. En base 16 se tiene que 9AE + 743 = 10F1. 2.2.1.1.

Propiedades de la adici´ on

Como en el cap´ıtulo anterior, luego de que se han hecho muchas adiciones, es posible observar comportamientos regulares, propiedades que le son comunes a todas ellas sin importar la base en que sean realizadas. Debido a que algunas de ellas aparecen con frecuencia, reciben nombres como: 2.2.1.1.1.

Propiedad asociativa

Si en una adici´on hay tres o m´as sumandos11 , hay varias maneras de efectuar la adici´on. En el caso de tres, podemos adicionar los dos primeros y al resultado le adicionamos el tercero o, primero adicionar los dos u´ltimos y adicionar este resultado al primero; la propiedad asociativa de la adici´on asegura que el resultado de la adici´on ser´a el mismo sin importar cu´al de las agrupaciones sea elegida para efectuar la operaci´on. 11

Los tres sumandos no tienen que ser diferentes.

48

´ REPRESENTACIONES POSICIONALES DE NUMEROS

Esto parece tan natural que casi podr´ıamos considerar innecesario mencionarlo, sin embargo, hay situaciones que pueden llevar a confusi´on cuando se mezclan operaciones que no tengan esta propiedad con otras que s´ı. Por ejemplo, el caso de tres comensales que van a un restaurante y piden tres platos cuyo valor unitario es $10 000; en el momento del pago el due˜ no del restaurante otorga un descuento de $5 000 a la cuenta total, el mesero decide devolver $1 000 a cada comensal y quedarse con $2 000 a manera de propina. En este punto empiezan los problemas, pues cada comensal pag´o en total $9 000, lo que nos da una suma de $27 000 que sumamos a los $2 000 que tom´o el mesero, dan $29 000 y han desaparecido $1.000 ¿D´onde est´a el error? 2.2.1.1.2.

Propiedad conmutativa

Cuando contamos, es intuitivamente evidente que el resultado no cambia si lo hacemos en un orden o en el orden contrario (contar al rev´es), las adiciones son cuentas simplificadas, por lo tanto tampoco deber´ıan variar, como en efecto sucede, por ejemplo: 2+3 =3+2 este comportamiento es el mismo en todos los sistemas de representaci´on de n´ umeros. Ejercicios

1. De seguro usted ya not´o otras regularidades en las adiciones, haga una lista de ellas. 2. Enuncie un argumento que explique la siguiente adici´ on. 123456789 123456789 987654321 987654321 2 2222222222

49

´ ´ ACTIVIDADES MATEMATICAS PARA EL DESARROLLO DE PROCESOS LOGICOS

Enuncie un resultado general para cualquier base. Construya adiciones similares cuyo resultados sean n´ umeros cuyas cifras sean s´olo unos, s´olo treses, etc. Repita el ejercicio en base 7; enuncie un resultado general para cualquier base. 3. En la adici´on12 7 + 77 + 777 + . . . + 777777 . . . encuentre el d´ıgito que ocupa el lugar 100 de la suma. Resuelva el mismo problema en base 14. 4. ¿Cu´antas veces aparece cada d´ıgito del 0 al 9 en la lista de los n´ umeros del 1 al 100? Resuelva el mismo problema en base 13. ❆ ❆ ❆ ❆ ❆ ❆ ❆ ❆ ❆ ❆ ❆ ❆ ❆ ❆ ❆ ❆ ❆ ❆ Habr´a notado la insistencia que hemos hecho en traducir cada ejercicio a cualquier base; en realidad, lo que importa es notar que hay ciertas relaciones entre los n´ umeros que son independientes del sistema que se utilice para representarlos. As´ı, un maya y un ruso encontrar´an las mismas relaciones entre los n´ umeros sin importar la simbolog´ıa utilizada. Justamente all´ı se encuentra la transici´on entre la aritm´etica y el ´algebra, en donde se estudian las leyes generales que relacionan los n´ umeros, independientes del lenguaje utilizado. ❆ ❆ ❆ ❆ ❆ ❆ ❆ ❆ ❆ ❆ ❆ ❆ ❆ ❆ ❆ ❆ ❆ ❆

2.2.2.

Sustracci´ on

13

En esta secci´on no queremos explicar una manera de sustraer, que tambi´en llamamos restar14 ; esperamos que usted invente sus propios procedimientos; le 12

Mathematics Teacher, Vol. 85, # 9, Diciembre 1992. Debemos insistir en que, en el sentido moderno del t´ermino operaci´ on, la sustracci´ on no es una operaci´ on entre n´ umeros naturales, pues para ello se requiere que todas las sustracciones posibles de n´ umeros naturales tengan un resultado en los n´ umeros naturales, aqu´ı s´olo es posible cuando el minuendo es mayor o igual que el sustraendo. Sin embargo, para nuestros prop´ ositos actuales este no es un obst´aculo insoslayable, por lo que lo pasaremos por alto, de momento. 14 El signo m´ as antiguo para indicar la sustracci´ on es (∧), que utilizaban los antiguos egipcios, se encuentra en el papiro de Rhind. 13

50

´ REPRESENTACIONES POSICIONALES DE NUMEROS

mostraremos enseguida un ejemplo de un algoritmo para restar en base 6 y le sugerimos que lo explique. 5435013 − 53435 5341134

Ejercicio Estudie las siguientes sustracciones y exprese en que base est´an hechas, ¿es posible que una misma sustracci´on sea correcta en bases distintas?, ¿en qu´e casos? 345323 − 54201 261122

753403 − 54201 688202

545023 − 34001 511022 ❆ ❆ ❆ ❆ ❆ ❆ ❆ ❆ ❆ ❆ ❆ ❆ ❆ ❆ ❆ ❆ ❆ ❆ Suponemos que debe ir soltando la mano para que se acostumbre a hacer operaciones en otras bases; el n´ umero necesario de ejercicios depende de su voluntad y comprensi´on. ❆ ❆ ❆ ❆ ❆ ❆ ❆ ❆ ❆ ❆ ❆ ❆ ❆ ❆ ❆ ❆ ❆ ❆ 51

´ ´ ACTIVIDADES MATEMATICAS PARA EL DESARROLLO DE PROCESOS LOGICOS

2.2.2.1.

Propiedades de la Sustracci´ on

Contrario a lo que sucede con la adici´on, la sustracci´on no tiene tantas regularidades, por ejemplo: 2.2.2.1.1.

La Sustracci´ on no es asociativa

Por c´alculo directo se ve que: (12 − 4) − 3 = 12 − (4 − 3) y si hacemos sustracciones y adiciones, esta combinaci´on de operaciones tampoco puede asegurarse que con cualquier agrupaci´on se obtenga el mismo resultado, por ejemplo: 30000 − (3000 + 2000) = (30000 − 3000) + 2000 ¿Le recuerda esta desigualdad alg´ un problema de comensales? 2.2.2.1.2.

La sustracci´ on no es conmutativa

Para que una sustracci´on pueda hacerse es necesario que el minuendo sea m´as grande15 que el sustraendo, de modo que si invertimos la operaci´on y tomamos el minuendo como sustraendo y el sustraendo como minuendo, la sustracci´on ya no se puede efectuar, por lo tanto la sustracci´on no es conmutativa. Ejercicio Escriba un algoritmo para la sustracci´ on en cualquier base, ¿encuentra algunas propiedades para la sustracci´ on? 2.2.2.2.

Relaciones entre la adici´ on y la sustracci´ on: Ecuaciones de la forma x + a = b

Las operaciones de adici´on y sustracci´on no son independientes, hay relaciones entre las dos que generalmente se utilizan como mecanismo de prueba para verificar si efectuamos alguna de ellas de manera correcta. 15

Por el momento usamos la idea de orden que tienen los estudiantes. En la siguiente secci´ on precisaremos su significado en el contexto de los n´ umeros naturales.

52

´ REPRESENTACIONES POSICIONALES DE NUMEROS

❆ ❆ ❆ ❆ ❆ ❆ ❆ ❆ ❆ ❆ ❆ ❆ ❆ ❆ ❆ ❆ ❆ ❆ En matem´aticas, por fortuna, existen mecanismos para determinar si lo que estamos haciendo es correcto o no; procedimientos de verificaci´on que nos permiten decidir, por nosotros mismos, si nuestras afirmaciones son v´alidas; ¡no debemos confirmarlo con alguna persona! ❆ ❆ ❆ ❆ ❆ ❆ ❆ ❆ ❆ ❆ ❆ ❆ ❆ ❆ ❆ ❆ ❆ ❆ Por el significado que le dimos a la adici´on y a la sustracci´on, si agregamos a una cantidad que representamos con la letra a otra cantidad que representamos con la letra b y obtenemos un resultado que representamos con la letra c, podemos expresar esta misma informaci´on de otras dos maneras: si a c le sustraemos a obtenemos b y si a c le sustraemos b obtenemos a. Simb´olicamente diremos que: a + b = c es lo mismo que c − a = b o tambi´en c − b = a as´ı, para probar si la sustracci´on c − a = b est´a bien efectuada, basta adicionar a a b para obtener como resultado c. Por ejemplo, la sustracci´on 8−3= 5 es correcta en cualquier base mayor que 9, puesto que en ellas se tiene que 5+3=8 Las anteriores relaciones se utilizan tambi´en para encontrar alguno de los n´ umeros que no est´an escritos16 en una adici´on o en una sustracci´on; por ejemplo, si queremos averiguar cu´al es el n´ umero que adicionado con 345 6 nos da como resultado 4325 6 , debemos escribir esta suma en otra de sus formas equivalentes y obtener el n´ umero buscado como el resultado de otra operaci´on. En nuestro ejemplo representemos con la letra x el n´ umero buscado; la suma es: 345 6 + x = 4325 6 , que puede escribirse, 4325 6 − 345 6 = x 16

Si en una igualdad donde participan operaciones entre n´ umeros ignoramos alguno (o varios) de ellos, la llamaremos una ecuaci´ on. Encontrar el (o los) n´ umero(s) que ignoramos es resolver la ecuaci´ on.

53

´ ´ ACTIVIDADES MATEMATICAS PARA EL DESARROLLO DE PROCESOS LOGICOS

de donde vemos que para hallar x debemos efectuar la sustracci´on y este proceso ya lo sabemos hacer; el resultado es 3540 6 . Para probar que el resultado es correcto adicionamos 3540 6 con 345 6 y debemos obtener 4325 6 . ❆ ❆ ❆ ❆ ❆ ❆ ❆ ❆ ❆ ❆ ❆ ❆ ❆ ❆ ❆ ❆ ❆ ❆ Toda esta discusi´on parece muy simple, pero aseg´ urese que lo entiende bien haciendo algunos ejercicios inventados por usted. ❆ ❆ ❆ ❆ ❆ ❆ ❆ ❆ ❆ ❆ ❆ ❆ ❆ ❆ ❆ ❆ ❆ ❆ Por supuesto, podemos partir de la segunda o tercera formas de la igualdad; es decir, podemos averiguar cu´al es el n´ umero que sustra´ıdo de 7652 9 nos da umero buscado c´omo resultado 231 9 ; si de nuevo representamos17 con x el n´ planteamos: 7652 9 − x = 231 9 que puede escribirse como 7652 9 − 231 9 = x resultado que puede obtener el lector sin mayor dificultad. Ejercicios 1. Coloque en cada casilla marcada por el s´ımbolo  un n´ umero, para que los resultados coincidan con los n´ umeros dados, trabajando en base 7:  5  6 + 4 6 3  1 2  3  ¿Cu´antas soluciones diferentes hay ? En esta clase de ejercicios, es m´ as importante el proceso de soluci´on que la soluci´on misma; los procedimientos de soluci´on, son muy u ´tiles para 17

Es costumbre llamar inc´ ognita al n´ umero buscado y notarlo x, aunque naturalmente podemos representarla con cualquier otro s´ımbolo.

54

´ REPRESENTACIONES POSICIONALES DE NUMEROS

generalizar problemas y se usan frecuentemente en la programaci´on de computadoras cuando los problemas permiten una sistematizaci´on tal. Escriba un algoritmo que permita resolver acertijos de este tipo, empezando con una cifra; por ejemplo, resolver problemas de la forma:  + 3 5 en base 6, luego en base 8, etc. Luego ponga dos veces el s´ımbolo ; por ejemplo:  + 3  ¿En qu´e condiciones existe la soluci´on?, ¿cu´ antas soluciones existen?, ¿hay una manera general de resolver el problema en cualquier base? Intente resolver el mismo problema pero ahora con dos cifras y una sola inc´ognita, luego con dos, etc. Si el empe˜ no se lo permite, formule y estudie un problema m´ as general. 2. Por cada problema bien resuelto en clase de matem´aticas, recibe 8 puntos como nota; pero en cambio por cada problema mal resuelto o no resuelto le descuentan 5. Si le presentan 26 problemas y usted tiene como nota 0. ¿Cu´antos problemas trabaj´o, y de ´estos, cu´antos resolvi´o bien? 3. (Tomado de Caro18 ) Dos muchachos atados a los extremos de una cuerda suben una escalera de 100 pelda˜ nos, y lo hacen altern´andose, es decir, que mientras uno sube el otro descansa. La longitud libre de la cuerda que los ata es igual a la separaci´on de 10 pelda˜ nos; se trata de saber cu´al llegar´a primero arriba. El que inicia el juego puede subir hasta diez escalones, pero se para, por ejemplo en el 4o . El otro, s´olo puede subir hasta el pelda˜ no 14, porque la 18

CARO, V., Los n´ umeros: su historia, sus propiedades, sus mentiras y verdades, Minerva, Bogot´a. 1936, p. 253.

55

´ ´ ACTIVIDADES MATEMATICAS PARA EL DESARROLLO DE PROCESOS LOGICOS

cuerda no da m´as; pero puede detenerse antes, por ejemplo en el 11. El primer jugador puede ahora llegar hasta el 21. Y as´ı contin´ uan hasta el fin. ¿En qu´e pelda˜ nos debe detenerse un jugador para llegar primero?19. 2.2.2.3.

Orden en los n´ umeros naturales

Una aplicaci´on de la adici´on, que es muy importante en matem´aticas, es su uso para definir un orden entre los n´ umeros, esto es una manera de saber cu´ando un n´ umero es m´as grande o m´as peque˜ no que otro. En el cap´ıtulo 1 dijimos que para distinguir si un n´ umero es menor que otro (en una representaci´on no posicional como la desarrollada all´ı) si su representaci´on estaba de forma expl´ıcita o impl´ıcita en los s´ımbolos que forman al otro. Esto significa que el n´ umero m´as peque˜ no se puede completar para conseguir el mayor, o dicho de otra forma un n´ umero natural n es mayor que otro n´ umero m si existe un tercer n´ umero natural p diferente de cero, que cumpla: m+p=n cuando esto suceda escribimos n > m. Ejercicio 1. Para comparar n´ umeros relativamente grandes, el procedimiento que utilizamos no es el descrito. ¿C´ omo saber cu´ al de los siguientes dos n´ umeros es el mayor, si est´ an escritos en base 12? 6532098672245677

y 6532098762245677

¿Importa la base? ¿Y si comparamos dos n´ umeros que no tengan el mismo n´ umero de cifras? Justifique las afirmaciones que haga. 2. Haciendo las cuentas en base 7, ¿para cu´ ales valores de x se cumple que: a) x > 3? b)

x + 12 > 35?

19

Obviamente, el juego puede hacerse sin escalera y sin cuerda, entre dos personas que apuestan a ver qui´en llega primero a 100, diciendo cada uno un n´ umero mediante la condici´on de que el n´ umero que dice el uno no puede exceder en m´ as de 10 al que dice el otro.

56

´ REPRESENTACIONES POSICIONALES DE NUMEROS

c) x − 42 > 63? d ) 54 − x > 16? Justifique sus procedimientos. 3. Con las 5 cifras del n´ umero 12345 se pueden formar 120 n´ umeros, si los n´ umeros se escriben en orden ascendente, desde 12345

12354

1243512453

hasta

54321

¿Qu´e n´ umero es el que ocupa el lugar 75?

2.2.3.

Multiplicaci´ on

20

Ejercicios

1. Proponga un procedimiento para multiplicar en base 7. Puede ser u ´til una comparaci´on con lo que conocemos para base 10. 2. Multiplicaci´ on por galeras: La Figura 2 hace parte de una p´agina de Aritm`etica, impresa en Treviso (Italia) en 1478. Descifre lo que en ella se encuentra escrito. 2.2.3.1.

Propiedades de la multiplicaci´ on

La multiplicaci´on21 es una operaci´on que se construye a partir de la adici´on, ¿ser´ a razonable esperar que algunas de las propiedades de la adici´ on, sean 20

Inicialmente usaremos el signo x para indicar una multiplicaci´ on, el mismo que fue empleado por William Oughtred en Inglaterra, en 1631. 21 La multiplicaci´ on es una operaci´ on elaborada, los griegos usaban la tabla pitag´ orica que conoc´ıan antes de Pit´ agoras. Los babilonios empleaban tablas de cuadrados, y para los romanos la operaci´ on era lenta y trabajosa.

57

´ ´ ACTIVIDADES MATEMATICAS PARA EL DESARROLLO DE PROCESOS LOGICOS

heredadas por la multiplicaci´on? Si observamos muchas multiplicaciones notamos que se tienen las propiedades:

58

´ REPRESENTACIONES POSICIONALES DE NUMEROS

2.2.3.1.1.

Propiedad asociativa

Verifique que la multiplicaci´ on tiene la propiedad asociativa. 2.2.3.1.2.

Propiedad conmutativa

¿No le parece extra˜ no que la multiplicaci´on sea conmutativa? C´omo es posible que sea lo mismo colocar 17 veces el 35 y adicionar, que colocar 35 veces el 17 y adicionar? Pareciera que las dos adiciones nada tuvieran en com´ un. Sin embargo verifique que la multiplicaci´ on es conmutativa sin importar la base en que sea efectuada. Proponga una explicaci´on para esta propiedad. 2.2.3.1.3.

Existe un elemento id´ entico para la multiplicaci´ on

El n´ umero 1 tiene un comportamiento particular en los n´ umeros naturales cuando se multiplica con los dem´as, si aplicamos la definici´on de multiplicaci´on22 obtenemos de manera natural que n×1=n Como en ocasiones anteriores, si queremos preservar la propiedad conmutativa para la multiplicaci´on, debemos convenir que 1 multiplicado por cualquier n´ umero tambi´en da como resultado el mismo n´ umero; simb´olicamente lo escribimos 1×n = n en donde la letra n puede ser reemplazada por un n´ umero cualquiera, aunque, como ya hemos dicho, no se puede efectuar una adici´on con un solo n´ umero. Tambi´en convenimos23 que 0×n=0 si queremos que la multiplicaci´on se mantenga conmutativa en todos los casos, puesto que n veces adicionado 0, s´ı es 0. 22

Algunas veces se usa el s´ımbolo x (no cursiva) para denotar multiplicaci´ on; en otras ocasiones el punto, y cuando no hay lugar a confusi´ on no se escribe signo alguno. 23 Este resultado se puede obtener de otras formas cuando hayamos estudiado un poco m´ as a fondo las propiedades de las operaciones adici´ on y multiplicaci´ on.

59

´ ´ ACTIVIDADES MATEMATICAS PARA EL DESARROLLO DE PROCESOS LOGICOS

2.2.3.1.4. adici´ on

Propiedad distributiva de la multiplicaci´ on respecto a la

Otra propiedad que resulta muy u ´til para simplificar los algoritmos de multiplicaci´on es que si multiplicamos un n´ umero por la suma de otros dos, el resultado es el mismo que adicionar los resultados de los productos del primero con cada uno de los sumandos; en s´ımbolos podemos expresar esto escribiendo: Si a, b, c representan n´ umeros naturales cualesquiera, entonces a × (b + c) = a × b + a × c Podr´ıamos visualizar un poco esta propiedad, pensando en la siguiente adici´on: 4 × (3 + 5) = (3 + 5) + (3 + 5) + (3 + 5) + (3 + 5) = (3 + 3 + 3 + 3) + (5 + 5 + 5 + 5) =4×3+4×5 o justificarla geom´etricamente con: 3

5

3 =

4

5 +

4

4

Figura 3 O con una distribuci´on espacial de puntos: 4

           

+

3

        

3 × (4 + 3)

4

=

           

=

(3 × 4)

3

+

        

(3 × 3)

El procedimiento m´as conocido para la multiplicaci´on en base 10 se deriva de la propiedad distributiva de la multiplicaci´on con respecto a la adici´on (explique) y es adaptable para multiplicar en cualquier base. La tabla de multiplicar en base 5 es: 60

´ REPRESENTACIONES POSICIONALES DE NUMEROS

BASE 5 ×

0

1

2

3

4

0

0

0

0

0

0

1

0

1

2

3

4

2

0

2

4

11

13

3

0

3

11

14

22

4

0

4

13

22

31

La tabla de multiplicar en base 12 tiene algunas curiosidades: BASE 12 ×

0

1

2

3

4

5

6

7

8

9

A

B

0

0

0

0

0

0

0

0

0

0

0

0

0

1

0

1

2

3

4

5

6

7

8

9

A

B

2

0

2

4

6

8

A

10

12

14

16

18

1A

3

0

3

6

9

10

13

16

19

20

23

26

29

4

0

4

8

10

12

18

20

24

28

30

34

38

5

0

5

A

13

18

21

26

2B

34

39

42

47

6

0

6

10

16

20

26

30

36

40

46

50

56

7

0

7

12

19

24

2B

36

41

48

53

5A

65

8

0

8

14

20

28

34

40

48

54

60

68

74

9

0

9

16

23

30

39

46

53

60

69

76

83

A

0

A

18

26

34

42

50

5A

68

76

84

92

B

0

B

1A

29

38

47

56

65

74

83

92

A1

Ejercicios 1. Verifique la propiedad distributiva de la multiplicaci´on respecto a la sustracci´ on. 2. Observe y describa algunas regularidades de la tabla 6. 61

´ ´ ACTIVIDADES MATEMATICAS PARA EL DESARROLLO DE PROCESOS LOGICOS

3. Compare esta tabla con la de la base 10. Obs´ervelas y encuentre regularidades entre los n´ umeros escritos en base 12 que sean m´ ultiplos de 2, con el fin de determinar algunos criterios de divisibilidad por 2. De manera similar, establezca criterios de divisibilidad por 3, 4, 6, 12, en base 12. 4. Coloque par´entesis para que obtenga una igualdad 5,9,5 + 2 − 8,3 + 1 = 22 5. Resuelva el siguiente acertijo en base 7.   × 6 6            4 5   

 6 

4

6. Invente un algoritmo para resolver este problema con factores de una sola cifra. 7. ¿En qu´e base est´a 31 = 2 × 17? 8. ¿Intuye alguna relaci´ on entre las propiedades de la adici´ on y las de la multiplicaci´ on? Elabore una argumentaci´ on que explique ´estas en t´erminos de aquellas. 9. En el papiro de Rhind24, un manual de antiguas matem´aticas egipcias, escrito alrededor de 1700 A.C., se describe una forma sumativa de multiplicar. 24

El m´ as antiguo de los papiros egipcios que data aproximadamente de la dinast´ıa doce (2000-1788 a. C.), el papiro de Mosc´ u, se halla en el British Museum de Mosc´ u. El papiro de Rhind, que se conoce tambi´en como papiro de Ahmes, por el nombre de su autor, tiene 30 cm de alto y 6 cm de largo, fue comprado en 1858 en una ciudad del Nilo por el escoc´es Henry Rhind, del cual se deriva su nombre. En ellos aparecen los conocimientos que sobre aritm´etica pose´ıan los egipcios, el sistema de numeraci´ on y algunas operaciones. Los papiros contienen problemas y soluciones, 85 en papiro Rhind y 25 en el papiro de Mosc´ u.

62

´ REPRESENTACIONES POSICIONALES DE NUMEROS

Para multiplicar, por ejemplo, 25 × 27 se hace lo siguiente25: •

• •

1 2 4 8 16

vez veces veces veces veces

27 27 27 27 27

es son son son son

27 54 108 216 432

Los n´ umeros marcados en la primera columna suman 25, por lo tanto 25 veces 27 se obtiene de adicionar los n´ umeros correspondientes en la u ´ltima columna. Este procedimiento nos sugiere un m´etodo para multiplicar en base 2: Sabemos que para multiplicar por dos, un n´ umero en base 2, basta agregar un 0; as´ı si queremos multiplicar 1 1 0 1 1 que corresponde a 27 en base 10, por 25 (en base 10), basta copiar: •

• •

1 2 4 8 16

vez veces veces veces veces

27 27 27 27 27

es son son son son

11011 110110 1101100 11011000 110110000

y adicionamos los valores se˜ nalados, obteniendo 1010100011. Quiz´a el m´etodo no sea tan pr´actico o inteligente para un humano de esta ´epoca, pero existen unos “tontos u ´tiles ”que no les preocupa hacer cuentas largas con tal que s´olo sean con 0 y 1. Con base en lo expuesto, escriba un algoritmo para multiplicar n´ umeros en base 2. ¿Puede extenderlo a otras bases? 1. En algunos pueblos de Rusia multiplican utilizando un procedimiento que no requiere el uso de las tablas de multiplicar, este es: 25

NEWMAN, J., Sigma El mundo de las matem´ aticas, vol I, Ed. Grijalbo, Barcelona, 1994, p. 97.

63

´ ´ ACTIVIDADES MATEMATICAS PARA EL DESARROLLO DE PROCESOS LOGICOS

a) Los factores se colocan uno frente al otro. b)

Al factor ubicado en el lado izquierdo se divide entre 2 sucesivamente hasta que el cociente sea 1; los cocientes obtenidos en el transcurso se escriben uno debajo de otro.

c) El factor de la derecha se duplica repetidamente hasta que el n´ umero correspondiente en la izquierda sea 1. d ) Se tachan o se˜ nalan en la columna de la derecha los n´ umeros cuyos correspondientes, en la columna izquierda, sean pares. e) Se adicionan las cantidades no tachadas; ´este es el producto buscado. Por ejemplo, multipliquemos 18 × 7: 18 9 4 2 1 126

7 * 14 28 * 56 * 112

Proponga una explicaci´on de este procedimiento. ¿Es v´ alido en otras bases? 2. Los a´rabes, desde el siglo X, usaban un m´etodo para multiplicar que llamaban el “m´etodo de las casas ”, que se podr´ıa representar de la siguiente manera: Los n´ umeros que vamos a multiplicar se colocan en la parte superior de un cuadril´atero dividido en cuadrados; en cada uno de ellos se escribe el producto de cada una de las cifras de uno de los factores por cada una de las cifras del otro factor y luego se adicionan en diagonales empezando por las unidades de 49. 987 987 81

72

63

< 987 × 9 >

72

64

56

< 987 × 8 >

63

56

49

< 987 × 7 > 64

´ REPRESENTACIONES POSICIONALES DE NUMEROS

Como la adici´on en diagonales algunas veces causaba problemas, autores ´arabes como Ibn al- Ban´a (fallecido en 1321) y Al-Kash´ı (fallecido en 1429) propusieron trazar diagonales en cada cuadrado para que el m´etodo fuera m´as seguro. De aqu´ı surge un m´etodo para multiplicar26 llamado gelosia o graticola, debido a Luca Pacioli, con este m´etodo la anterior multiplicaci´on se efectuar´ıa as´ı: Inicialmente se disponen los n´ umeros alrededor de un cuadril´atero dividido en cuadrados en los que se inscriben los totales (a uno y otro lado de las diagonales, orientadas de acuerdo con la posici´on de los n´ umeros de partida), como en la figura 5: 9 9

8

7

7

4

6

8 7

1

6 2

6 2

3

9

6

8

9

7

5 4

5 3

7

4 6

Figura 5

La adici´on de n´ umeros en diagonal, empezando por el cuadrado inferior derecho, da el producto 974169, que se inscribe en los lados libres de la gelosia27. Explique por qu´e este procedimiento funciona. Haga ejemplos en otras bases. 3. Un procedimiento para multiplicar28, llamado castellucio, tambi´en de Luca Pacioli, que puede considerarse como un antecesor de la forma moderna, 26

GOLDSTEIN, C., Mundo cient´ıfico, vol.15, No. 161, oct. 1995. Esta palabra designaba el entramado de madera o hierro que, como el mucharabiah ´arabe, permite ver sin ser visto desde el interior de una casa y protege el pudor de las mujeres. 28 GOLDSTEIN, C., op cit., p. 47 27

65

´ ´ ACTIVIDADES MATEMATICAS PARA EL DESARROLLO DE PROCESOS LOGICOS

se ilustra en la siguiente figura, multiplicando 9876 por 6789: 9

8

7

6

6 6 1

6

7

8

9

6

1 5

1 4 4

0 3 7 4

1 1 5 0

0 2 2 7

0 0 3 3

0 0 0 4

6

7

0

4

8

1

6

4

Explique por qu´e este procedimiento funciona. Haga ejemplos en otras bases.

2.2.4.

Divisi´ on

29

Si conocemos el resultado de un producto pero ignoramos alguno de los factores, es posible hallarlo mediante una divisi´on 30 ; el factor desconocido lo llamamos cociente 31 , que representamos con la letra c; el factor conocido lo llamamos divisor y lo representamos con la letra d; el resultado del producto lo llamamos dividendo y lo notamos con la letra D. Dicho de otro modo, si D÷d = c es porque c × d = D La divisi´on de D (dividendo) entre d (divisor) con cociente c, se puede indicar de uno de los tres modos siguientes32: 29

Como la sustracci´on, la divisi´ on no es una operaci´ on entre n´ umeros naturales, en el sentido moderno de la palabra, pues no toda divisi´ on entre n´ umeros naturales tiene como resultado un n´ umero natural; pero de nuevo esto no nos causa problemas para nuestros prop´ ositos actuales. 30 La divisi´ on es la m´as compleja de las “operaciones ”de la aritm´etica, por lo que su aparici´ on fue un poco tard´ıa. Babilonios e hind´ ues fueron los primeros en conocerla. Los m´etodos actuales para resolver la divisi´ on se derivan de los hind´ ues, quienes dispon´ıan en una mesa de arena los elementos de la operaci´on; dividendo, divisor, cociente y residuo. Estos conocimientos fueron transmitidos a Europa por los a´rabes; Leonardo de Pisa los expuso en 1202 en Italia. 31 Etimol´ ogicamente la palabra cociente significa, cu´ antas veces. 32 Los ´arabes indicaron la divisi´ on en forma de fracciones; Rahn emple´ o el signo (÷) para la

66

´ REPRESENTACIONES POSICIONALES DE NUMEROS

D

D÷d =c

d c

D = Dd = c d

Por supuesto, tambi´en se tiene que si D ÷ d = c, entonces D ÷ c = d. Esta clase de divisi´on se llama exacta, se dice tambi´en que el dividendo es m´ ultiplo del divisor. Como ya dijimos, las operaciones de multiplicaci´on y divisi´on exacta no son independientes, hay relaciones entre ellas, an´alogas a las que existen entre la adici´on y la sustracci´on, que tambi´en, en este caso, se utilizan como mecanismo de prueba para verificar si efectuamos alguna de ellas de manera correcta. Las tres escrituras siguientes expresan exactamente lo mismo: a×b=c

es lo mismo que

c÷b = a

o tambi´en

c÷a =b

as´ı, para probar si la divisi´on c ÷ a = b est´a bien efectuada, basta multiplicar a × b y obtener como resultado c. 2.2.4.1.

Propiedades de la divisi´ on

Al igual que la sustracci´on, la divisi´on no es conmutativa y no es asociativa, pero s´ı, se cumple parcialmente la propiedad distributiva de la divisi´on respecto a la adici´on si la divisi´on est´a a la derecha de la suma. 2.2.4.1.1.

Propiedad distributiva a derecha de la divisi´ on respecto a la adici´ on

Si a, b, c representan n´ umeros naturales cualesquiera, entonces (a + c) ÷ b = (a ÷ b) + (c ÷ b) divisi´ on en un libro publicado en 1669, indicando que el punto encima de la l´ınea da la posici´ on del dividendo o numerador de una fracci´ on y el punto por debajo de la l´ınea, la del divisor o denominador. En 1202, Leonardo de Pisa (Fibonaci, hijo de Bonaci ) introdujo el uso de la raya horizontal entre los n´ umeros para indicar la divisi´ on (tomado de los textos a´rabes). En 1684, Leibnitz introdujo como signo de divisi´ on los dos puntos (:) usado actualmente en muchos pa´ıses.

67

´ ´ ACTIVIDADES MATEMATICAS PARA EL DESARROLLO DE PROCESOS LOGICOS

en palabras, si tenemos la suma de dos n´ umeros dividida entre un n´ umero, el resultado equivale a adicionar los resultados de las divisiones de cada sumando con el divisor dado; o escrito de una manera m´as usual, as´ı: a+b a b = + c c c Pero debemos tener cuidado, porque si la suma est´a a la derecha de la divisi´on la igualdad ya no se tiene, o sea que no se cumple: b ÷ (a + c) = (b ÷ a) + (b ÷ c) o de otra forma b b b = + a+c a c por ejemplo: 24 ÷ (6 + 2) = (24 ÷ 6) + (24 ÷ 2) 24 ÷ 8 = 4 + 12 3 = 16 2.2.4.2.

Ecuaciones de la forma ax = b

Como en el caso de la adici´on y la sustracci´on, estas relaciones se utilizan tambi´en para encontrar alguno de los n´ umeros que no est´an escritos en una multiplicaci´on o una divisi´on; por ejemplo, si queremos averiguar cu´al es el n´ umero que multiplicado con 35 7 nos da como resultado 453 7 , debemos escribir esta multiplicaci´on en otra de sus formas equivalentes y obtener el n´ umero buscado como el resultado de otra operaci´on. En nuestro ejemplo, representemos con la letra x el n´ umero buscado; la multiplicaci´on33 es: 35 7 • x = 453 7 que puede escribirse, 453 7 ÷ 35 7 = x 33

En este caso, por ejemplo, no es aconsejable usar × para denotar la multiplicaci´ on, por la posible confusi´ on con el n´ umero buscado x.

68

´ REPRESENTACIONES POSICIONALES DE NUMEROS

de donde vemos que para hallar x debemos efectuar la divisi´on y este proceso ya lo sabemos hacer; el resultado es 12 7 . Para probar que el resultado es correcto34 multiplicamos 35 7 • 12 7 y obtenemos 453 7 Ejercicios 1. Construya algunos ejercicios escribiendo productos y divisiones donde uno de los n´ umeros no est´ a escrito de manera expl´ıcita35 , hasta que el proceso sea natural para usted. 2. Si a partir del 37 ( en base 10 ) multiplicamos por m´ ultiplos de 3 obtenemos 37 × 3 =111 37 × 6 =222 37 × 9 =333 37 × 12 =444 37 × 15 =555 37 × 18 =666 37 × 21 =777 ¿Existen otro par de n´ umeros que nos den los mismos resultados? 3. Observe la siguiente secuencia de hechos curiosos e intente encontrar una regularidad: i. En base 3 el n´ umero 2 multiplicado por sus dos primeros m´ ultiplos (exceptuando el cero) da 11 y 22 respectivamente: 2 × 2 = 11 2 × 11 = 22 34

Seguramente ya not´ o que casi todas las ideas (y hasta las palabras) coinciden con la secci´ on sobre relaciones entre la adici´ on y la sustracci´ on, lo que dice que no estamos aprendiendo cosas nuevas; los dos procesos son esencialmente el mismo. 35 Un detalle importante es que el punto de partida es un producto o una divisi´ on donde escondemos un n´ umero con el prop´osito de hallarlo despu´es; no estamos hablando de encontrar un n´ umero que multiplicado por otro nos d´e un resultado establecido de antemano, porque posiblemente dicho n´ umero no exista. Por ejemplo, en base 10, no existe un n´ umero natural que multiplicado por 3 nos d´e como resultado 8. Debemos estar seguros de que la soluci´ on existe; es decir, que es un n´ umero natural, si no fuera as´ı no podr´ıamos aplicarle las reglas que establecemos para ellos.

69

´ ´ ACTIVIDADES MATEMATICAS PARA EL DESARROLLO DE PROCESOS LOGICOS

ii. En base36 4 el n´ umero 13 multiplicado por 3 o por un m´ ultiplo de 3 (exceptuando el 0), da como producto un n´ umero formado por tres cifras iguales al m´ ultiplo de 3 utilizado; as´ı: 13 × 3 = 111 13 × 12 = 222 13 × 21 = 333 iii. . En base 5 el n´ umero 124 multiplicado por 4 o por sus m´ ultiplos (menos el cero), da como producto un n´ umero formado por cuatro cifras iguales al m´ ultiplo de 4 utilizado: 124 × 4 = 1111 124 × 13 = 2222 124 × 22 = 3333 124 × 31 = 4444 iv. En base 6, al multiplicar 1235 por 5 o por sus m´ ultiplos (desde 1 hasta 5), se genera como producto un n´ umero de cinco cifras iguales al m´ ultiplo de 5 utilizado, as´ı: 1235 × 5 = 11111 1235 × 14 = 22222 1235 × 23 = 33333 1235 × 32 = 44444 1235 × 41 = 55555 v. En base 12 si multiplicamos 123456789B por B o por sus m´ ultiplos (m´ ultiplos tomados desde 1 hasta B) encontramos como producto un n´ umero de once cifras iguales al m´ ultiplo de B usado, verif´ıcalo: 123456789B × B = 11111111111 123456789B × 1A = 22222222222 36

Desde ahora se omitir´a el sub´ındice que se˜ nala la base en la cual estamos trabajando, debido a que ´esta se expl´ıcita en el subt´ıtulo.

70

´ REPRESENTACIONES POSICIONALES DE NUMEROS

Son todos los que est´an pero no est´an todos los que son, porque en base 7 podemos obtener la siguiente tabla: 202 × 4 = 1111 202 × 11 = 2222 202 × 15 = 3333 202 × 22 = 4444 202 × 26 = 5555 202 × 33 = 6666 que no es de la misma forma que ven´ıamos observando. ¿En qu´e bases podemos elaborar una tabla con la caracter´ıstica de la anterior?, ¿cu´ ando es posible, de cu´antas maneras diferentes puede hacerse? 4. En una secci´on anterior observamos que el orden de los n´ umeros naturales puede definirse utilizando la adici´on. Con un procedimiento an´ alogo defina un nuevo orden, pero en este caso utilice la multiplicaci´ on. Estudie los conjuntos de n´ umeros que son “m´ as grandes”que un n´ umero dado. ¿Hay elementos comunes en estos conjuntos cuando se escogen como iniciales n´ umeros distintos? 2.2.4.3.

Un procedimiento para la divisi´ on

La idea fundamental, cuando intentamos dividir una cantidad entre otra es, como dijimos en cap´ıtulo 1, repartir la primera cantidad en grupos que tengan la segunda cantidad, y esto no siempre es posible hacerlo de manera exacta, generalmente sobra un residuo; ¿c´omo procedemos para efectuar dicha repartici´on en este caso, si las cantidades est´an escritas en una base cualquiera? Antes de seguir intente encontrar una manera propia. Para construir un algoritmo para efectuar divisiones en una base cualquiera, podemos utilizar dos ideas: la suposici´on de que la base 10 nada tiene de especial con respecto a las otras y el conocimiento que tenemos desde la escuela para dividir en base 10. Si este procedimiento sirve para base 10 debe servir para cualquier base. Podemos describirlo as´ı: 1. Tomamos tantas cifras de la izquierda del dividendo como cifras tenga el divisor. ¿Por qu´e? 71

´ ´ ACTIVIDADES MATEMATICAS PARA EL DESARROLLO DE PROCESOS LOGICOS

2. Si el divisor es menor o igual que el formado con las cifras tomadas del dividendo, entonces: 2.1. Buscamos un n´ umero que multiplicado por el divisor quede lo m´as pr´oximo posible a ´el, sin sobrepasar el n´ umero formado por las cifras tomadas en el dividendo; este n´ umero hace parte del cociente. ¿Por qu´e? 2.2. Multiplicamos el n´ umero que encontramos por el divisor y lo sustraemos de las cifras tomadas del dividendo. ¿Por qu´e? 2.3. Tomamos el resultado de esta sustracci´on (residuo), le colocamos a la derecha la cifra que sigue a las tomadas en el dividendo y repetimos el paso 2. ¿Por qu´e? 3. Si el n´ umero formado por las cifras tomadas del dividendo es menor que el divisor, tomamos una cifra adicional en el dividendo y efectuamos el paso 2. Cuando no haya m´as cifras disponibles en el dividendo termina el proceso. Ejemplo En base 8 queremos dividir 54706 entre 723 Iniciamos en el paso 1, como 547 es menor que 723, seg´ un el paso 3, debemos encontrar un n´ umero que multiplicado por 723 nos quede lo m´as pr´oximo posible a 5470, sin sobrepasarlo. Un posible n´ umero es el que multiplicado por 7 nos aproxime por defecto a 54; es decir, 6, puesto que 7 × 6 = 52 Ahora debemos multiplicar 6 × 723 = 5362 resultado que debemos sustraer de 5470 para obtener como residuo 106 y primera cifra del cociente 6. Enseguida colocamos a la derecha de 106 la cifra 6, obteniendo 1066 como nuevo dividendo. 72

´ REPRESENTACIONES POSICIONALES DE NUMEROS

Repetimos el proceso y conseguimos como resultado final un cociente de 61 y como residuo 143. Ejercicios 1. Reemplace cada cuadro por el n´ umero que corresponda para que la divisi´on sea correcta en base 7.   

56

 

 

  

6 0

Invente un algoritmo para resolver este problema con divisores de una sola cifra. 2. En base 10 se tiene: 1×8+1= 9 12 × 8 + 2 = 98 123 × 8 + 3 = 987 1234 × 8 + 4 = 9876 12345 × 8 + 5 = 98765 123456 × 8 + 6 = 987654 Construya una tabla semejante en base 7 3. En base 10 se tiene: 1 × 8 + 1 = 9 = 32 3 × 8 + 1 = 25 = 52 6 × 8 + 1 = 49 = 72 10 × 8 + 1 = 91 = 92 73

´ ´ ACTIVIDADES MATEMATICAS PARA EL DESARROLLO DE PROCESOS LOGICOS

4. . (Tomado de Caro37 ) Durante la primera guerra mundial (1914 - 1918), los ingleses distribu´ıan carb´on a diferentes lugares del Reino Unido. A uno de sus municipios llegaron una vez 28 toneladas con instrucciones de repartirlas equitativamente entre 7 corregimientos. El alcalde, un hombre rudo e ignorante, junto con el concejo, decidi´o dividir 28 entre siete; esta funci´on le correspond´ıa al secretario, quien efectu´o la operaci´on de esta manera:

28

7

21

13

0

“Siete en ocho cabe una vez; uno por siete es siete, a ocho, uno; bajo el 2; siete en 21 cabe 3 veces; 3 por 7 es igual a 21, a 21, cero”. Un concejal dijo, “una divisi´on se prueba mediante una multiplicaci´on y ´esta se hace as´ı”: 13 7 7 21 28 y dijo: “siete por una siete; siete por 3, 21 y 7 m´as 21 da 28. No hay duda de que la divisi´on est´a bien hecha”

Pero un campesino estim´o: “La multiplicaci´on es una adici´on repetida”. Y colocando el n´ umero 13 siete veces, sum´o subiendo por la columna de la 37

CARO, V., op. cit. p. 254.

74

´ REPRESENTACIONES POSICIONALES DE NUMEROS

derecha y descolg´andose por la izquierda, as´ı: 13 13 13 13 13 13 13 28 “3 y 3, 6; y 3, 9; y 3, 12; y 3, 15; y 3, 18; y 3, 21; y 1, 22; y 1, 23; y 1, 24; y 1, 25; y 1, 26; y 1, 27; y 1, 28”. Entonces a cada corregimiento le correspondieron 13 toneladas de carb´on. Si hay errores ub´ıquelos y explique por qu´e son errores. Plantee una situaci´ on similar en otra base. 2.2.4.4.

Ecuaciones de la forma ax + b = c

Cuando una divisi´on no es exacta; es decir, queda un residuo distinto de cero, existe una relaci´on entre los t´erminos de ella, que tambi´en se usa para probar si fue realizada de manera correcta. Si escribimos D para el dividendo, d para el divisor, c para el cociente y r para el residuo, la relaci´on es naturalmente38 : D = d×c+r Con lo que hemos aprendido hasta ahora, deber´ıamos estar en capacidad de encontrar uno de los n´ umeros que participan en la relaci´on si conocemos todos los dem´as, esto es resolver ecuaciones de la forma ax+b = c, o de las otras formas que ya hemos resuelto. Ejercicios

1. Proponga y resuelva ecuaciones de la forma ax + b = c 38

Esta relaci´ on es conocida como el algoritmo de la divisi´ on.

75

´ ´ ACTIVIDADES MATEMATICAS PARA EL DESARROLLO DE PROCESOS LOGICOS

2. Reemplace las letras por d´ıgitos para que las operaciones resulten correctas en base 10. ¿Es posible una soluci´on en base 9?, ¿s´ olo una? ABCDE × 4 EDCBA 3. Al multiplicar el n´ umero 2178 por 4 en base 10 se invierte; es decir, el resultado es 8712. Encuentre un n´ umero de 4 d´ıgitos que al multiplicarlo por 9 se invierta. Repita el ejercicio en otra base.

2.3.

Ecuaciones con dos variables

Hasta ahora hemos resuelto ecuaciones del estilo a + x = b, a − x = b, x − a = b, ax = b, a/x = b y x/a = b; con a y b n´ umeros naturales, en diferentes bases num´ericas siempre que ´estas tienen soluci´on dentro del conjunto de los n´ umeros naturales. Si observamos estas ecuaciones, notamos que en cada una de las igualdades no conocemos un n´ umero, ¿c´omo resolvemos ecuaciones en las cuales desconozcamos dos n´ umeros? Es decir, qu´e estrategias proponemos para dar soluci´on a ecuaciones como: x + y = 32 donde x, y y 32 son n´ umeros en base 7. Una idea natural consiste en dar valores a x y resolver la ecuaci´on de una sola variable y, as´ı: Si x = 1, entonces 1 + y = 32 o sea y = 31. Si x = 5, entonces 5 + y = 32 76

´ REPRESENTACIONES POSICIONALES DE NUMEROS

o sea y = 24. Y as´ı sucesivamente para cada valor de x encontramos un valor para y, como lo muestra la siguiente tabla. x

0

1

2

3

4

5

6

10

···

26

30

31

32

y

32

31

30

26

25

24

23

22

···

3

2

1

0

Pero, ¿es aplicable el m´etodo en cualquier ecuaci´on de la forma: ax + by = c , donde a, b, c son n´ umeros naturales? Estudiemos otro ejemplo en busca de formular alguna conjetura. En base siete, resolvamos la ecuaci´on: 3x + 5y = 126 Iniciemos proponiendo una soluci´on por tanteo, luego de varios intentos encontramos las siguientes soluciones: (3, 15), (11, 12), (16, 6), (24, 3)y(32, 0) donde hemos representado con (x0, y0 ) una soluci´on de la ecuaci´on propuesta. Si hacemos una lista con las soluciones: (3, 15) (11, 12) (16, 6) (24, 3) (32, 0) Observamos que los n´ umeros que ocupan el lugar de x0 van aumentando de cinco en cinco: 3 + 5 = 11, 11 + 5 = 16, 16 + 5 = 24, 24 + 5 = 32, 5 es el n´ umero que multiplica a y; en cambio, los n´ umeros que ocupan el lugar de y0 disminuyen de tres en tres: (15 - 3 = 12, 12 - 3 = 6, 6 - 3 = 3, 3 - 3 = 0) y 3 es el n´ umero que multiplica a x, ¡como ya lo debi´o notar! An´alogamente sucede con la primera ecuaci´on que tratamos. Conjeturamos que: 77

´ ´ ACTIVIDADES MATEMATICAS PARA EL DESARROLLO DE PROCESOS LOGICOS

Si (x0 , y0) es soluci´ on de ax + by = c, donde a, b, y c son n´ umeros naturales, on entonces (x0 + b, y0 − a) o (x0 − b, y0 + a) es tambi´en, soluci´on de la ecuaci´ umeros naturales. siempre que y0 − a o y x0 − b sean n´ Esta afirmaci´on la podemos demostrar de la siguiente manera (s´olo demostraremos la primera parte, la otra queda como ejercicio para el lector) Prueba: Como (x0 , y0) es soluci´on de ax + by = c; se cumple que : ax0 + by0 = c Haciendo uso de la propiedad modulativa de la adici´on entre n´ umeros naturales y del convenio correspondiente a que 0 = ab − ab; se tiene: ax0 + by0 + (ab − ab) = c Aplicando ahora la propiedad asociativa y conmutativa de la adici´on; tenemos: ax0 + (ab + by0) − ab = c Asociando nuevamente, (ax0 + ab) + (by0 − ab) = c Conmutando ab y utilizando la propiedad distributiva de la multiplicaci´on reumero specto a la adici´on y a la sustracci´on, esta u ´ltima, s´olo si y0 − a es un n´ natural; se concluye que: a(x0 + b) + b(y0 − a) = c Lo cual significa que (x0 +b, y0 −a) es tambi´en soluci´on de la ecuaci´on inicialmente dada.

2.3.1.

Representaci´ on gr´ afica de las soluciones

Podemos obtener otra representaci´on de las soluciones de las ecuaciones que estamos considerando, si dibujamos un par de semirrectas con origen com´ un en 78

´ REPRESENTACIONES POSICIONALES DE NUMEROS

un punto O de un plano formando un a´ngulo39 α; a partir del origen en cada semirrecta se elige un punto denominados A y B respectivamente, los segmentos determinados OA y OB se utilizan como unidades de medida y los puntos A y B representan el n´ umero 1 en cada semirrecta, luego se replica cada uno de estos segmentos en la semirrecta correspondiente para obtener representaciones de los n´ umeros naturales.

Si en una semirrecta dibujamos los valores de x y en la otra los de y, una soluci´on de la ecuaci´on ax + by = c; la representamos con una pareja de n´ umeros naturales (x0, y0 ) que en el plano corresponde a la intersecci´on de las rectas paralelas40 a cada una de las semirrectas dadas trazadas desde los puntos x0 , y0 .

En el caso de nuestro u ´ltimo ejemplo, las soluciones de la ecuaci´on

En el caso de nuestro u ´ltimo ejemplo, las soluciones de la ecuaci´on

3x + 5y = 126

est´an dadas por los puntos

(3, 15), (11, 12), (16, 6), (24, 3) y (32, 0)

que representamos gr´aficamente:

39

Generalmente, se escoge un ´angulo recto pero esta condici´on no es necesaria. Esta condici´on tampoco es necesaria; pueden escogerse rectas perpendiculares, o de otra forma. 40

79

´ ´ ACTIVIDADES MATEMATICAS PARA EL DESARROLLO DE PROCESOS LOGICOS

y 16  15 14 13 12 11 10 6 5 4

(3, 15)









(11, 12)







3x + 5y = 126



(16, 6)









(24, 3)

3 2





1 























2

4

6

11

13

15

20

22

24

26

31

(32, 0)





33

Observamos que todos los puntos resultan colineales. Ejercicio 1. Halle las soluciones de las siguientes ecuaciones y graf´ıquelas, ¿qu´e encuentra?: a) 4x − 3y = 1, en base cinco. b)

12x − y = 20, en base seis.

c) 2y − 5x = 245, en base seis. 2. Con base en los resultados obtenidos en 1, haga una conjetura en la que determine c´ omo encontrar las soluciones de una ecuaci´ on de esta forma dada una soluci´on, e intente probarla. 3. Explique c´omo encontrar una ecuaci´on dadas dos soluciones de ´este. Determine un procedimiento general. 80

x

´ REPRESENTACIONES POSICIONALES DE NUMEROS

Si tenemos una lista de soluciones de una ecuaci´on desconocida como la siguiente, es posible determinar la ecuaci´on sin recurrir a un procedimiento espec´ıfico. Veamos: x

1

2

3

4

5

6

···

17

···

N

y

7

11

15

19

23

27

···

?

···

?

¿cu´al es el valor que le corresponde a y cuando x es 17? Observando la tabla encontramos una relaci´on entre los valores de x y y: Si x = 2 entonces y = 7 + 4 = 11 Si x = 3 entonces y = 7 + 4 + 4 = 7 + 2(4) = 15 Si x = 4 entonces y = 7 + 4 + 4 + 4 = 7 + 3(4) = 19 Si x = 17 entonces y = 7 + 16(4) = 71 Luego, la ecuaci´on que determinan estas soluciones es: y = 7+(x−1)4 = 4x+3 Al graficar las soluciones obtenemos:

y 20

18 16 14 12 10 8



4 2 















y = 4x + 3



6 

























1

2

3

4

5

6

7

8

9

10

Vemos que el corte de la semirrecta con el eje y es 3. 81



x

´ ´ ACTIVIDADES MATEMATICAS PARA EL DESARROLLO DE PROCESOS LOGICOS

Si hacemos la gr´afica de una ecuaci´on de la forma y = mx + b, observamos que el corte de la gr´afica de la ecuaci´on con el eje y es b, pues x = 0. Ejercicios 1. Plantee ecuaciones con dos variables y grafique las soluciones. 2. Estudie el significado de los cambios en m y en b en la gr´ afica de las soluciones de las ecuaciones de la forma y = mx + b.

2.4.

Cambios de base

En lo hecho hasta ahora, hemos enfatizado que un mismo n´ umero puede ser representado con distintos s´ımbolos en sistemas num´ericos diferentes o con s´ımbolos iguales en distintas bases. Debemos desarrollar un procedimiento para cambiar de una base a otra, para poder traducir informaci´on entre bases. En la escuela, para cambiar de una base a otra cualquiera, se acostumbra pasar por la base 10, por familiaridad con ella, pero en realidad esto no es necesario. Si queremos pasar un n´ umero que est´a en base 4 a base 7, lo natural es hacer grupos de 7 unidades, luego grupos de 7 septenas, con los resultantes grupos de 7 biseptenas, etc. Es decir, hacer divisiones sucesivas por 7, pero escribiendo 7 en base 4 o sea 1 3. Ejemplos 1. Para pasar 84 en base 10 a base 2, hacemos divisiones sucesivas y obtenemos: 8 4 0 4 0

2 4 2 0

2 2 1 1

2 1 0 0

82

2 5 1

2 2 0

2 1

´ REPRESENTACIONES POSICIONALES DE NUMEROS

Por lo tanto, 84 en base diez corresponde a 1010100 en base dos. (Hemos resaltado los residuos en cada una de las divisiones y el u ´ltimo cociente para facilitar la lectura). 2. Pasar el n´ umero 322 de base 4 a base 7. a) Dividimos sucesivamente 322 por 7, con el cuidado de escribir ´este u ´ltimo en base 4; es decir, 13. 3 2 2 1 3 -3 2 2 0 0 2 -1 3 -0 0 1 2

1 3 1

b) Escribimos el primer residuo como cifra de las unidades, el segundo como cifra de las septenas, y as´ı sucesivamente hasta terminar con el u ´ltimo cociente. En nuestro caso el n´ umero 322 en base 4 corresponde al n´ umero 112 en base 7. Describiremos otro procedimiento para cambiar de base, usado frecuentemente cuando se quiere pasar de una base cualquiera K a base 10, pero que tambi´en puede usarse para pasar de una base a otra cualquiera, cambiando el 10 por la otra base. 1. Se escribe el n´ umero en base K. 2. Se multiplica cada una de sus cifras (escritas en base 10) por el valor de la posici´on que ocupan (tambi´en escrito en base 10) y se expresa el resultado en base 10. 3. Se adicionan los resultados anteriores y el total es el n´ umero en base 10. Ejemplo Para pasar el n´ umero 1023 de base 5 a base 10, debemos escribir en base 10, el resultado de: 1 × 53 + 0 × 52 + 2 × 5 + 3 × 1 esto es 125 + 0 + 10 + 3 = 138. 83

´ ´ ACTIVIDADES MATEMATICAS PARA EL DESARROLLO DE PROCESOS LOGICOS

Y si queremos pasarlo a base 8 el resultado es 175 8 + 0 8 + 12 8 + 3 8 = 212 8 Este procedimiento para cambiar de base utiliza multiplicaciones y el anterior utiliza divisiones; si ´estas son operaciones inversas, ¿c´ omo explica que se utilicen para el mismo prop´ osito? Cuando un n´ umero se escribe como la suma de los productos de cada una de sus cifras por la potencia de la base que indica su posici´on, se dice que el n´ umero est´a escrito en forma polin´ omica. Por ejemplo, 10 1101 2 = 1 × 10 11 2 + 1 × 10 2 + 0 × 10 2 + 1 × 1

En las representaciones posicionales, a diferencia de las dem´as, el cero, que es tan u ´til en la escritura de un n´ umero en una base cualquiera, tambi´en aporta dificultades. La primera de ellas es la imposibilidad de dividir por ´el, ya que si lo hacemos introducimos contradicciones a nuestro sistema como se muestra a continuaci´on.

2.5.

Un problema l´ ogico: la divisi´ on por 0

Si queremos repartir 0 cosas en cualquier n´ umero de grupos, obviamente en cada grupo van a quedar 0 cosas, esto significa que 0 dividido por cualquier n´ umero es 0. Pero, ¿ qu´e sucede con el problema inverso?; si queremos repartir 2 cosas en 0 grupos, ¿cu´antas cosas quedan en cada grupo? Incluso si le damos la vuelta al problema y buscamos un n´ umero (cociente) que multiplicado por otro (divisor) nos d´e como resultado el dividendo, nos vemos en dificultades, pues no existe ning´ un n´ umero que multiplicado por 0 nos d´e 2. Supongamos, por un momento, que no nos damos por vencidos e intentamos encontrar un n´ umero x que resuelva el problema (este es otro truco muy usado ´ por los matem´aticos, sobre todo en Algebra); es decir, un x tal que: 2/0 = x 84

´ REPRESENTACIONES POSICIONALES DE NUMEROS

entonces se tendr´ıa que x·0=2 pero esto no puede ser porque x · 0 = x · (0 + 0) = x·0+x·0 que puede escribirse como: x·0= x·0−x·0 de donde concluimos que x·0=0 y se tendr´ıa que ¡¡ 0 = 2 !! lo que nos destrozar´ıa el coraz´on. Una forma m´as sutil de presentar las dificultades que implica dividir por cero, aparece en el siguiente falso razonamiento: supongamos que x e y son dos n´ umeros y que: x=y entonces, si multiplicamos ambos lados de la ecuaci´on por x, obtenemos x2 = xy Y si sustraemos en ambos lados y 2 tenemos x2 − y 2 = xy − y 2 escribiendo ambos miembros de la igualdad como productos conseguimos: (x + y)(x − y) = y(x − y) 85

´ ´ ACTIVIDADES MATEMATICAS PARA EL DESARROLLO DE PROCESOS LOGICOS

y dividiendo en ambos miembros de la ecuaci´on por (x − y) llegamos a que x+y =y y como x = y entonces y+y =y de donde 2y = y y si suponemos que y es distinto de cero, podemos dividir por y, obteniendo que ¡¡ 2 = 1 !! lo que es rid´ıculo. En conclusi´on, la divisi´on por 0, introduce en matem´aticas contradicciones que romper´ıan su coherencia l´ogica interna; por lo tanto, debemos cuidarnos de efectuar tales divisiones. Por todas estas dificultades, no debemos infringir el

Primer mandamiento de la aritm´etica: Nunca dividir´as por 0

86

CAP´ITULO 3 LAS OPERACIONES SUPERIORES ´ DE LA MATEMATICA 3.1.

Potenciaci´ on

La potenciaci´on 1 es una operaci´on que consiste en repetir un n´ umero2 llamado umero llamado exponente y multiplicar base 3 tantas veces como lo indique otro n´ para obtener un resultado que llamamos potencia. Por ejemplo, en base 7

53 = 236

La base es 5, el exponente 3 y la potencia 236. Esta expresi´on significa que 5 × 5 × 5 = 236 1

Los primeros que aplicaron la elevaci´ on a potencia fueron los sacerdotes mesopot´amicos, quienes empleaban una tabla de cuadrados para multiplicar dos n´ umeros, restando el cuadrado de su promedio, del cuadrado de su semidiferencia. Los griegos estudiaban especialmente los cuadrados y cubos. Diofanto, siglo III (D.C), ide´ o la notaci´ on de las potencias escribiendo x, xx, xxx, etc, para expresar la primera, segunda, tercera potencias de x. Renato Descartes (1596 - 1650), introdujo la notaci´ on x, x2 , x3 , etc. 2 En este contexto, la palabra n´ umero hace referencia a los que hemos construido en el proceso de contar, sin importar su representaci´ on, incluyendo el cero. Estos n´ umeros los hemos llamado n´ umeros naturales. 3 La base de un sistema num´erico y la base de la operaci´on potenciaci´ on, naturalmente no son lo mismo y casi siempre el contexto en el que se usa aclara su sentido.

87

´ ´ ACTIVIDADES MATEMATICAS PARA EL DESARROLLO DE PROCESOS LOGICOS

El exponente lo escribimos, seg´ un la costumbre en la parte superior derecha de la base. Nuevamente, como en la multiplicaci´on, las expresiones a0, a1 no tienen sentido, pero para evitar mencionar siempre estas excepciones, convenimos en que para todo n´ umero natural a a0 = 1,

si a = 0 y a1 = a

Esto, para preservar las leyes que gobiernan la potenciaci´on, que enunciaremos enseguida.

3.1.1.

Propiedades de la potenciaci´ on

Es f´acil verificar que se tienen las siguientes propiedades elementales: 1. ax · ay = ax+y ax = ax−y ay  y 3. ax = axy 2.

(si a = 0 y x ≥ y)4

Las dos primeras igualdades son consecuencias directas de la definici´on, puesto que si repetimos x veces a y multiplicamos, este resultado lo multiplicamos por y veces a, naturalmente la a aparece repetida x + y veces en la multiplicaci´on. Similarmente, en la divisi´on del producto de x veces a entre el producto de y veces a, siempre que a no sea 0 y que x sea mayor o igual que y, por cada a del a dividendo quitamos una del divisor, puesto que = 1, nos quedan x − y veces a. a La tercera igualdad se deduce f´acilmente de la primera. Por otra parte, si x = y en la ecuaci´on 2, entonces ax 1 = y = ax−y = a0 a Observe el comportamiento del 0 y del 1 en estas propiedades5 . 4 5

La segunda condici´ on es necesaria para que la sustracci´ on tenga sentido. Notemos que la expresi´ on 00 no se ha definido, pues la propiedad 2 no es v´ alida para a = 0.

88

´ LAS OPERACIONES SUPERIORES DE LA MATEMATICA

3.1.1.1.

La Potenciaci´ on no es conmutativa

Existe una analog´ıa evidente entre el proceso de construcci´on de la multiplicaci´on a partir de la suma y de la potenciaci´on a partir de la multiplicaci´on. La suma es conmutativa y la multiplicaci´on tambi´en, ¿no le parece extra˜ no que la potenciaci´on no sea conmutativa, proviniendo de una operaci´on conmutativa? Ejercicio Construya ejemplos de potenciaci´ on y trate de averiguar por qu´e ella no es conmutativa. 3.1.1.2.

La Potenciaci´ on no es asociativa

La potenciaci´on tampoco es asociativa como lo muestra el siguiente contraejemplo, en base 8:  2 3 3 = 3(2 ) 3 En el primer caso tenemos 11 como base y 3 como exponente lo que nos da 1331; la segunda expresi´on tiene 3 como base y 10 como exponente, lo que da por resultado 14641. 3.1.1.3.

Propiedad distributiva de la potenciaci´ on con respecto a la multiplicaci´ on

Ya hemos dicho que la potenciaci´on tiene parecido en su construcci´on con la multiplicaci´on, este comportamiento puede llevarse a´ un m´as lejos. La multiplicaci´on es distributiva con respecto a la suma, en la que tuvo su origen, y la potenciaci´on es distributiva con respecto a la multiplicaci´on; los argumentos son parecidos a los que esbozamos en el caso de la multiplicaci´on, por lo que dejaremos los detalles al lector interesado. Lo anterior significa que si a y b son n´ umeros naturales, entonces se tiene que:  n ab = an bn es decir, que cuando queremos hacer la potencia n de un producto, basta multiplicar las potencias n de los factores.

89

´ ´ ACTIVIDADES MATEMATICAS PARA EL DESARROLLO DE PROCESOS LOGICOS

Ejercicio Haciendo una analog´ıa con la multiplicaci´ on, ¿es posible un razonamiento geom´etrico que explique la ley distributiva de la potenciaci´on con respecto a la multiplicaci´on? Justifique su respuesta. 3.1.1.4.

Propiedad distributiva de la potenciaci´ on con respecto a la divisi´ on

Aunque no lo dijimos expl´ıcitamente, la multiplicaci´on es distributiva tambi´en con respecto a la resta (cuando ´esta se pueda hacer), y por analog´ıa, podemos esperar que la potenciaci´on sea distributiva con respecto a la divisi´on, de la siguiente manera:  a n a n = n b b como en efecto se cumple, siempre que la divisi´on sea posible; es decir, que el resultado sea un n´ umero natural y que el divisor no sea 0. Ejercicios 1. Escriba un ejemplo donde se muestre que no se cumple la propiedad distributiva de la potenciaci´ on respecto a la suma, es decir: No es cierto que

(a + b)n = an + bn

2. La siguiente lista muestra ciertos n´ umeros escritos en base 7; coloque en la casilla  el n´ umero en base 7 que corresponda para lograr en cada caso una igualdad: 3 × 5 = 2 − 1 4 × 6 = 2 − 1 5 × 10 = 2 − 1 12 × 14 = 2 − 1 25 × 30 = 2 − 1 43 × 45 = 2 − 1 Escriba una lista similar en base 4. ¿Puede generalizarse esto a cualquier base? 90

´ LAS OPERACIONES SUPERIORES DE LA MATEMATICA

Repita el ejercicio con las siguientes listas: En base 7 5 × 1 = 2 − 4 6 × 2 = 2 − 4 10 × 3 = 2 − 4 11 × 4 = 2 − 4 12 × 5 = 2 − 4 En base 10, 1 × 8 + 1 = 9 = 32 3 × 8 + 1 = 25 = 52 6 × 8 + 1 = 49 = 72 10 × 8 + 1 = 81 = 92 Encuentre un modelo general6 . 3. Algunas tareas en matem´aticas pueden parecer muy dispendiosas y, sin embargo, su soluci´on puede obtenerse de manera relativamente sencilla si observamos algunas regularidades. Por ejemplo: ¿cual es la cifra de las umero est´a escrito en base 10?, ¿y si est´ a en base unidades de 31999 si el n´ 9?, ¿ en otras bases? Si observa la siguiente secuencia en base 10, es posible encontrar una pista: 31 32 33 34 35 36 37 38 6

=3 =9 = 27 = 81 = 243 = 729 = 2187 = 6561

Los resultados obtenidos en estos ejercicios son identidades algebraicas, ellas son v´alidas en cualquier base y para cualquier n´ umero.

91

´ ´ ACTIVIDADES MATEMATICAS PARA EL DESARROLLO DE PROCESOS LOGICOS

4. . Encuentre el n´ umero de d´ıgitos de 412 × 520 escrito en base 10. 5. Encuentre un n´ umero menor que 9, tal que aumentado en 1 da el doble de un cuadrado y cuyo cuadrado aumentado en 1 da otra vez el doble de un cuadrado. ¿El resultado depende de la base?

3.2.

Radicaci´ on

En la potenciaci´on participan como ya dijimos tres n´ umeros: la base, el exponente y la potencia. Si existen (esta condici´on es muy importante) tres n´ umeros a, b y c, que satisfacen la relaci´on: ab = c y conocemos dos de ellos, podemos obtener el otro. La operaci´on para determinar c conociendo a y b la hemos llamado potenon ciaci´on. El proceso7 que permite averiguar a conociendo b y c se llama radicaci´ y la escribimos con un nuevo s´ımbolo llamado signo radical, el n´ umero c es llamado cantidad subradical, b es el ´ındice o grado de la ra´ız y a es la ra´ız b de c; en s´ımbolos8 : √ b c=a Insistimos en que el significado de la expresi´on anterior es exactamente el mismo de la expresi´on ab = c Por ejemplo, en base 10 se tiene que √ 3 64 = 4 significa que 43 = 64 El cero de nuevo ocasiona dificultades, pues 7

Al igual que la sustracci´ on y la divisi´ on, la radicaci´ on no es una operaci´ on entre n´ umeros naturales, pues no todas las ra´ıces tienen soluci´on; aqu´ı s´olo tratamos con las que la tienen. 8 Cuando b = 2, es costumbre no escribirlo.

92

´ LAS OPERACIONES SUPERIORES DE LA MATEMATICA

Si b = 0 y c = 1 no existe un u ´nico n´ umero a como resultado de la operaci´on, 0 puesto que, a = 1 para todo a = 0. Si b = 0 y c = 1 no existe un n´ umero a tal que a0 = c. Esto significa que para el caso b = 0 la operaci´on no est´a definida, o sea que √ 0 c no existe para ning´ un n´ umero natural c = 1 y para c = 1 no es u ´nico. Ejercicio Para cualquier n´ umero natural c ¿Qu´e significa la expresi´ on

3.2.1.

√ 1

c?

Propiedades de la radicaci´ on

Como la radicaci´on es s´olo otra forma de escribir la potenciaci´on, sus propiedades, pueden deducirse de las de aquella; por ejemplo, se tiene la: 3.2.1.1.

Propiedad distributiva de la radicaci´ on con respecto a la multiplicaci´ on

La ra´ız n de un producto es el producto de las ra´ıces n de cada uno de los factores.  √ √ n n (ab) = n a b Esta propiedad es la correspondiente a (ab)n = an bn y se deduce de ella; veamos: √ √ si llamamos w = n a y z = n b, tenemos que: wn = a y z n = b, por lo tanto ab = wn z n  n = wz 93

´ ´ ACTIVIDADES MATEMATICAS PARA EL DESARROLLO DE PROCESOS LOGICOS

lo que significa que: wz =

√ n

ab

o sea √ √ √ n n n a b = ab Siempre y cuando las ra´ıces existan; es decir, sean n´ umeros naturales. 3.2.1.2.

Propiedad distributiva de la radicaci´ on con respecto a la divisi´ on

An´alogamente sucede con la divisi´on  n

√ n a a = √ n b b

Resaltamos de nuevo que estas propiedades se cumplen solamente cuando las ra´ıces y los cocientes est´en definidos, en particular que las ra´ıces y los cocientes sean n´ umeros naturales y los divisores sean distintos de 0. Es deseable que el lector se acostumbre a las distintas notaciones y ejercite el tr´ansito de una a la otra, esto puede ayudarle mucho en su comprensi´on. Para ello le sugerimos que verifique las siguiente propiedad:  n

√ m

a=

√ a

nm

Esta propiedad es an´aloga a la propiedad 3 de la potenciaci´on. Ejercicio ¿Existen propiedades para la radicaci´ on an´ alogas a las propiedades 1 y 2 de la potenciaci´on? Justifique. 94

´ LAS OPERACIONES SUPERIORES DE LA MATEMATICA

3.2.2.

C´ alculo de ra´ıces

Hemos presentado algunas propiedades de la radicaci´on, pero no tenemos, a´ un, m´etodo alguno para calcular ra´ıces; por ejemplo, si quisieramos evaluar9 cu´anto es  5 31446 ( 7) estamos ante un problema que posiblemente no tenemos idea de c´omo resolver. Si traducimos el problema en t´erminos de potenciaci´on, lo que queremos es encontrar un n´ umero x tal que x5 = 31446 ( 7) En esta forma, una primera instancia de soluci´on es ensayar con un n´ umero cualquiera, lo elevamos a la quinta potencia, si nos da un resultado m´as grande umero menor y si nos da menor enque 31446 7 , ensayamos de nuevo con un n´ sayamos con uno m´as grande. Este proceso puede ser demorado pero innegablemente funciona. 3.2.2.1.

Ecuaciones de la forma x2 = b

Ilustremos el procedimiento anterior mediante un problema m´as simple, resolvamos por ejemplo la ecuaci´on10: 2124 (5) = x2 Para encontrar x, observemos que el cuadrado de x termina en 4, lo que significa que la u ´ltima cifra de x debe ser 2 ´o 3 y x debe ser de dos cifras, puesto que su cuadrado es de 4 cifras. Con estas pistas podemos ensayar, por ejemplo, con x = 22 (5) , en cuyo ca 2 so obtenemos 22 (5) = 2034 (5) ; como este resultado es inferior al buscado, ensayamos con uno mayor, por ejemplo x = 32 (5) y obtenemos el resultado correcto. 9

Hemos dicho que no toda ra´ız de n´ umeros naturales es un n´ umero natural, por eso es necesario estar seguros de que la soluci´on existe antes de plantear el ejercicio. Realmente lo que hacemos es calcular una potenciaci´on y luego la reescribimos como radicaci´on ocultando la base. 10 Cuando el m´ aximo exponente de la inc´ ognita es 2, las ecuaciones se llaman Ecuaciones de segundo grado o Ecuaciones cuadr´ aticas.

95

´ ´ ACTIVIDADES MATEMATICAS PARA EL DESARROLLO DE PROCESOS LOGICOS

Este proceso nos muestra, entre otras cosas, que si un n´ umero es el cuadrado de otro es porque el n´ umero se puede escribir como un producto; es decir, 2124 (5) = 32 (5) × 32 (5) , lo que nos induce a pensar que el proceso de calcular ra´ıces est´a vinculado con el problema de encontrar factores. ❆ ❆ ❆ ❆ ❆ ❆ ❆ ❆ ❆ ❆ ❆ ❆ ❆ ❆ ❆ ❆ ❆ ❆ Algunos problemas en matem´aticas est´an relacionados con otros problemas que aparentemente no tienen que ver con el primero, pero si la relaci´on existe, puede ser m´as f´acil resolver uno de ellos y usarlo como ayuda para resolver el otro. ❆ ❆ ❆ ❆ ❆ ❆ ❆ ❆ ❆ ❆ ❆ ❆ ❆ ❆ ❆ ❆ ❆ ❆ Como la tarea que tenemos es escribir un n´ umero como el producto de otros, ocup´emonos entonces de este problema; este proceso se conoce como factorizaci´on. 3.2.2.1.1.

Los n´ umeros primos

Cualquier n´ umero que pensemos lo podemos escribir, algunos de varias formas, como un producto de otros n´ umeros, por ejemplo: 8 = 4 × 2 = 8 × 1 = 2 × 2 × 2 = 23 35 = 7 × 5 = 35 × 1 Pero existen ciertos n´ umeros que s´olo se pueden escribir como el producto de dos n´ umeros distintos, ´el mismo y 1, a estos n´ umeros los llamamos n´ umeros primos, por ejemplo: 2= 2×1 3= 3×1 5= 5×1 7= 7×1 Esto significa que otros n´ umeros est´an compuestos por factores diferentes a 1 y a s´ı mismos; debemos entonces encontrar una manera de de saber cu´ales son dichos factores. Esto es averiguar cu´ales n´ umeros son primos y cu´ales no. Una manera artesanal de hallar los n´ umeros primos consiste en escribir una tabla desde 1 hasta el n´ umero que se desee, luego tachar los n´ umeros de 2 en 2 96

´ LAS OPERACIONES SUPERIORES DE LA MATEMATICA

a partir de 2 (sin incluirlo11); con esto eliminamos los que est´an compuestos por el 2,; de 3 en 3 a partir de 3 (sin incluirlo), de 5 en 5, etc. Esta tabla se conoce como la Criba de Erat´ ostenes. Los n´ umeros que no han sido tachados (salvo el 1) son los n´ umeros primos; en la siguiente tabla hacemos el proceso, en base 14, hasta 100. 1 ✗ 11 21 31 ✗ 41 51 ✗ 61 ✗ 71 81 91 ✗ A1 ✗ B1 ✗ C1 ✗ D1

2 ✗ 12 ✗ 22 ✗ 32 ✗ 42 ✗ 52 ✗ 62 ✗ 72 ✗ 82 ✗ 92 ✗ A2 ✗ B2 ✗ C2 ✗ D2

3 13 23 ✗ 33 43 53 ✗ 63 73 ✗ 83 ✗ 93 A3 B3 C3 ✗ D3

✗ 4 ✗ 14 ✗ 24 ✗ 34 ✗ 44 ✗ 54 ✗ 64 ✗ 74 ✗ 84 ✗ 94 ✗ A4 ✗ B4 ✗ C4 ✗ D4

5 15 ✗ 25 35 45 ✗ 55 65 75 ✗ 85 95 ✗ A5 ✗ B5 C5 ✗ D5

✗ 6 ✗ 16 ✗ 26 ✗ 36 ✗ 46 ✗ 56 ✗ 66 ✗ 76 ✗ 86 ✗ 96 ✗ A6 ✗ B6 ✗ C6 ✗ D6

7 ✗ 17 ✗ 27 ✗ 37 ✗ 47 ✗ 57 ✗ 67 ✗ 77 ✗ 87 ✗ 97 ✗ A7 ✗ B7 ✗ C7 ✗ D7

✗ 8 ✗ 18 ✗ 28 ✗ 38 ✗ 48 ✗ 58 ✗ 68 ✗ 78 ✗ 88 ✗ 98 ✗ A8 ✗ B8 ✗ C8 ✗ D8

✗ ✗ ✗ 9 A B C D ✗ 1C ✗ ✗ 1B ✗ 1D 19 1A ✗ 2C ✗ 2B ✗ 2D 29 2A ✗ 3A ✗ ✗ 3B 3C ✗ 3D 39 ✗ 4A ✗ ✗ 4B 4C ✗ 4D 49 ✗ 5C ✗ 5B ✗ 5D 59 5A ✗ 6A ✗ 6C ✗ 6B ✗ 6D 69 ✗ 7C ✗ ✗ 7B ✗ 7D 79 7A ✗ 8A ✗ 8C ✗ ✗ 8B ✗ 8D 89 ✗ ✗ ✗ 99 9A 9B 9C 9D ✗ AB AC ✗ AD A9 AA ✗ BC ✗ BB ✗ BD B9 BA ✗ CA ✗ CB CC ✗ CD C9 ✗ ✗ DB DC ✗ DD D9 DA

✗ 10 ✗ 20 ✗ 30 ✗ 40 ✗ 50 ✗ 60 ✗ 70 ✗ 80 ✗ 90 ✗ A0 ✗ B0 ✗ C0 ✗ D0 ✗ 100

Este m´etodo que aprendimos en la escuela es u ´til para n´ umeros peque˜ nos; pero si se trata de averiguar cu´ando es primo un n´ umero n relativamente grande, no es procedente hacer la criba y tenemos que recurrir a otras herramientas. Podr´ıamos ensayar a dividir el n´ umero por 2, para ver si est´a compuesto por 2, luego dividirlo por 3, y as´ı sucesivamente hasta (n − 1). Pero la tarea sigue siendo molesta, pues debemos hacer un gran n´ umero de divisiones en el caso de n´ umeros grandes. Una salida f´acil es contratar un empleado, con suficientes habilidades para que no cometa errores y que adem´as sea suficientemente veloz. En la actualidad disponemos de m´aquinas que hacen la tarea por nosotros, por ejemplo, las calculadoras manuales o, si se quiere un poco m´as de efectividad, una computadora. 11

No empezamos a contar de uno en uno a partir de uno por razones obvias, ni incluiremos al uno entre los n´ umeros primos.

97

´ ´ ACTIVIDADES MATEMATICAS PARA EL DESARROLLO DE PROCESOS LOGICOS

Sin embargo, ellas requieren que les sean dados cada uno de los pasos a seguir; esto es, debemos elaborar un programa que le permita a la m´aquina determinar si un n´ umero es primo o no. El programa PRIMOS1 cuyo c´odigo en lenguaje Pascal (versi´on 7) aparece en el ap´endice 1, permite saber si un n´ umero dado es primo o no; el programa PRIMOS2 da una lista de los n´ umeros primos menores o iguales a un n´ umero dado. Este tambi´en aparece en el ap´endice 1. Ejercicios 1. ¿Cu´al es el procedimiento que utilizan los programas PRIMOS1 Y PRIMOS2 para decidir si un n´ umero es primo o no ? 2. “¿Cu´antas ovejas tienes a tu cuidado?”, preguntaron a un pastor; ´el contest´o: “No lo s´e fijamente porque apenas puedo contar hasta diez. Pero si agrupo mis ovejas por pares, me sobra una; si las agrupo de tres en tres, de cuatro en cuatro, de cinco en cinco o de seis en seis, siempre me sobra una ; en cambio, si las agrupo de siete en siete, no me sobra ninguna. Esto es lo u ´nico que s´e ”12 .¿Cu´antas ovejas tiene el pastor? 3.2.2.1.2.1.

Factores primos y divisibilidad

Si un n´ umero (distinto de 1 y de 0) no es primo, podemos escribirlo de una u ´nica forma, salvo el orden de los factores, como un producto de n´ umeros primos; este enunciado se conoce como el teorema fundamental de la aritm´etica 13. Nos ocuparemos ahora de encontrar alguna forma para descomponer un n´ umero en sus factores primos; por ejemplo podemos comenzar dividiendo el n´ umero dado por n´ umeros primos 2, 3, 5, etc. hasta conseguir residuo 0; cuando esto ocurra, tenemos un factor primo. Si el n´ umero resulta ser m´ ultiplo de 2 decimos que es un n´ umero par ; los 14 n´ umeros naturales que no son pares se llaman n´ umeros impares . 12

CARO, V., op cit., p. 247. BURTON, W J., Teor´ıa de los N´ umeros, Trillas, 1969, p. 29. 14 Los egipcios y los griegos fueron los primeros en reconocer esta clasificaci´ on. 13

98

´ LAS OPERACIONES SUPERIORES DE LA MATEMATICA

Muchas veces no es necesario hacer una divisi´on para saber si un n´ umero es m´ ultiplo de otro o no. Por ejemplo, sabemos que el n´ umero 0 es par y se escribe de la misma forma en todas las bases. En base diez un n´ umero es par, si su cifra de las unidades lo es, esto es debido a que las cifras de las decenas, centenas, etc. est´an multiplicadas por potencias de diez y estas son n´ umeros pares, de modo que la u ´nica cifra que determina si un n´ umero es par o no, es la cifra de las unidades. Ejercicios 1. ¿Es v´alido el anterior razonamiento en otras bases pares? Observe la tabla 2 y enuncie un criterio de divisibilidad por 2 en base par. 2. En la tabla 2 observe los n´ umeros pares en base 3, 5, 7, 9. Enuncie alguna regularidad N´ umero de la Base 16

10

9

8

7

6

5

4

3

2

2

2

2

2

2

2

2

2

2

10

4

4

4

4

4

4

4

10

11

100

6

6

6

6

6

10

11

12

20

110

8

8

8

10

11

12

13

20

22

1000

A

10

11

12

13

14

20

22

121

1010

C

12

13

14

15

20

22

30

220

1100

E

14

15

16

20

22

24

32

222

1110

10

16

17

20

22

24

31

100

1001

10000

12

18

20

22

24

30

33

102

1010

10010

14

20

22

24

26

32

40

110

1012

10100

16

22

24

26

31

34

42

112

1021

10110

18

24

26

30

33

40

44

120

1100

11000

1A

26

28

32

35

42

101

122

1102

11010

1C

28

31

34

40

44

103

130

1111

11100

1E

30

33

36

42

50

110

132

1120

11110

20

32

35

40

44

52

112

200

1122

100000

99

´ ´ ACTIVIDADES MATEMATICAS PARA EL DESARROLLO DE PROCESOS LOGICOS

Seguramente, en el ejercicio anterior usted encontr´o un criterio de divisibilidad por 2 en cualquier base impar. Intentemos una explicaci´on para lo que sucede en base 3. Un n´ umero cualquiera se escribe como suma de potencias de 3, y ninguna de estas potencias es un n´ umero par15 , pero si tenemos una lista de n´ umeros impares y deseamos obtener de ella una lista de n´ umeros pares, no es muy dif´ıcil encontrar una soluci´on; podemos, por ejemplo16, a cada uno restarle 1 (o sumarle 1, o cualquier n´ umero impar), pensando inicialmente en base 10, que es donde sabemos cuando un n´ umero es par; as´ı, observamos que: 31 − 1 = 2 32 − 1 = 8 33 − 1 = 26 34 − 1 = 80 Con esto podemos escribir un n´ umero cualquiera en base 3, en forma polin´omica donde cada una de las potencias es par, si modificamos un poco su escritura. Por ejemplo, el n´ umero 221121 en base 3 se escribe: 221121 = 2 × 35 + 2 × 34 + 1 × 33 + 1 × 32 + 2 × 3 + 1 × 1 pero se puede escribir como: 221121 = 2 × (35 − 1) + 2 × (34 − 1) + 1 × (33 − 1) + 1 × (32 − 1) + 2 × (3 − 1) + 1 + (2 + 2 + 1 + 1 + 2) si adicionamos lo que quitamos en cada una de las potencias, que es lo que aparece en el u ´ltimo par´entesis. Tenemos ahora una expresi´on donde cada una de los t´erminos es un n´ umero par m´as una suma adicional, que es la suma de las cifras del n´ umero. El n´ umero ser´a par si esta u ´ltima suma es par. Luego, 221121 (3) no es par. 15

Verifique con varias potencias de 3 y vea que ninguno de los n´ umeros hallados es par. Las reglas que enunciamos aqu´ı, no son teoremas matem´aticos hasta que no se haga una demostraci´on de ellos, por ahora son intuiciones cuya prueba requiere de una herramienta que desarrollaremos un poco m´ as adelante llamada el principio de inducci´ on matem´ atica. 16

100

´ LAS OPERACIONES SUPERIORES DE LA MATEMATICA

En resumen, para que un n´ umero sea par en base 3, es necesario que la suma 17 de sus cifras sea par . No se puede negar que el truco es ingenioso. Ejercicios 1. Enuncie criterios de divisibilidad por 3 en base 3, 6, 9, y los dem´ as m´ ultiplos de 3. 2. Observe una lista de m´ ultiplos de 3 en bases 4, 7, 10, 13, etc. Enuncie un criterio de divisibilidad por 3 en estas bases. 3. Escriba listas de m´ ultiplos de 3 en bases 5, 8, 11, etc. Enuncie criterios de divisibilidad por 3 en estas bases 18 . 4. ¿Cu´antos n´ umeros naturales menores que 200 no son divisibles por 2, por 3, ni por 5? 5. ¿Cu´al es el menor n´ umero que al dividirlo por 2, 3, y 4 deja residuo 1 y es divisible por 51? 6. ¿Cu´anto valen x y b en 

3(230 + x)

2

= 492b04 ?

3.2.2.1.2.2. Descomposici´ on de un n´ umero en factores primos Con alguno de los procedimientos descritos, podemos encontrar todos y cada uno de los factores primos de un n´ umero dado y escribir ´este como un producto de n´ umeros primos.

17

El resultado es v´ alido en cualquier base impar, su demostraci´ on se basa en la afirmaci´on “(2n + 1)k − 1 es par para todo n´ umero natural n y k”, que se demuestra por inducci´on. 18 Los hind´ ues conoc´ıan criterios de divisibilidad por 3, 7 y 9, naturalmente en base 10.

101

´ ´ ACTIVIDADES MATEMATICAS PARA EL DESARROLLO DE PROCESOS LOGICOS

Ejemplo 1. En base 10 tenemos que: 48 = 2 × 2 × 2 × 2 × 3 = 24 × 3 8 = 23 27 = 33 15 = 3 × 5 24 = 23 × 3 36 = 22 × 32 120 = 23 × 3 × 5 2. En base 7, el n´ umero 31446 lo podemos escribir como producto de n´ umeros primos, efectuando las divisiones sucesivas por 2 (ya que la suma de sus cifras es un n´ umero par), luego por 3, de la forma siguiente: 31446 14223 5445 2556 1263 465 144 36 12 3 1

2 2 2 2 2 3 3 3 3 3

de manera que 31446 = 25 × 35 Si utilizamos la propiedad distributiva de la potenciaci´on respecto al producto obtenemos:  5 31446 = 2 × 3 y hemos resuelto, el problema de hallar la ra´ız 5 de 31446 en base 7. 102

´ LAS OPERACIONES SUPERIORES DE LA MATEMATICA

3. Calculemos

 1020100 (3)

El problema es equivalente a hallar x tal que: x2 = 1020100 (3) Descomponemos 1020100 (3) en factores primos y obtenemos: 1020100 (3) 121200 (3) 22100 (3) 2210 (3) 221 (3) 12 (3) 1

2 (3) 2 (3) 10 (3) 10 (3) 12 (3) 12 (3)

De esta manera, 1020100 (3) = 22 × 102 × 122 y por tanto tenemos que: x2 = 22 × 102 × 122 y as´ı: x=

 1020100(3) = (2 × 10 × 12) (3) = 1010 (3)

Ejercicios 1. Prop´ongase ejercicios de la forma   12 1012 (3) , 6 31446 (7)

,

 3

6457 (8)

,

etc.

hasta que adquiera habilidad para calcular ra´ıces en cualquier base. 2. Combinando lo que hemos hecho hasta ahora, proponga ejercicios para resolver ecuaciones de la forma: ax2 + b = c en cualquier base. 103

´ ´ ACTIVIDADES MATEMATICAS PARA EL DESARROLLO DE PROCESOS LOGICOS

3.2.2.2.

Ecuaciones de la forma ax2 + bx = c

Abordemos ahora un problema que requiere de mayor elaboraci´on, consiste en resolver ecuaciones de la forma: ax2 + bx = c Este problema fue tratado por los griegos, (Euclides) y por los a´rabes19 (Alkhowarizmi y Tabit Ben Qurra) y ambos le dieron ingeniosas soluciones, por cierto muy parecidas. Describiremos el m´etodo griego (pero le sugerimos que antes de seguir intente usted alg´ un mecanismo de soluci´ on) utilizando el siguiente ejemplo: Encontrar un n´ umero x tal que x2 + 4x = 140. a) Si interpretamos x como el lado de un cuadrado, x2 ser´a su a´rea y 4x puede interpretarse como el a´rea de un rect´angulo de lados 4 y x respectivamente; es decir, que la cantidad x2 + 4x es el ´area de la figura 1:

x

x

4

x2

4x

Figura 1 b) La chispa de ingenio20 est´a en que este dibujo puede cambiarse por otro de la misma a´rea de la siguiente forma: Dividamos el rect´angulo de a´rea 4x en dos rect´angulos de a´rea 2x y coloquemos uno de ellos a la derecha del de a´rea x2 y el otro debajo para formar la siguiente figura: 19

Van der WAERDEN, B. L., A History of Algebra, Springer, Berlin, 1985. No se preocupe si no se le ocurri´o antes esta idea; muchos sabios de la humanidad tardaron a˜ nos en encontrarla. (¡Y parece tan simple!) 20

104

´ LAS OPERACIONES SUPERIORES DE LA MATEMATICA

x

x

2

x2

y

2 Figura 2 c) Esta figura no es un cuadrado, pero podemos completarlo agregando en la esquina inferior derecha, un cuadrado, cuya a´rea conocemos. x

x

2

x2

y

4

2 Figura 3

d) De esta manera, el a´rea de la regi´on sombreada equivale a 140 (pues corresponde a x + 4x) unidades cuadradas y el a´rea del cuadrado en blanco es 4 unidades cuadradas; es as´ı, como el a´rea total corresponde a 140 + 4 = 144, luego el lado del cuadrado grande, llam´emoslo y, es 12, entonces x = 10 unidades, y hemos solucionado la ecuaci´on inicial. Ejercicio Extienda este m´etodo para resolver ecuaciones generales de la misma forma. 105

´ ´ ACTIVIDADES MATEMATICAS PARA EL DESARROLLO DE PROCESOS LOGICOS

3.2.2.3.

Ecuaciones de la forma x2 + c = bx

Intentemos un m´etodo an´alogo a ´este para resolver ecuaciones de la forma: x2 + c = bx Empecemos, por ejemplo, con la ecuaci´on: x2 + 21 = 10x a) Inicialmente dibujamos un cuadrado de lado x, cuya a´rea representamos con x2. b) Dibujamos un rect´angulo que represente 21 unidades cuadradas, pero repartidas, de manera que uno de los lados sea x. (Figura 4) c) El rect´angulo compuesto por x2 + 21 tiene como a´rea 10x, como se ve en la ecuaci´on original; por lo tanto, uno de los lados del rect´angulo tiene como longitud 10 unidades, puesto que el otro mide x. d) Si trazamos un segmento de tal manera que divida en dos partes iguales al rect´angulo cuyo lado es 10, obtenemos dos casos, veamos el primero: Que x sea m´as peque˜ no o igual a 5 (esto no lo podemos saber desde el principio, porque desconocemos el valor de x, pero s´ı podemos dibujarlo como ayuda); este caso, se presenta en la figura 4:

x

x2

c = 21

10 Figura 4 Para encontrar el valor de x, procedemos de manera similar a la del problema anterior: primero completamos un cuadrado de lado 5 que incluya al cuadrado de lado x como en la figura 5: 106

´ LAS OPERACIONES SUPERIORES DE LA MATEMATICA

x

x2

(5 − x)

2

5

5 Figura 5 Este cuadrado est´a compuesto por dos rect´angulos de igual a´rea y por dos cuadrados, uno de a´rea x2 y el otro de a´rea (5 − x)2. Si sumamos las a´reas de este u ´ltimo con 21 que es el a´rea del rect´angulo c, obtenemos el a´rea del cuadrado de lado 5 (Figura 6), porque: Si llamamos a al a´rea del rect´angulo que est´a contiguo al cuadrado de lado x, tenemos que x2 + a = 21 − a

x

21 − a

x2

(5 − x)

2

5 Figura 6

107

5

´ ´ ACTIVIDADES MATEMATICAS PARA EL DESARROLLO DE PROCESOS LOGICOS

Esto significa que la suma de las ´areas de los dos rect´angulos contiguos al cuadrado de a´rea x2 (los cuadrados denotados con a) con el a´rea del cuadrado de lado x es 21, lo que en s´ımbolos escribimos como: (x2 + a) + a = (21 − a) + a = 21 Por lo tanto, el a´rea del cuadrado de lado (5 − x) m´as 21 nos da el a´rea del cuadrado de lado 5, como lo hab´ıamos afirmado; en s´ımbolos escribimos: (5 − x)2 + 21 = 52 De esto ya podemos concluir que el a´rea del cuadrado de lado (5 − x) es 4 y por lo tanto su lado es 2, de donde podemos concluir que x es 3. Y nuestra ecuaci´on est´a resuelta.

Ejercicios

1. Estudie el caso en que x sea mayor o igual que 5 (la figura 7 puede ser una ayuda).

2. Con base en este ejemplo, describa un procedimiento que permita resolver ecuaciones de la forma: x2 + c = bx

3. ¿Qu´e sucede con la ecuaci´on x2 + 26 = 10x?, ¿hay otros casos como este?

4. Proponga una soluci´on geom´etrica para ecuaciones de la forma ax2 = bx + c. 108

´ LAS OPERACIONES SUPERIORES DE LA MATEMATICA

x

x

x2

21

5 Figura 7

3.3.

Logaritmaci´ on

21

Si en la expresi´on ab = c conocemos la base a y la potencia c, el proceso para determinar el exponente b se llama logaritmaci´ on 22; se dice que b es el logaritmo en base a de c y lo escribimos: loga c = b 21

La logaritmaci´on tampoco es una operaci´ on entre n´ umeros naturales, por lo que en este contexto s´olo la interpretamos como una forma equivalente de escribir una potenciaci´ on. 22 En el a˜ no 1586 el suizo Jobst B¨ urgi (1552-1632), concibi´ o la idea del logaritmo, pero hasta el a˜ no 1620 public´ o sus tablas logar´ıtmicas bajo el t´ıtulo Arithmetische und geometrische Progress Tabulen. En 1614, John Napier (1550-1617) publica las primeras tablas de logaritmos, pero sin explicar su construcci´ on. En 1619, se publica su m´etodo con el t´ıtulo Mirifici logarithmorum canonis constructio

109

´ ´ ACTIVIDADES MATEMATICAS PARA EL DESARROLLO DE PROCESOS LOGICOS

De nuevo, es bueno resaltar que esto es s´olo otra forma de escribir ab = c; es decir, que log2 16 = 4 significa que 4 es el exponente que debemos ponerle a 2 para que la potencia sea 16. Otra vez, como en la potenciaci´on y en la radicaci´on tenemos problemas con el cero. Si c = 0 no existe un u ´nico b tal que ab = 0, ya que esta situaci´on s´olo se da si a = 0 para cualquier b = 0. Adem´as, si a = 0 no existe ning´ un b para c = 0 y para c = 0 todos los n´ umeros b satisfacen la relaci´on. Por lo tanto, no est´an definidos ni el logaritmo en cualquier base de cero ni el logaritmo en base cero. En cuanto al proceso para hallar logaritmos y sus relaciones con la potenciaci´on y la radicaci´on, conviene tener en cuenta las siguientes observaciones: 1. No debemos confundir la base de un logaritmo con la base de un sistema num´erico; cuando haya lugar a confusi´on debe tenerse cuidado. 2. Tenemos tres expresiones distintas para mirar una sola relaci´on fundamental, es decir: 34 = 81 significa que √ 4 81 = 3 y a su vez que log3 81 = 4 3. Si escogemos como base a cualquier n´ umero b entonces: b1 = b significa que 110

logb b = 1

´ LAS OPERACIONES SUPERIORES DE LA MATEMATICA

4. Para cualquier base b, el logaritmo de 1 es cero, porque: b0 = 1 significa que

logb 1 = 0

5. Los logaritmos m´as usados son los de base 10, que se llaman logaritmos vulgares o de Briggs 23 ; en ´estos el sub´ındice 10 se omite; es decir, que cuando no hay sub´ındice se sobreentiende que la base es 10, en ellos tenemos que: 100 = 1 101 = 10 102 = 100

3.3.1.

por lo tanto por lo tanto por lo tanto

log 1 = 0 log 10 = 1 log 100 = 2.

Propiedades de la logaritmaci´ on

Una propiedad de los logaritmos, muy usada por los antiguos24 para hacer c´alculos complicados, es: loga x · y = loga x + loga y Esto significa que a trav´es de los logaritmos podemos convertir multiplicaciones en sumas, lo que simplifica las cosas. Veamos el origen de esta propiedad, que de nuevo est´a relacionada con las de la potenciaci´on: Supongamos que x = ab Como ya sabemos, esto es lo mismo que decir b = loga x 23

Henry Briggs fue el primero que hizo las tablas de logaritmos en base 10, hacia 1631. Una primera idea de esta propiedad de los logaritmos surge de Arqu´ımedes, quien comparando las sucesiones aritm´eticas con las geom´etricas 24

1 2

2 4

3 8

4 16

5 32

6 64

7 128

8 256

9 512

hab´ıa observado que “para multiplicar entre s´ı dos n´ umeros cualesquiera de la sucesi´ on de abajo, debemos sumar los dos n´ umeros de la sucesi´on de arriba situados encima de aquellos dos. Luego debe buscarse en la misma sucesi´on de arriba dicha suma. El n´ umero de la sucesi´on inferior que le corresponda debajo ser´a el producto deseado”.

111

´ ´ ACTIVIDADES MATEMATICAS PARA EL DESARROLLO DE PROCESOS LOGICOS

An´alogamente y = ac es lo mismo que c = loga y Si multiplicamos las dos expresiones, obtenemos: x · y = ab+c Esto significa, en t´erminos de logaritmos, que b + c = loga x · y y si reemplazamos b y c por su valor, obtenemos: b + c = loga x + loga y O lo que es lo mismo: loga x · y = loga x + loga y Tambi´en con la ayuda de los logaritmos es posible convertir divisiones en restas25 y potenciaciones y radicaciones en multiplicaciones y divisiones respectivamente. Ejercicio 1. Verifique las relaciones 26 :  x = log a x − loga y loga y   loga xy = y · loga x

si y = 0

25

Antes del invento de los logaritmos exist´ıa un m´etodo para reemplazar la multiplicaci´ on de dos n´ umeros por una resta basado en la identidad a·b=

(a + b)2 (a − b)2 − 4 4

Se hac´ıa una tabla de los cuartos de los cuadrados de los n´ umeros (a + b) y (a − b) y restando se hallaba el producto. A mediados del siglo XIX el matem´atico franc´es, A.Cossart, edit´o una “tabla de los cuadrados de los cuadrados de los n´ umeros del 1 al 1.000 millones, con ayuda de la cual se halla el producto exacto de los n´ umeros por un m´etodo m´as simple y m´as c´omodo que los logaritmos”. 26 Estas propiedades fueron enunciadas expl´ıcitamente por primera vez, alrededor de 1650, por el inventor de la regla de c´ alculo, William Oughtred (1574-1660).

112

´ LAS OPERACIONES SUPERIORES DE LA MATEMATICA

De nuevo, recordamos que todas estas propiedades se cumplen cuando las operaciones est´an definidas; es decir, cuando existan n´ umeros naturales que b satisfagan la relaci´on x = a y los divisores en las divisiones sean distintos de 0. 2. Proponga un procedimiento para calcular logaritmos en un sistema num´erico de base cualquiera y calcule por ejemplo: log66 (6501 (7) ), log21(1212201 (4) ). Construya sus propios ejemplos. 3. Si queremos adivinar un n´ umero entre 1 y 10 formulando preguntas cuya respuesta s´olo sea s´ı o no, con el m´ınimo n´ umero de preguntas posible, podr´ıamos intentar encerrar el n´ umero de la siguiente forma: i) ¿El n´ umero es mayor o igual que 5? Si la respuesta es s´ı, preguntamos : ii) ¿El n´ umero es mayor o igual que 8? Y as´ı sucesivamente, hasta conseguir la respuesta: 4 preguntas son suficientes. ¿Cu´al es el m´ınimo n´ umero de preguntas posible, cuyas respuestas sean s´ı o no, para adivinar un n´ umero entre 1 y 100?, ¿entre 1 y 1000?, ¿entre 1 y 1000000? ¿Se le ocurre otro tipo de preguntas que nos lleve a una soluci´on? Los caminos que se pueden seguir para buscar una soluci´on, generalmente no son u ´nicos. Por ejemplo, ¿ser´ au ´til el sistema binario para resolver este problema? Si modificamos el intervalo en el cual consideramos los n´ umeros e intentamos adivinar un n´ umero entre 1 y 16, entre 1 y 53 , entre 24 y 75 . ¿Cu´al es el menor n´ umero de preguntas en cada caso? 4. Si en un torneo de f´ utbol participan 32 equipos, y el campeonato se desarrolla eliminando al que pierde un juego, el torneo durar´ıa 5 fechas si no hay empates. Si son 128 equipos, ¿cu´antas fechas son necesarias ? 113

´ ´ ACTIVIDADES MATEMATICAS PARA EL DESARROLLO DE PROCESOS LOGICOS

Si en el torneo participan 9 equipos, hay varias formas de jugarlo, con las mismas condiciones de eliminaci´on. Proponga algunas alternativas y determine en cada una de ellas el n´ umero de fechas necesarias. ¿De cu´ al forma obtenemos el m´ınimo n´ umero de fechas ? ¿Qu´e ocurre si en el torneo participan 350 equipos? 5. Utilizando solamente tres doses, las operaciones y los convenios usuales del ´algebra elemental (en la ra´ız cuadrada no se escribe el ´ındice), podemos escribir los otros n´ umeros, por ejemplo:  2 4= 2×2 8 = 22 × 2 umeros de 1 a 10. ¿Es posible la soluci´ on Escriba27 en forma similar los n´ con tres treses ?

3.4.

Una aplicaci´ on de la potenciaci´ on: la escritura de n´ umeros grandes

Como ya hab´ıamos visto en el cap´ıtulo anterior la potenciaci´on permite escribir n´ umeros grandes con pocos s´ımbolos; por ejemplo, tomemos 10 como base y expresemos sus primeras potencias en sistema decimal: umero de ojos que hay en un sal´on de clase en 1. 102 es aproximadamente el n´ un colegio distrital. umero de kil´ometros de Bogot´a a Barranquilla. 2. 103 es aproximadamente el n´ umero de cabellos que hay en la cabeza de una 3. 105 es aproximadamente el n´ adolescente y el n´ umero de habitantes de una ciudad como Fusagasuga, en umero de cabellos que tiene una persona 1998. Para la estimaci´on28 del n´ joven promedio, realizamos el siguiente procedimiento: 27

Despu´es de algunos intentos se dar´a cuenta que las cuatro operaciones b´ asicas de la aritm´etica no son suficientes para este prop´ osito; se requiere pensar en exponentes y logaritmos. 28 Usaremos libremente algunos conceptos que no justificamos por considerar que no son relevantes para nuestros prop´ ositos.

114

´ LAS OPERACIONES SUPERIORES DE LA MATEMATICA

i Contamos el n´ umero de cabellos que hay en un cent´ımetro cuadrado29 , lo que nos da aproximadamente 100 = 102 . ii Medimos los di´ametros de varias cabezas y los promediamos; el resultado es 20 cent´ımetros. iii Con este dato calculamos la superficie de una cabeza promedio (suponi´endola esf´erica para simplificar un poco); el resultado es: S = 4πr2 = 4π × 102

cm2

Como solamente la mitad de la cabeza (aproximadamente) est´a poblada de cabellos, este n´ umero debe dividirse por 2. iv Multiplicamos la medida de la superficie por el n´ umero de cabellos 2 4 en cada cm ; obtenemos 6, 28 × 10 . La potencia de 10 m´as cercana a este n´ umero es 105 , cantidad que tomaremos por simplicidad, pues no requerimos mayor precisi´on30. 4. 102 9 es aproximadamente el n´ umero de gotas de agua que hay en el mar. Para calcular el n´ umero de gotas que hay en todos los oc´eanos de la tierra procedemos de manera similar. i) Sabemos31 que el radio de la tierra es aproximadamente 6400km = 6, 4×108 cm; con ello calculamos el volumen de la tierra con la f´ormula: 4 V = πr3 3 Obtenemos V ∼ = 1027 cm3 . ii) La proporci´on de agua en el volumen del planeta Tierra es aproximadamente 3/4, luego debemos multiplicar por este factor, pero eso no influye mucho, puesto que el factor es casi 1. 29

Un dato m´ as aproximado es 165 cabellos por cent´ımetro cuadrado. TAHAN, M., El Hombre que calculaba, Ediciones universales, Bogot´ a, p. 225. 30 En El hombre que calculaba se da como superficie poblada de cabello 775 cm2 y 127. 875 cabellos en total. 31 En el a˜ no 300 A.C. Erat´ ostenes midi´o el radio de la tierra con la ayuda de una estaca; la descripci´on de su procedimiento y su resultado se encuentra en el libro SAGAN, C., Cosmos, p.15.

115

´ ´ ACTIVIDADES MATEMATICAS PARA EL DESARROLLO DE PROCESOS LOGICOS

iii) Si suponemos que en un cm3 cabe alrededor de 100 gotas, el n´ umero 29 m´aximo de gotas que puede haber en el mar es 10 . 5. El n´ umero de a´tomos del universo es 1085 , aproximadamente. El c´alculo del n´ umero de a´tomos del universo es similar y tiene en cuenta que hay alrededor de cien mil millones de galaxias y cada una de ellas tiene, en promedio, cien mil millones de estrellas. Nuestro Sol es una estrella promedio, aproximadamente igual a un mill´on trescientos mil veces la Tierra. Si conocemos la masa de la Tierra y sabemos el n´ umero de a´tomos que hay en una mol de sustancia (el n´ umero de Avogradro 6, 023 × 1023 ), podemos completar el c´alculo. 6. Una cuenta ligada al origen del ajedrez es la siguiente: Si pudi´eramos colocar en cada una de las casillas de un tablero de ajedrez granos de trigo, de esta manera: en el primer cuadro un grano, en el segundo dos granos, en el tercero cuatro, ocho en el siguiente y as´ı sucesivamente, hasta el cuadro sesenta y cuatro, el n´ umero de granos de trigo necesarios para llenar todos los escaques del tablero se escribe en base 10: 18 446 744 073 709 551 615 Para reunir esta cantidad, ser´ıa necesario sembrar de trigo los cinco continentes de la Tierra y recoger todo el producido de 76 cosechas32 . 7. Miremos ahora otra cuenta fant´astica referida en el a´lgebra recreativa de Perelman33. Tambi´en, con respecto a este noble juego, ¿cu´antas partidas de ajedrez se pueden jugar?: “Al mover la primera pieza, las blancas tienen 20 jugadas a elegir (16 con los peones y 2 con cada caballo), las negras tienen las mismas opciones para su primera jugada, en total 20 × 20 = 400 variantes para 1 jugada. 32

CARO, V., op. cit., p. 241 Tomado de KRAITCHIK, M., La Matem´ atica de los juegos y distracciones matem´ aticas en PERELMAN, Y., Algebra recreativa, Mir, 1989, p. 36-38 33

116

´ LAS OPERACIONES SUPERIORES DE LA MATEMATICA

En la segunda jugada las posibilidades aumentan, si ambas han movido P4R (pe´on cuatro rey) se tienen 29 jugadas para elegir, y as´ı van aumentando; supongamos que hay 20 variantes en promedio para las primeras 5 jugadas, 30 para cada una de las dem´as y una partida dura alrededor de 40 movimientos, por lo tanto, el n´ umero de partidas ser´a: (20 × 20)5 × (30 × 30)35 = 2010 × 3070 = 210 × 370 × 1080 ∼ = 103 × 370 × 1080 ∼ = 2 × 1083 × 1033 ∼ = 2 × 10116 puesto que 370 = 368 × 32 ∼ = (34 )17 × 10 ∼ = 8017 × 10 ∼ = 251 × 1018 ∼ = 2 × (210 )5 × 1018 ∼ = 2 × 1033 Si todos los humanos s´olo jugaran ajedrez, moviendo una pieza cada segundo, durar´ıamos m´as de 10100 siglos jugando sin repetir partida”. ¡Y nuestro universo tiene alrededor de 108 siglos de edad!

3.5.

Aplicaciones de bases diferentes a la base diez34

En el cap´ıtulo dos ya hab´ıamos hecho menci´on a algunas aplicaciones del sistema binario, octal, hexadecimal, etc.; sin embargo, existen aplicaciones de bases menos usuales, entre ellas: 34

MOSTER´IN, J., The Natural Numbers as a Universal Library en Philosophy of Mathematics Today. Editors AGASSI, E., DARVAS, G., Academic Publishers, 1997.

117

´ ´ ACTIVIDADES MATEMATICAS PARA EL DESARROLLO DE PROCESOS LOGICOS

1. La informaci´on gen´etica de todo individuo est´a codificada en ADN, al asignar a la adenina el 0, a la timina el 1, a la guanina el 2 y la citosina el 3, tendr´ıamos un sistema base 4; luego, la cadena de caracteres de ADN representa u ´nicamente un n´ umero natural; es decir, si tenemos por ejemplo la cadena TCCAGT, ´esta se encuentra representada por el n´ umero 133021 = 1 × 45 + 3 × 44 + 3 × 43 + 0 × 42 + 2 × 41 + 1 × 40 As´ı, el genoma completo de cualquier organismo puede representarse con un n´ umero natural. 2. Si consideramos las letras del abecedario, los signos de puntuaci´on y los ˜ O, P, Q, espacios en blanco: A, B, C, D, E, F, G, H, I, J, K, L, M, N, N, R, S, T, U, V, W, X, Y, Z,., ,, ;, :, -, ¡, !, ¨, ¿, ?, (, ), “, , ”, / obtenemos un sistema posicional cuya base es 43 y as´ı, a toda informaci´on, a todo libro hecho y por hacer en idioma espa˜ nol, le corresponde un n´ umero natural. 3. Todo sonido se puede codificar con diferentes niveles de amplitud seg´ un la quantizaci´on de sonidos wave (El sistema que se usa para digitalizar sonidos que se guardan en discos compactos CD). El d´ıgito amplitud 1 representa el n´ umero 0, el d´ıgito amplitud 2, el n´ umero 1. . . . hasta el d´ıgito amplitud 65536, el cual representa el n´ umero 65535. Estos n´ umeros forman un sistema de base 216 = 65536 que sirve para asignar a cada melod´ıa y a cada ruido un n´ umero natural. 4. 4. Tambi´en es posible codificar o mejor digitalizar im´agenes, pel´ıculas, etc., umero usando un sistema de numeraci´on cuya base es k ∗ d (donde k es el n´ disponible de colores y d es el n´ umero de grados de brillo para cada pixel). Los d´ıgitos del sistema son combinaciones de un color con un grado de brillo. El d´ıgito (color 1, brillo 1) representa el n´ umero 0, el d´ıgito (color 1, brillo 2), el n´ umero 1, y as´ı sucesivamente hasta el (color k, brillo d ) ,el cual representa el n´ umero k ∗ d − 1 . en general para cualquier n (1 ≤ n ≤ k) y cualquier m (1 ≤ m ≤ d) el d´ıgito (color n, brillo m) representa el n´ umero (n − 1)d + m − 1.

118

CAP´ITULO 4 ´ SISTEMAS NUMERICOS EN ALGUNAS CULTURAS Introducci´ on En este cap´ıtulo presentamos algunas soluciones que culturas antiguas notables le dieron al problema de contar y los sistemas de n´ umeros inventados por ellas. Una excelente presentaci´on de este tema se encuentra en el libro “Las cifras”de Georges Ifrah1. Iniciamos con un breve recuento de las formas m´as primitivas de conteo no posicional, incluyendo el sistema romano, para contrastarlas con lo desarrollado por nosotros en el cap´ıtulo 1. Luego estudiamos algunos sistemas posicionales, como el sistema num´erico de la cultura maya por tener una dificultad intr´ınseca de c´alculo debido a la forma como fue construido; el sistema num´erico de los incas que es un sistema posicional de base 10, donde se privilegia el uso de instrumentos como la yupana y el quipu, la primera similar a un a´baco y el segundo, un arreglo de cuerdas anudadas, para realizar operaciones b´asicas. Finalmente, veremos los sistemas babilonio y egipcio, donde se resaltan el uso de algoritmos para realizar las operaciones b´asicas. 1

IFRAH, G., Las Cifras: Historia de una gran invenci´ on, Alianza Editorial,1987.

119

´ ´ ACTIVIDADES MATEMATICAS PARA EL DESARROLLO DE PROCESOS LOGICOS

4.1.

Los primitivos

La idea de contar es una de las m´as primitivas2 en los seres humanos, bien sea con los dedos, como los indios de las islas de Borneo contando uno, dos, tres . . . , muchos; contar con a´bacos, con complejas construcciones, como el agrupamiento de grandes piedras, conocido como Stonehenge, en el sur de Inglaterra, o con los m´as modernos ordenadores. Como hemos visto, el proceso de contar consiste en establecer una correspondencia entre los objetos a contar y un conjunto de objetos o s´ımbolos que representan a los objetos iniciales, pero que son m´as f´aciles de manipular, como los dedos de una persona o los nudos en una cuerda o algunos signos dibujados en un trozo de piedra o de madera, poniendo uno o varios s´ımbolos por cada objeto a contar. Para registrar cada cantidad, el hombre invent´o palabras y signos que expresaban el conjunto de n´ umeros representados. Inicialmente, se desarrollaron sistemas de numeraci´on primitivos, como los que hicimos en el primer cap´ıtulo, que les permit´ıan dise˜ nar calendarios y efectuar algunos c´alculos simples, luego en culturas m´as avanzadas, se implementaron notaciones num´ericas m´as perfeccionadas, hasta conseguir los sistemas posicionales que construimos en el cap´ıtulo 2. En algunos pueblos, los n´ umeros recib´ıan nombres comunes; por ejemplo, los Tumanacos, una cultura sudamericana, usaban para 5 la misma palabra que usaban para decir “una mano entera”. El t´ermino que designaba al 6 significaba “uno de la otra mano”; el siete eran “dos de la otra mano”, y an´alogamente para 8 y 9 ; el 10 era “ambas manos”. Para expresar de 11 a 14, los tumanacos extend´ıan ambas manos y contaban “uno del pie, dos del pie...”, y as´ı sucesivamente, hasta el 15, que era “un pie completo”. El sistema continuaba expresando el 16 como “uno del otro pie”y as´ı hasta 19. La palabra que expresaba veinte era la misma empleada para decir “un indio”. El 21 era “uno de la mano del otro indio”. “Dos indios 2 significaba 40”, “tres indios”60, etc. 2

En el a˜ no 1937, Karl Absolom encontr´ o en una excavaci´ on en Checoslovaquia, un f´ osil de hueso de lobo de 30.000 a˜ nos de antig¨ uedad, sobre el cual se distinguen claramente marcas talladas, las cuales sugieren la existencia de secuencias de conteo. Se observaron sobre el hueso 55 marcas agrupadas de cinco en cinco, lo que lleva a pensar que la agrupaci´ on natural de los dedos sirvi´ o de base para su organizaci´ on.

120

´ SISTEMAS NUMERICOS EN ALGUNAS CULTURAS 3 ´ Algunos pueblos del Africa descubrieron, como muchos otros pueblos, que resulta sumamente dif´ıcil contar y calcular si se emplea una palabra o un s´ımbolo distinto para cada n´ umero. En lugar de inventar una nueva palabra para cada n´ umero, se forman t´erminos a partir de los que se designan los n´ umeros de base y se establecen relaciones aritm´eticas entre ellos. En los sistemas orales de numeraci´on africanos existen muchos ejemplos de este procedimiento.

´ Los bulanda de Africa Occidental, tienen un sistema cuyo principal n´ umero era seis, de modo que siete se dice seis m´as uno; ocho, seis m´as dos, etc. Entre los huku, de Uganda, se forman los t´erminos correspondientes a trece, catorce, quince, agregando uno, dos, tres, al n´ umero doce; as´ı, bakumba igimo (trece), significa 12 + 1. Un n´ umero peque˜ no, como es el caso del cinco, presenta la ventaja de facilitar el c´alculo oral o mental. Por ejemplo, 7+8 equivale en ese sistema a: (5+2)+(5+ 3). Como 2 + 3 = 5, se llega f´acilmente a encontrar como equivalente 5 + 5 + 5, o sea 10 + 5, o lo que es lo mismo a 3 veces 5. Por ejemplo, 2 × 7 equivale a (4 + 3) + (4 + 3), pero como 4 + 3 + 3 = 10, la respuesta puede formularse tambi´en como 10 + 4. Desde tiempos remotos existe ´ en Africa una larga tradici´on de c´alculo mental. En otras lenguas africanas, para formar los vocablos que designan los n´ umeros, no s´olo se utiliza la adici´on y la multiplicaci´on, sino tambi´en la sustracci´on. As´ı, los yoruba, de Nigeria, utilizan, para decir diecis´eis, la expresi´on eerin din logun, que significa “cuatro antes que veinte”, mientras que entre los luba - hemba, del Zaire, siete se dice habulwa mwanda (ocho menos uno), y nueve habulwa likumi (diez menos uno).

4.2.

El sistema de los romanos

El sistema introducido por los romanos hace unos 2.000 a˜ nos y probablemente 4 un en Europa hasta fines derivado del inventado por los etruscos , fue de uso com´ de la Edad Media. 3

´ ´ GERDES, P y CHERINDA M., OtextitContar en Africa, en: El correo de la Unesco (Par´ıs). vol 46, Noviembre de 1993. pp. 37 - 39. 4 El pueblo etrusco conoci´ o el alfabeto, procedente de la antigua lengua a´tica, orient´ andose su escritura de derecha a izquierda. No se conoce una literatura etrusca, limit´andose los textos encontrados a libros sagrados. Sin embargo, ten´ıan amplio conocimiento de las matem´aticas,

121

´ ´ ACTIVIDADES MATEMATICAS PARA EL DESARROLLO DE PROCESOS LOGICOS

Los romanos hicieron uso del alfabeto latino para los s´ımbolos de sus n´ umeros, como lo muestra la siguiente tabla: 1

5

10

50

100

500

1000

I

V

X

L

C

D

M

Los s´ımbolos son de dos tipos: Primarios: I uno, X diez, C cien, M mil Y secundarios: V cinco, L cincuenta, D quinientos Las reglas de escritura son: 1. Si el n´ umero est´a conformado por dos s´ımbolos diferentes, se debe tener en cuenta que si el n´ umero menor est´a a la derecha del mayor, entonces se adiciona a ´este y si est´a a la izquierda se le resta. Por ejemplo: VI 6 IV 4

CX 110 XC 90

LX 60 XL 40

2. Un s´ımbolo primario no debe repetirse m´as de tres veces en la escritura de una misma cantidad. Los s´ımbolos secundarios deben escribirse s´olo una vez. Por ejemplo: III 3, MCCC 1300, XXX 30, LXX 70, CCC 300, MMM 3000 3. Una raya escrita encima de un s´ımbolo, significa que ´este ha sido multiplicado por 1.000; dos rayas significa que ha sido multiplicado por 1.000.000; por lo tanto, para cantidades mayores que 3.999; es decir, MMMCMXCIX, se escriben s´ımbolos que representen n´ umeros menores que ´este y se le colocan las rayas necesarias. Por ejemplo: IV 4 IV 4,000 IV 4,000,000 siendo, al parecer, los creadores de las llamadas cifras romanas. Conoc´ıan la astronom´ıa, y estudiaron los fen´ omenos celestes. Med´ıan el tiempo en un a˜ no de trescientos cuarenta d´ıas, divididos en diez meses.

122

´ SISTEMAS NUMERICOS EN ALGUNAS CULTURAS

Ejercicio 1. Con siete palillos podemos formar con n´ umeros romanos la expresi´on indicada enseguida (dos palillos deben ser usados para formar el signo de igualdad).Mueva s´olo un palillo para que la ecuaci´ on se transforme en verdadera. VI=II 2. Idee procedimientos para efectuar operaciones de suma y resta de n´ umeros romanos. ¿Hay alguna manera de multiplicarlos y dividirlos?

4.3.

El sistema num´ erico de los mayas

La civilizaci´on maya5 (originaria de Guatemala) centr´o sus esfuerzos en el tiempo y su medida, tanto que crearon tres calendarios: uno era denominado Tzolkin (dioses) y era dedicado a sus celebraciones m´ısticas, constaba de 260 d´ıas; el segundo ten´ıa como funci´on medir el tiempo desde un remoto origen; el tercero (que es el m´as conocido y estudiado, ya que con base en ´el fue creado su sistema num´erico) era llamado haab o calendario solar, constaba de 18 meses de veinte d´ıas cada uno y un per´ıodo extra de cinco d´ıas y un cuarto de d´ıa. Fue una de las culturas m´as desarrolladas y notables de la Am´erica precolombina. Durante el primer milenio de la era cristiana los mayas tuvieron una idea precisa de los movimientos del Sol, de la Luna, de Venus y posiblemente tambi´en de los planetas Marte, Mercurio y J´ upiter, y ten´ıan adem´as la capacidad de predecir eclipses de Sol y de Luna. Su precisi´on en la medida del tiempo les permiti´o superar c´alculos realizados en Europa en la misma ´epoca. Por ejemplo, se dieron cuenta de que un a˜ no solar no tiene exactamente 365 d´ıas sino 365,242.000 d´ıas. Los c´alculos m´as recientes dan 365,242.198 d´ıas para el a˜ no solar verdadero; sin embargo, el a˜ no gregoriano que se usa en casi todos los pa´ıses actualmente tiene 365,242500 d´ıas, lo que constituye un error de 3,02 diezmil´esimas frente a un error de apenas 1,98 diezmil´esimas del a˜ no maya. Los mayas tambi´en fueron muy precisos en lo que respecta a la duraci´on media de una lunaci´on. Los c´alculos contempor´aneos dan un valor de 29,5305910 y 5

VON HAGEN, V.W., El mundo de los mayas, Editorial Diana, M´exico, 1960.

123

´ ´ ACTIVIDADES MATEMATICAS PARA EL DESARROLLO DE PROCESOS LOGICOS

los astr´onomos de la ciudad de Copan encontraron que 149 lunaciones equival´ıan a 4.400 d´ıas, lo que para una lunaci´on da la cantidad de 29,53020 d´ıas. Los astr´onomos de la ciudad de Palenque hicieron el mismo c´alculo sobre 81 lunaciones y encontraron un resultado m´as preciso todav´ıa: 2 392 d´ıas; es decir, 29,53086 d´ıas para una lunaci´on media. Los mayas no conoc´ıan el vidrio, ni desarrollaron telescopios, ni inventaron relojes que permitieran registrar per´ıodos de tiempos inferiores a un d´ıa y tampoco desarrollaron la noci´on de fracci´on. La unidad de tiempo m´as peque˜ na de estos astr´onomos era el d´ıa. Med´ıan el d´ıa solar (es decir, el lapso de tiempo que transcurre entre dos pasos consecutivos del sol por el meridiano del lugar, que sirve de observatorio), con un instrumento muy simple que era una especie de cuadrante solar muy rudimentario. Efectuaban las observaciones astron´omicas utilizando dos tiras de madera cruzadas sobre las que reposaba un largo tubo de jadeita que permit´ıa la visibilidad6 . En el campo de las matem´aticas descubrieron el principio de notaci´on posicional para los n´ umeros e inventaron el cero, aproximadamente cien a˜ nos antes de la invenci´on del sistema ar´abigo. Los manuscritos mayas, especialmente el c´odice Dresde (un tratado de astronom´ıa y de adivinaci´on que hab´ıa sido copiado en el siglo IX de nuestra era, de un original redactado tres o cuatro siglos antes) revelan que entre los sacerdotes mayas exist´ıa un sistema de base 20, con un cero, donde el valor de la cifras estaba determinado por su posici´on en la escritura de los n´ umeros. que designaban respectivamente a los Los s´ımbolos principales fueron • y n´ umeros 1 y 5; ellos se combinaban para formar otros hasta el 19, por ejemplo: 1 •

5

7 ••

10

6

13 •••

HAMMOND, N., Unearthing the Oldest Know Maya en National Geographic, vol. 162, No 1, July, 1982.

124

´ SISTEMAS NUMERICOS EN ALGUNAS CULTURAS

Escrib´ıan los n´ umeros de abajo hacia arriba. El cero lo representaban con una concha de caracol marino, que aqu´ı representaremos con el s´ımbolo @. Las unidades de cada orden van aumentando como potencias de veinte7, excepto las unidades de tercer orden que corresponden a 18 de segundo orden; esto motivado no es de 18 meses y no de 20. en que en su calendario solar8 , un a˜ Para escribir un n´ umero compuesto por dos o´rdenes, se colocaba la cifra de las unidades en el nivel inferior y la cifra de las veintenas en el segundo. As´ı, 21 se escribe: • •

1 × 20 1

y 79 que es igual a 3 × 20 + 19, se escribe: ••• • • ••

3 × 20 19

El tercer piso indicaba los m´ ultiplos de 360, por ejemplo: 12 × 360

•• ••• • • ••

3 × 20 19

7

Los mayas contaban con los dedos de las manos y de los pies. En la lengua quich´e el n´ umero 20 significa: toda la persona 8 En su calendario se ten´ıan las siguientes correspondencias: 20 kines

= 1 uinal, o 20 d´ıas

18 20 20 20

= 1 tun, o 360 d´ıas = 1 kat´ un, o 7,200 d´ıas = 1 bakt´ un, o 144,000 d´ıas. = 1 pict´ un, o 2,880,000 d´ıas.

uinales tunes katunes baktunes

20 pictunes = 1 calabt´ un, o 57,600,000 d´ıas. 20 calabtunes = 1 kinchilt´ un, o 1,152,000,000 d´ıas 20 kinchiltunes = 1 alaut´ un, o 23,040,000,000 d´ıas.

125

´ ´ ACTIVIDADES MATEMATICAS PARA EL DESARROLLO DE PROCESOS LOGICOS

Corresponde a: 4399 = 12 × 360 + 3 × 20 + 19 El n´ umero 13 495 se escrib´ıa: • •• •••

puesto que 13495 = 1 × 7200 + 17 × 360 + 8 × 20 + 15 Y para escribir 115 212, utilizamos el cero dos veces: • @ @ •• debido a que 115212 = 16 × 7200 + 0 × 360 + 0 × 20 + 12. El descubrimiento de la notaci´on posicional no estuvo en la cultura occidental hasta que en la Edad Media los dieron a conocer los a´rabes, quienes a su vez lo hab´ıan tomado de la cultura hind´ u. El sistema de numeraci´on maya, que hemos descrito, tiene ventajas te´oricas frente a otros sistemas m´as conocidos, como el sistema de los n´ umeros romanos; por ejemplo, para escribir los n´ umeros del 1 al 19, inclusive, en la notaci´on romana son necesarios tres s´ımbolos, o sea las letras I, V y X, y hacer dos operaciones aritm´eticas, adici´on y sustracci´on. En el sistema maya se necesitan solamente dos s´ımbolos, la barra y el punto, y basta una sola operaci´on aritm´etica: la adici´on. Los mayas inventaron otro sistema de numeraci´on llamado “de cabezas”, donde se dispon´ıa de 20 s´ımbolos individuales para los n´ umeros del 0 al 19. Este sistema se forma con una sucesi´on b´asica de 14 jerogl´ıficos o glifos con figuras de cabezas humanas, diferenciadas entre s´ı por sus rasgos espec´ıficos; las 14 cabezas correspond´ıan a 14 deidades patronas de cada n´ umero, del 0 al 13. Los 126

´ SISTEMAS NUMERICOS EN ALGUNAS CULTURAS

6 s´ımbolos faltantes se forman colocando una parte representativa de la cabeza del 10 (el maxilar inferior, pues este s´ımbolo era una calavera), debajo de las cabezas del 4 al 9, para tener as´ı los glifos del 14 al 19 y completar las 20 cifras necesarias. Ejercicios 1. Escriba un algoritmo para pasar del sistema maya al sistema decimal y viceversa. 2. Intente definir operaciones b´asicas, como suma, resta multiplicaci´on y divisi´on en el sistema num´erico maya, ¿ qu´e dificultad encuentra?

4.4.

El sistema de numeraci´ on inca

Lo conocido de la cultura inca, se lo debemos a los datos recogidos por el cronista inca-espa˜ nol, Felipe Guam´an Poma de Ayala, en su libro “Nueva cor´ onica y buen gobierno”. Los incas9 ten´ıan un sistema social organizado desde los puric (peones), hasta el hono-curaca (jefe principal del Ayllu), pasando por los cancha-camayo y los pachaca-curaca. Por cada diez subalternos hab´ıa un superior inmediato, luego del hono-curaca estaba el gobernador de provincia quien hac´ıa de mandatario de cuartel; el imperio se divid´ıa en cuatro cuarteles, la posici´on m´as alta la ocupaba el sapa-inca (emperador). Pose´ıan un sistema de pesas y balanzas de gran exactitud y en su calendario los meses ten´ıan una duraci´on de 27 + 1/3 d´ıas y un a˜ no ten´ıa 328 noches. Entre los cancha-camayos, exist´ıa una clase de trabajadores quienes eran los encargados de hacer los registros y las cuentas; tales registros se hac´ıan en quipus10. 9

WASSEN, H., El antiguo a ´baco peruano seg´ un el manuscrito de Guaman Poma, G¨ otemburgo, 1940. BURNS, W., La tabla de C´ alculo de los incas, Bolet´ın de Lima. COSSIU, F., El mundo de los incas, Editorial Fondo de Cultura Econ´ omica, M´exico, 1969. PAREJA, D., Instrumentos prehisp´ anicos de c´ alculo: el quipu y la yupana, Instituto de Investigaciones y posgrados, Universidad del Quind´ıo, Armenia, 1986. PAREJA, D., Arithmetical Algorithms of the Incas, Universidad del Quind´ıo. VON HAGEN W., Los Incas, Editor Joaqu´ın Mortiz, Colecci´ on culturas b´ asicas del mundo, M´exico, 1964. 10 Quipu en quechua significa nudo.

127

´ ´ ACTIVIDADES MATEMATICAS PARA EL DESARROLLO DE PROCESOS LOGICOS

4.4.1.

La representaci´ on de los n´ umeros en el quipu

El quipu consta de una cuerda delgada, que pod´ıa tener un metro de longitud, de la cual penden otros cordones con distintas longitudes y colores; en uno de ellos hay varios nudos, que pueden ser simples, dobles o triples. El quipu se utilizaba para registrar datos importantes, como la cantidad de producci´on en las cosechas, el n´ umero de habitantes, etc., y s´olo pod´ıa ser le´ıdo por personas especializadas, denominados kipucamayocs. En la figura 2, se muestran c´omo se escrib´ıan los n´ umeros 1, 2, 9 y 8624 en el sistema de numeraci´on incaico; las unidades se representaban en el extremo de las cuerdas y se ascend´ıa de acuerdo a si los nudos simbolizaban decenas, centenas, etc.

El quipu permit´ıa en forma f´acil efectuar la suma de cantidades previamente anotadas en cuerdas contiguas, como se muestra en la figura 3. Se suman las unidades de los tres n´ umeros 4 + 3 + 7 = 14, se anuda en la cuerda de la izquierda, que representa la suma, el 4 en el nivel de las unidades y se lleva 1 para el nivel de las decenas. Se suman las decenas 1 + 3 + 2 + 6 = 12, se anuda el dos en el nivel de las decenas y llevamos una centena. Finalmente, se suman las centenas; el resultado se puede leer en la primera cuerda de izquierda a derecha. Al terminar la operaci´on, se ajustaban los nudos de la parte superior y las cuatro cuerdas se ataban en el extremo opuesto. Con esto se indicaba que en el 128

´ SISTEMAS NUMERICOS EN ALGUNAS CULTURAS

quipu se hab´ıa efectuado una adici´on.

4.4.2.

La Yupana, el “´ abaco precolombino”

La primera representaci´on conocida de la yupana11 apareci´o en 1615 en la obra “Nueva cor´ onica y buen gobierno”, la tabla all´ı expuesta es de forma rectangular constituida por cinco filas y cuatro columnas, en cada una de las casillas se encuentran c´ırculos negros y blancos distribuidos por columnas, en la primera se encuentran cinco, en la segunda tres, en la tercera dos y en la u ´ltima un c´ırculo, como se muestra enseguida:

11

El nombre yupana proviene del vocablo quechua yupay, que significa contar. Actualmente se conoce como tabla de c´alculo de los incas.

129

´ ´ ACTIVIDADES MATEMATICAS PARA EL DESARROLLO DE PROCESOS LOGICOS

◦◦ ◦◦ ◦ •• •• • •• •• • •• •• • •• •• •

◦◦ ◦

◦ ◦



•• ◦

◦ •



•• •

• •



•• •

• •



•• •

• •



Figura 4

A ra´ız de este esquema surgieron diferentes interpretaciones con respecto al significado de los c´ırculos negros y los c´ırculos blancos; algunos personajes afirman que los c´ırculos blancos indican posiciones desocupadas y los negros representan n´ umeros, los cuales eran simbolizados por los incas con granos de ma´ız, piedrecillas, semillas, etc. Otros, por el contrario, aseguran que los c´ırculos negros indican posiciones para sumar y los blancos para restar; sin embargo, de esta u ´ltima acepci´on no se conoce desarrollo alguno.

Por otra parte, personas que se han interesado por estudiar las matem´aticas desarrolladas por la cultura inca, como William Burns Glynn, proponen que en los c´ırculos se ubican granos, piedras, etc. de tal manera que cada c´ırculo tiene un valor de uno, pero adquieren valor diferente de acuerdo a la fila donde se encuentra ubicado; adem´as, la casilla donde s´olo hay un c´ırculo se utiliza como “memoria”. Enseguida mostraremos c´omo representar n´ umeros en la yupana y c´omo sumar de acuerdo a la propuesta realizada por Burns:

La yupana se coloca en posici´on horizontal, como se muestra en la figura: 130

´ SISTEMAS NUMERICOS EN ALGUNAS CULTURAS











◦◦

◦◦

◦◦

◦◦

◦◦

◦ ◦◦

◦ ◦◦

◦ ◦◦

◦ ◦◦

◦ ◦◦

◦◦ ◦◦ ◦◦ ◦◦ ◦◦ ◦◦◦ ◦◦◦ ◦◦◦ ◦◦◦ ◦◦◦ Figura 5 Cada c´ırculo tiene un valor de acuerdo a la columna donde se encuentra; es decir, si ´este pertenece a la segunda columna (de derecha a izquierda), cada c´ırculo tendr´a un valor de 10, si pertenece a la tercera, tendr´a un valor de 102 , y as´ı sucesivamente. Esto, debido a que el sistema de numeraci´on usado por los incas es decimal. Los c´ırculos de la primera fila (de arriba hacia abajo) se utilizan como memoria y las otras filas son posiciones para ubicar granos, piedrecillas, etc. Cada vez que se completen los diez c´ırculos de una columna, los barremos o desocupamos y colocamos uno en la memoria que luego ser´a trasladado a la columna posterior, de la siguiente forma: Decenas Unidades





◦◦

◦◦

◦ ◦◦

◦ ◦◦

Unidades

Unidades





◦◦ =

◦◦ ◦◦ ◦◦• ◦◦◦

◦ ◦◦ ◦◦ ◦◦◦

•• =

• •• •• •••

Figura 6 Un procedimiento para sumar en la yupana, con las reglas establecidas, es el siguiente: Sumemos, por ejemplo: 437 y 253. Inicialmente colocamos uno de los sumandos en la yupana y el segundo en la parte superior de ´esta, as´ı: 131

´ ´ ACTIVIDADES MATEMATICAS PARA EL DESARROLLO DE PROCESOS LOGICOS

••

253

437

➪ ➪

• •••• • • •











◦◦

◦◦

◦◦

◦◦

◦◦

◦ ◦◦

◦ ◦◦

◦ ◦◦

◦ ◦◦

◦ ••

◦◦ ◦◦ •◦ ◦◦ •• ◦◦◦ ◦◦◦ ••• ••• ••• Figura 7

Colocamos las piedrecillas o canicas de la parte superior sobre la yupana, conservando las columnas; es decir, en la columna uno, transferimos las tres piedrecillas a la columna de las unidades, y as´ı sucesivamente. Como en la primera columna quedan los diez c´ırculos llenos y una canica por fuera, barremos y llevamos uno a la memoria; as´ı, podemos ubicar la piedrecilla que sobraba en la yupana. De manera similar, realizamos las dem´as sumas en cada columna. Luego, la suma ser´a igual a 690, que se representa de la siguiente manera:

690













◦◦

◦◦

◦◦

◦•

◦◦

◦ ◦◦

◦ ◦◦

◦ ◦•

• ••

◦ ◦◦

◦◦ ◦◦ •• •• ◦◦ ◦◦◦ ◦◦◦ ••• ••• ◦◦◦ Figura 8

Hay propuestas12 para usar la yupana como herramienta de ayuda en las aulas escolares, pero el tema a´ un est´a en exploraci´on. Ejercicio 12

MORA, L., VALERO, N., LUQUE, C., La yupana como herramienta pedag´ ogica, Memorias del X Encuentro de Geometr´ıa y sus aplicaciones, Bogot´a, Junio de 1999, p. 155.

132

´ SISTEMAS NUMERICOS EN ALGUNAS CULTURAS

Construya un modelo de quipu y uno de yupana. Proponga procedimientos para efectuar adiciones, sustracciones, multiplicaciones y divisiones.

4.5.

El sistema num´ erico de Babilonia

Los antiguos Sumerios13 construyeron una aritm´etica para elaborar un calendario 5700 a˜ nos antes de Cristo; ellos mismos desarrollaron un sistema num´erico con 60 s´ımbolos que luego fue heredado y utilizado con mucha habilidad por los babilonios. Los babilonios habitaban en la antigua Mesopotamia, regi´on que hoy corresponde a Irak. A fines del siglo XIX se encontraron en las ruinas de Mesopotamia, unas 400 tablillas de arcilla y fragmentos de otras con textos matem´aticos que han sido copiadas, transcritas y traducidas. Estas tablillas datan principalmente de dos per´ıodos: unas, de alrededor del a˜ no 2000 a.C. y la mayor´ıa pertenecen a un periodo entre el a˜ no 600 a. C. y el a˜ no 300 d.C. Las del primer per´ıodo son las m´as importantes en lo que se refiere a la historia de la matem´atica. La lengua y la escritura utilizadas en las tablillas de per´ıodo m´as antiguo es el acadio, que se superpuso al tipo de lenguaje y escritura sumerios; las palabras de la lengua acadia consist´ıan en una o m´as s´ılabas y cada s´ılaba ven´ıa representada por un grupo de signos que se reduc´ıan esencialmente a peque˜ nos segmentos rectil´ıneos. Los acadios utilizaban para escribir un prisma de secci´on triangular, que apoyaban sobre la tablilla en una posici´on inclinada, produciendo as´ı unas se˜ nales en forma de ‘cu˜ na’orientadas en distintas direcciones, por ello su escritura se llam´o cuneiforme. La aritm´etica alcanz´o su m´as alto grado de desarrollo en la civilizaci´on babil´onica durante el per´ıodo acadio. Estas tablillas de arcilla no s´olo las utilizaron los sumerios, caldeos y babilonios, sino tambi´en los hititas, asirios y otros pueblos de la antig¨ uedad. Como en otras culturas, fueron los astr´onomos babilonios quienes desarrollaron su sistema de numeraci´on. Los 59 primeros n´ umeros se representaban en una forma decimal. 13

CHURCHILL, E. M., Contando y midiendo, Uteha, 1965, p. 30.

133

´ ´ ACTIVIDADES MATEMATICAS PARA EL DESARROLLO DE PROCESOS LOGICOS

La cu˜ na vertical representaba una unidad, la cu˜ na horizontal diez unidades. Los dem´as n´ umeros se formaban escribiendo estos s´ımbolos en diferentes combinaciones.

Alrededor del a˜ no 1700 a.C. se dieron cuenta que sus s´ımbolos pod´ıan representar otros valores dependiendo de su posici´on, dando origen a la notaci´ on posicional; escrib´ıan los numerales en grupos separados por espacios; su escritura se hac´ıa de derecha a izquierda; cada grupo representaba, respectivamente, unidades, grupos de 60 unidades, grupos de 60 × 60 = 3600 unidades14 , etc. Para hallar el valor representado en un n´ umero, se efectuaban las multiplicaciones correspondientes y se sumaban los productos resultantes. Aunque en los m´as antiguos textos babilonios no se muestra la presencia de un s´ımbolo espec´ıfico para el cero, empleaban un espacio en blanco, m´as o menos destacado, para indicar la ausencia de unidades de posici´on dada, pero evidentemente esto pod´ıa ser mal interpretado y resultar confuso; luego, durante el periodo seleucida inventaron una doble cu˜ na para representarlo. Incluso en este periodo no se utiliz´o ning´ un s´ımbolo para indicar una o m´as posiciones vac´ıas por el extremo derecho del n´ umero, como en nuestra notaci´on. Es interesante observar la manera c´omo se obtiene alguna informaci´on de tablillas antiguas en una labor de adivinaci´on por ensayo y error; presentamos un ejemplo tomado del libro Matem´aticas, episodios hist´ oricos 15, de Asger Aaboe. En la figura 10 observamos la reproducci´on del haz (anverso) y del env´es (reverso) de una tablilla de la Antigua Babilonia; en ambas caras la escritura 14

En la actualidad se sigue empleando el sistema sexagesimal para expresar medidas de tiempo (horas, minutos, segundos) y de a´ngulos (grados, minutos y segundos). 15 AABOE, A., Matem´ aticas Episodios Hist´ oricos, Norma, p. 21

134

´ SISTEMAS NUMERICOS EN ALGUNAS CULTURAS

consiste en signos simples colocados en dos columnas, se˜ nalados por Col. I y Col. II. En total hay 24 renglones, pero de momento ignoraremos el u´ltimo.

En la columna I del primer rengl´on, la primera anotaci´on es una cu˜ na vertical; la segunda, dos cu˜ nas verticales; la tercera es tres, y as´ı sucesivamente. Es natural interpretar estos 9 renglones como 1, 2, 3... 7, 8, 9 porque este es el n´ umero de cu˜ nas en cada rengl´on. En cada fila, los trazos se agrupan en ternas, con lo que se facilita la lectura; despu´es del 9 aparece un s´ımbolo nuevo: una cu˜ na angular. Si admitimos que representa 10, entonces no hay dificultad en los ocho renglones siguientes ya que constan de esta cu˜ na y los signos del 1 al 8; podemos, en consecuencia, leerlos como 11, 12, 13... 18. En la l´ınea siguiente encontramos16 un signo especial para el 19, pero corrientemente se representa por una cu˜ na angular y nueve cu˜ nas verticales. En los siguientes renglones vamos encontrando, dos, tres, cuatro, cinco cu˜ nas angulares que deben representar el 20, 30, 40 y 50 respectivamente. En la columna II, leemos en los seis primeros renglones 9, 18, 27, 36, 45, 54; lanzando una hip´otesis podemos decir que esta columna es una tabla de multiplicar por 9, entonces el 7 rengl´on debe expresar el 63 y el octavo el 72, 16

AABOE, A., op.cit. p.22.

135

´ ´ ACTIVIDADES MATEMATICAS PARA EL DESARROLLO DE PROCESOS LOGICOS

pero en estos renglones encontramos una cu˜ na vertical seguida de un 3 en el s´eptimo, y de un 12 en el octavo. Nos vemos obligados a concluir que la cu˜ na vertical debe representar el 60; si transcribimos esas dos l´ıneas como 1,3 y 1,12 y le damos al primer 1 el significado de 60, obtendremos 1, 3 = 1 × 60 + 3 = 63

1, 12 = 1 × 60 + 12 = 72

y

Los siguientes renglones se pueden transcribir como: 1, 21 = 81 1, 30 = 90 1, 39 = 99 1, 48 = 108 1, 57 = 117 Todo, bas´andonos en la hip´otesis ya establecida, de que la segunda columna es una tabla de multiplicar por 9. El d´ecimo rengl´on tiene dos cu˜ nas verticales y un 6, lo podemos transcribir como 2,6: 2, 6 = 126 que debe corresponder a 14 × 9 = 126. As´ı, podemos interpretar los siguientes renglones como: 2, 15 = 135 2, 24 = 144 2, 33 = 153 2, 42 = 162 2, 51 = 171 En el siguiente rengl´on hay 3 cu˜ nas verticales cuyo significado es 180; si estuviera seguido por alg´ un signo pudi´eramos identificar el cero, y transcribirlo de la forma 3,0 que ser´ıa 3 × 60 + 0 = 180. En consecuencia, daremos por sentado que los babilonios no ten´ıan s´ımbolo para el cero en el extremo derecho de un n´ umero, sino que procuraban darle a entender al lector que se ha dejado un espacio vac´ıo. 136

´ SISTEMAS NUMERICOS EN ALGUNAS CULTURAS

Dos renglones m´as abajo podemos comprobar esta afirmaci´on; frente al 40 de la columna I aparece un 6 en la columna II, que transcribiremos como 6,0, esto es, 6 × 60 + 0 = 360, que concuerda con 40 × 9. Las otras dos l´ıneas, tambi´en confirman nuestro supuesto: 4, 30 = 4 × 60 + 30 = 270 = 9 × 30 7, 30 = 7 × 60 + 30 = 450 = 9 × 50 Hemos comprobado que el texto tiene un significado si aceptamos que los signos num´ericos alteran su valor de acuerdo al lugar que ocupan, de tal manera que si un signo se desplaza un espacio hacia su izquierda, su valor num´erico se multiplica por 60. Si se analizan otros textos, nuestro supuesto quedar´a ampliamente confirmado. Por ejemplo, un n´ umero transcrito como 1, 25, 30 corresponde a: 1 × 602 + 25 × 60 + 30 = 5130 Pero no todo es tan claro; el n´ umero anterior pudo ser trascrito como 1,25,30,0 o como 1,25,30,0,0, pero este no es un defecto muy grande ya que en general el contexto aclara a cual n´ umero se refiere17.

4.5.1.

Las operaciones aritm´ eticas

Como en el sistema babilonio los s´ımbolos para el uno y para el diez eran los s´ımbolos b´asicos; los n´ umeros del 1 al 59 se constru´ıan combinando estos s´ımbolos, de manera que las operaciones de restar y sumar se reduc´ıan a a˜ nadir o quitar s´ımbolos. Para representar la suma los babilonios reun´ıan las dos expresiones en una sola. Las multiplicaciones las efectuaban usando tablas, “los babilonios fueron de los m´as infatigables compiladores de tablas aritm´eticas de la historia”18 ; construyeron tablas de multiplicar (similares a la tabla del nueve ense˜ nada atr´as), en donde se encontraban los m´ ultiplos de p (siendo ´este el n´ umero principal 19 de la tabla de multiplicar) as´ı: 17

AABOE, A., op.cit., p. 24. BELL, E. T., Historia de las matem´ aticas, Fondo de Cultura Econ´ omica, 1995, p. 40. 19 T´ermino usado por AABOE, A., op.cit. p. 37. 18

137

´ ´ ACTIVIDADES MATEMATICAS PARA EL DESARROLLO DE PROCESOS LOGICOS

1 2 3 · · · 19 20 30 40 50

p 2p 3p · · · 19p 20p 30p 40p 50p

que culminan, por lo general, en p2 . Con base en tablas de este estilo es posible encontrar cualquier m´ ultiplo de p; por ejemplo, si dese´aramos conocer 32p a la manera de los babilonios, har´ıamos la adici´on entre 30p y 2p, valores dados en la tabla. No se tiene conocimiento de tabla de multiplicar para p = 17, mientras que s´ı existe una tabla para p = 44, 26, 40. Todo indica que los n´ umeros principales de las tablas de multiplicar hallados, son los n´ umeros presentes en otras tablas, denominadas tablas modelos de rec´ıprocos; una muestra de ellas es la siguiente, que escrita en sistema sexagesimal es: Col. I

Col. II

Col. I

Col. II

Col. I

Col. II

2

30

16

3,45

45

1,20

3

20

18

3,20

48

1,15

4

15

20

3

50

1,12

5

12

24

2,30

54

1,6,40

6

10

25

2,24

1

1

8

7,30

27

2,13,20

1, 4

56,15

9

6,40

30

2

1, 12

50

10

6

32

1,52,30

1, 15

48

12

5

36

1,40

1, 20

45

15

4

40

1,30

1, 21

44,26,40

138

´ SISTEMAS NUMERICOS EN ALGUNAS CULTURAS

Al multiplicar los n´ umeros de la columna I con los de la columna II se obtienen siempre potencias de 60, esto es, 1, 0 o 1, 0, 0 o . . .; pero los babilonios expresaban cualquier potencia de sesenta con un 1; de esta manera, en la columna II est´an umeros escritos en la columna I; as´ı, los n´ umeros de la los rec´ıprocos 20 de los n´ columna II no son interpretados como n´ umeros naturales, sino como cocientes entre un n´ umero natural dado y una potencia de 60. Por ejemplo, en la s´eptima fila de la columna I del primer recuadro de la tablilla de rec´ıprocos aparece el n´ umero 6,40, esto es: 6, 40 = 6 × 60 + 40 = 400 el cual, al ser multiplicado por 9 (su correspondiente en la columna I) da como producto 602 ; as´ı: 6, 40 × 9 = 602 De donde se obtiene: 602 9 6, 40 1 = 602 9

6, 40 =

que es igual a: 6 × 60 + 40 40 6 1 + 2= = 2 60 60 60 9 ´ltimo Luego, el n´ umero 6 de 6, 40 representa 6/60 y el n´ umero 40, 40/602 . As´ı, el u n´ umero presente en la columna II: 44, 26, 40 rec´ıproco de 81 (1,21), significa: 26 40 44 + + 602 603 604 Esta tabla de rec´ıprocos junto con las tablas de los n´ umeros principales eran utilizadas por los babilonios para realizar divisiones, teniendo en cuenta el hecho de que: 1 a = a× b b 20

Se denomina rec´ıproco de un n´ umero a, a aquel n´ umero que al ser multiplicado por a, se obtiene 1 como producto.

139

´ ´ ACTIVIDADES MATEMATICAS PARA EL DESARROLLO DE PROCESOS LOGICOS

1 1 1 En algunas tablillas se dan s´olo valores aproximados para , , . . . , etc., 7 11 13 porque estas fracciones tienen expresiones sexagesimales infinitas peri´odicas. Cuando aparec´ıan, en los problemas m´as antiguos, fracciones con denominadores que inclu´ıan factores primos distintos de 2, 3 o 5, entonces el mismo factor aparec´ıa tambi´en en el numerador y se cancelaba uno con otro. Tambi´en hicieron tablas de cuadrados, ra´ıces cuadradas, cubos y ra´ıces cuadradas exactas para los cuadrados perfectos y aproximadas para los dem´as. Una aprox√ imaci´on babilonia es 2 = 1, 414213 . . . y las seis primeras cifras correctas son 1, 414214 . . . Ejercicio Escriba 1340 y 345 en base 60 y efect´ ue 1340 ¸ 345 siguiendo el procedimiento babilonio.

4.6.

El sistema egipcio

La informaci´on que tenemos de la matem´atica Egipcia proviene fundamentalmente de dos papiros: el papiro de Mosc´ u y el papiro Rhind, que se conoce tambi´en como papiro de Ahmes por el nombre de su autor. El papiro de Rhind tiene 30 cm de alto y 6 de largo y se encuentra actualmente en el British Museum, excepto unos pocos fragmentos que se conservan en el Broklyn Museum. Fue comprado en 1858 en una ciudad del Nilo por el escoc´es Henry Rhind, del cual se deriva su nombre. Este papiro no est´a escrito en forma jerogl´ıfica sino con tinta y en cursiva; est´a expuesto el sistema de numeraci´on y algunas operaciones, entre ellas se encuentra una forma sumativa de multiplicar. Los papiros contienen problemas y soluciones: 85 en el papiro Rhind y 25 en el papiro Mosc´ u. Los egipcios utilizaron dos sistemas de escritura: uno pict´orico jerogl´ıfico grabado en madera o en piedra, con dibujos de objetos o animales que significaban de alguna manera la idea del n´ umero que se quer´ıa representar21 y el otro hier´atico, de forma cursiva, m´as apropiado para la escritura sobre los papiros. Estos s´ımbolos fueron creados alrededor del a˜ no 3000 a.C. 21

MESERVE, B., SOBEL M., Introducci´ on a las matem´ aticas, Editorial Revert´e, 1967.

140

´ SISTEMAS NUMERICOS EN ALGUNAS CULTURAS

El sistema jerogl´ıfico de numeraci´on es decimal y dispone de un s´ımbolo particular para cada potencia de 10, se rige por un principio de adici´on; es decir, se adiciona el valor de los diferentes s´ımbolos que componen la escritura de un n´ umero, sin que el orden sea importante. Usaban s´ımbolos distintos para cada potencia de diez22, pero no consideraron el valor posicional; por lo tanto, la posici´on de los s´ımbolos no afectaba el n´ umero que se quer´ıa representar. | ∩ y ∩ | son formas diferentes de escribir once, o sea, diferentes nombres o s´ımbolos para el mismo n´ umero; la primera notaci´on es la m´as frecuente en los jerogl´ıficos y los papiros, donde se aprecian una serie de anotaciones escritas a manera de contabilidad comercial23. Desde el 1 hasta el 9 trazaban l´ıneas verticales; el diez ten´ıa un s´ımbolo parecido a una herradura o cuenco boca a bajo; cien, una espiral o rollo de pergamino; mil, estaba representado por una flor de loto; diez mil, por un dedo apuntando, y un mill´on, por el dibujo de un hombre con las manos extendidas en actitud de asombro, como se muestra en la figura 11:

Cada s´ımbolo se pod´ıa repetir hasta 9 veces, para formar otros n´ umeros, por ejemplo: |

||

|||

|||||||||

1

2

3

9

∩∩

∩∩ ||||

∩∩∩

24

50 22

BALDOR, A., Aritm´etica, Cultural Colombiana, 1972, p. 36. MESERVE, B.E., SOBEL, M., Introducci´ on a las Matem´ aticas, Editorial Reverte Mexicana S.A, M´exico, 1971. 23

141

´ ´ ACTIVIDADES MATEMATICAS PARA EL DESARROLLO DE PROCESOS LOGICOS

4.6.1.

Las Operaciones

La aritm´etica egipcia fue esencialmente aditiva; para las sumas y las restas usuales se limitaban a combinar o a cancelar los diferentes s´ımbolos hasta llegar al resultado concreto. La multiplicaci´on y la divisi´on tambi´en se reduc´ıan a procesos aditivos, pero el c´alculo era un poco m´as complicado. La multiplicaci´on se hac´ıa inicialmente mediante un proceso de duplicaci´ on, basado en el hecho de que cualquier n´ umero puede expresarse como una suma de potencias de dos. Por ejemplo, como 19 = 1 + 2 + 16, entonces el producto de 19 × 25 se determina duplicando sucesivamente el n´ umero 25, as´ı : 1 × 25 = 25 2 × 25 = 50 4 × 25 = 100 8 × 25 = 200 16 × 25 = 400 Luego se suman los m´ ultiplos de 25 que correspondan a 1, 2, 16: 19 × 25 = (1 + 2 + 16) × 25 = 25 + 50 + 400 = 475 Con el tiempo, adoptaron un procedimiento m´as r´apido para multiplicar, conocido como el m´etodo de duplicaci´ on y mediaci´ on, que consiste en duplicar uno de los factores y sacar la mitad del otro. Por ejemplo para determinar el producto 19 × 25 se va sacando mitad a 19 sucesivamente, sin tener en cuenta los residuos de cada paso, y al mismo tiempo se va multiplicando el 25, como se expone a continuaci´on : 19 9 4 2 1

➜ ➜ ➜ ➜ ➜

25 50 100 200 400

El proceso se termina cuando se obtiene un 1 en la columna de los n´ umeros que se han ido dividiendo entre dos. A cada uno de los n´ umeros de esta columna 142

´ SISTEMAS NUMERICOS EN ALGUNAS CULTURAS

le corresponde un n´ umero en la columna de los n´ umeros que se han duplicado. El producto de 19 × 25 se obtiene como la suma de los n´ umeros que se oponen a los n´ umeros impares de la columna de las mitades: 19 × 25 = 25 + 50 + 400 = 475 Ejercicio Explique, ¿por qu´e funciona el m´etodo descrito? Para dividir un n´ umero por otro. Por ejemplo, para dividir 19 por 8 proced´ıan de la siguiente manera: 1 2 1/2 1/4 1/8

8 16 4 2 1

y obten´ıan como respuesta: 2 + 1/4 + 1/8 La idea consiste en tomar el n´ umero de ochos y de partes de ochos que sumen 19. Los egipcios utilizaron la matem´atica en la administraci´on de los templos y asuntos de estado, en el c´alculo de salarios pagados a los trabajadores, en el c´alculo de vol´ umenes de los graneros y a´reas de los campos, en el cobro de impuestos estimados seg´ un el ´area de tierra, en el c´alculo de n´ umero de ladrillos para la construcci´on de edificios, etc. Los papiros contienen problemas relativos a la cantidad de granos necesarios para producir cantidades de cerveza o la cantidad de granos necesarios para obtener un grano de otra calidad cuya proporci´on relativa a la de la primera fuera conocida. Calcularon la duraci´on del a˜ no solar observando la estrella Sirio; el momento en que era visible justo antes de la salida del sol recib´ıa el nombre de salida heliacal de Sirio, y el intervalo entre dos de ellas consecutivas era de aproximadamente 365 1/4 d´ıas. Adoptaron un calendario civil con un a˜ no de 365 d´ıas. La concentraci´on en Sirio se debi´o, al parecer, al hecho de que las aguas del Nilo 143

´ ´ ACTIVIDADES MATEMATICAS PARA EL DESARROLLO DE PROCESOS LOGICOS

comenzaban a subir aproximadamente ese d´ıa, que se eligi´o como primer d´ıa del a˜ no. El a˜ no de 365 d´ıas se dividi´o en 12 meses de 30 d´ıas m´as 5 d´ıas extras al final. Como los egipcios no intercalaron el d´ıa adicional cada 4 a˜ nos, el calendario civil iba retras´andose poco a poco con respecto a las estaciones y al cabo de 1460 a˜ nos volv´ıa a la situaci´on inicial; a este intervalo se le llama ciclo s´otico, del nombre egipcio para Sirio. Su calendario fue adoptado por Julio C´esar en el a˜ no 45 a.C, pero transformado en el a˜ no 365 1/4 d´ıas por consejo del griego alejandrino Sos´ıgenes.

144

CAP´ITULO 5 ´ LOS NUMEROS NATURALES EN OTRAS RAMAS DE LA ´ MATEMATICA

Introducci´ on: En este cap´ıtulo ponemos en juego procesos l´ogicos que los matem´aticos practican a menudo, que no son dif´ıciles por s´ı mismos, pero que requieren entrenamiento y trabajo. El 90 por ciento de la actividad matem´atica es sudor del pensamiento; mucho empe˜ no, mucho error, pero siempre, muchas ideas para volver a empezar. Plantearemos algunos problemas que consideramos interesantes y divertidos, lo que es fundamental para el enamoramiento requerido por esta forma peculiar de razonar. Esbozaremos algunas posibles soluciones que sirvan como ayudas, lo que no implica que sean el camino. No estamos interesados en una respuesta correcta para cada ejercicio, sino en que el estudiante ejercite sus propios procesos de pensamiento y proponga alternativas de soluci´on. 145

´ ´ ACTIVIDADES MATEMATICAS PARA EL DESARROLLO DE PROCESOS LOGICOS

5.1.

Fen´ omenos peri´ odicos relacionados

Los siguientes 4 problemas tienen algo en com´ un; intente encontrar las coincidencias. 1. Dos sat´elites X y W tienen ´orbitas circulares alrededor de la tierra, X da una vuelta en 90 minutos y W en 120. Si X y W son representados por los puntos de la figura 1, y giran en el mismo sentido, ¿cu´antos minutos pasar´ an para que juntos est´en en la posici´on de partida por primera vez?, ¿por segunda vez?, ¿por s´eptima vez? Si cambiamos el per´ıodo (tiempo que gasta en dar una vuelta) de X a 45 minutos y el de W a 30, ¿el problema se resuelve de la misma forma? ¿Es v´alido el esquema de soluci´on para cualquier valor de X y W? Este problema, como casi todos, tiene varias maneras para ser resuelto. Proponga por lo menos 3 formas, intentando que cada una de ellas sea, en alg´ un sentido, mejor que la anterior. ¿Podr´ıa resolverse utilizando el n´ umero de vueltas, en lugar del per´ıodo?

X

W



Figura 1

Si dos sat´elites se encuentran por primera vez al cabo de 60 minutos, ¿cu´ ales son los posibles periodos de cada uno?; ¿y si se encuentran al cabo de un n´ umero cualquiera k de minutos? 2. ¿Cu´al es la menor capacidad que debe tener un estanque que se puede llenar en un n´ umero exacto de minutos por cualquiera de tres grifos que vierten, el 146

´ ´ LOS NUMEROS NATURALES EN OTRAS RAMAS DE LA MATEMATICA

primero 36 litros por minuto, el segundo 54 litros por minuto y el tercero 60 litros por minuto? ¿Y si cambiamos los valores del primero por un n´ umero k de litros por minuto, el segundo por un n´ umero m y el tercero por un n´ umero n?

3. Tres aviones salen de una misma ciudad, el primero cada 8 d´ıas, el segundo cada 10 d´ıas y el tercero cada 20. Si salen juntos del aeropuerto el d´ıa 2 de enero, ¿cu´ales ser´ an las dos fechas m´ as pr´ oximas en que volver´ an a salir juntos? (el a˜ no no es bisiesto).

4. Si queremos encontrar un m´ ultiplo com´ un entre dos n´ umeros naturales, una opci´on elemental es multiplicarlos; si el problema consiste en encontrar un m´ ultiplo com´ un pero cada vez menor, podemos dividir el resultado anterior entre alg´ un factor com´ un de los n´ umeros, pues ´este ha sido multiplicado dos veces en el producto, si lo que debemos es encontrar el m´ınimo com´ un m´ ultiplo de ellos podemos repetir el proceso anterior con cada uno de los factores comunes de los dos n´ umeros. Con base en estas observaciones, escriba un algoritmo para hallar el m´ınimo com´ un m´ ultiplo de dos n´ umeros cualesquiera. Enuncie una relaci´ on entre el m´ınimo com´ un m´ ultiplo y la multiplicaci´on de dos n´ umeros cualesquiera.

5. Hallar el m´ınimo com´ un m´ ultiplo entre dos n´ umeros naturales a y b, que notaremos [a, b], es una operaci´on similar a la multiplicaci´on, en el sentido de que si damos dos n´ umeros cualesquiera su m´ınimo com´ un m´ ultiplo es otro n´ umero natural. Podemos, entonces, construir una tabla para esta nueva operaci´on; por ejemplo, la tabla 1 muestra el m.c.m de algunos n´ umeros escritos en base 12: 147

´ ´ ACTIVIDADES MATEMATICAS PARA EL DESARROLLO DE PROCESOS LOGICOS

BASE 12 [, ]

0

1

2

3

4

5

6

7

8

9

A

B

0

0

0

0

0

0

0

0

0

0

0

0

0

1

0

1

2

3

4

5

6

7

8

9

A

B

2

0

2

2

6

4

A

6

12

8

9

18

B

3

0

3

6

3

10

4

0

4

5

0

5

6

0

6

7

0

7

8

0

8

9

0

9

A

0

A

B

0

B

a) Complete la tabla. b)

Explique las dos primeras filas y las dos primeras columnas.

c) ¿En cu´ales casos es lo mismo multiplicar dos n´ umeros que hallar su m´ınimo com´ un m´ ultiplo? d ) ¿La operaci´on es conmutativa?, ¿tiene elemento id´entico? e) Enuncie otras regularidades. 6. Si a y b son dos n´ umeros naturales, la relaci´on entre la multiplicaci´on a × b y el m´ınimo com´ un m´ ultiplo de ellos, la llamamos el m´aximo com´ un divisor de a y b y lo notamos (a, b), tenemos que (a, b) =

a×b [a, b]

y resulta otra operaci´on entre n´ umeros naturales que tambi´en tiene su tabla correspondiente: 148

´ ´ LOS NUMEROS NATURALES EN OTRAS RAMAS DE LA MATEMATICA

BASE 12 (,)

0

0

1

2

3

4

5

6

7

8

9

A

B

1

2

3

4

5

6

7

8

9

A

B

1

1

1

1

1

1

1

1

1

1

1

1

1

2

2

1

2

1

2

1

2

1

2

1

2

1

3

3

1

1

3

1

1

3

1

1

3

1

1

4

4

1

2

5

5

1

1

6

6

1

2

7

7

1

1

8

8

1

2

9

9

1

1

A

A

1

2

B

B

1

1

a) Sugiera una explicaci´on para el nombre de esta operaci´ on. b) Complete la tabla. c) Explique las dos primeras filas y las dos primeras columnas. d ) ¿La operaci´on es conmutativa?, ¿tiene elemento id´entico? e) Enuncie otras regularidades. f ) Multiplique las tablas 1 y 2 casilla a casilla, ¿qu´e obtiene?; ¿lo esperaba?

5.2.

Los datos y las hip´ otesis en un problema

Frecuentemente encontramos en matem´aticas problemas donde, al parecer, faltan datos o las hip´otesis que se deben manejar no est´an expl´ıcitas; en estos casos se requiere de algo de paciencia y observaci´on. En el siguiente ejercicio es muy importante la lectura y traducci´on de las condiciones del problema y la formulaci´on de conjeturas o hip´otesis bajo las cuales la soluci´on propuesta es v´alida: 149

´ ´ ACTIVIDADES MATEMATICAS PARA EL DESARROLLO DE PROCESOS LOGICOS

Un club de 99 miembros hizo una rifa con tres premios. En ella cada ganador ten´ıa como premio un n´ umero de entradas al teatro, igual a la suma de los d´ıgitos de su boleta. Uno de los miembros, que no asisti´o, pregunt´o qui´enes hab´ıan ganado y no le quisieron decir; luego pregunt´o por el n´ umero total de entradas repartidas y le dijeron que si le respond´ıan, le dir´ıan los n´ umeros premiados, y ´el contest´o: “ya s´e”. ¿C´ omo lo supo?

5.3.

Algunas veces no se puede

En algunas ocasiones encontramos problemas que tienen una soluci´on para algunos valores de los datos del problema, pero que en general no se pueden resolver con datos arbitrarios. Veamos un ejemplo: Con las siguientes seis cifras 1, 1, 2, 2, 3 y 3 es posible escribir un n´ umero donde los “1”est´en separados por un d´ıgito, los “2”por dos d´ıgitos y los “3”por 3. Por ejemplo: 3 1 2 1 3 2 y por supuesto, escritos en orden inverso 2 3 1 2 1 3 S´olo con las cifras 1, 1, 2 y 2 no es posible escribir un n´ umero donde los “1”est´en separados por un d´ıgito y los “2”por dos d´ıgitos, pues en cualquier configuraci´on nos falta una cifra. Con las cifras 1, 1, 2, 2, 3, 3, 4 y 4 escriba un n´ umero donde los “1”est´en separados por un d´ıgito, los “2”por dos d´ıgitos, los “3”por 3 y los “4”por 4. El mismo problema con los n´ umeros 1, 1, 2, 2, 3, 3, 4, 4, 5 y 5 no tiene soluci´on, tampoco si los n´ umeros van hasta 6, pero s´ı la tiene para 7 y 8. Una soluci´on para 7 es: 46171435623725 Encuentre una para 8. ¿Para cu´ales valores de k, el problema de escribir 112233445566 . . . , kk con las condiciones expuestas es posible? 150

´ ´ LOS NUMEROS NATURALES EN OTRAS RAMAS DE LA MATEMATICA

❆ ❆ ❆ ❆ ❆ ❆ ❆ ❆ ❆ ❆ ❆ ❆ ❆ ❆ ❆ ❆ ❆ ❆ Hay problemas en matem´aticas que no tienen soluci´on; no por incapacidad del matem´atico, sino porque la soluci´on no existe, generalmente porque las condiciones lo impiden. ❆ ❆ ❆ ❆ ❆ ❆ ❆ ❆ ❆ ❆ ❆ ❆ ❆ ❆ ❆ ❆ ❆ ❆ Por ejemplo, es imposible dividir un a´ngulo arbitrario en tres a´ngulos iguales utilizando solamente regla y comp´as, construir un cubo cuyo volumen sea el doble de un cubo dado o construir un cuadrado de a´rea equivalente a un c´ırculo dado1 ; tambi´en es imposible encontrar una f´ormula que permita resolver cualquier ecuaci´on de grado 5 o superior, utilizando las operaciones b´asicas del ´algebra2 . Demostrar que un problema no tiene soluci´on suele ser m´as complicado que encontrar una; tal vez sea Evariste Galois el matem´atico m´as recordado por ´ demostr´o la imposibilidad haber generado soluciones a problemas de este tipo. El de soluci´on de algunos problemas milenarios, como la duplicaci´on del cubo, la trisecci´on del a´ngulo y la ecuaci´on general de quinto grado por radicales. Generalmente, en la b´ usqueda de soluciones para problemas insolubles, se aprende mucho sobre otros problemas que s´ı tienen soluci´on y se desarrollan teor´ıas nuevas. Galois no s´olo resolvi´o los problemas cl´asicos, sino que puso las bases para el desarrollo de la teor´ıa de grupos y de la teor´ıa de Galois3 .

5.4.

Regularidades y secuencias

5.4.1.

Cuadrados m´ agicos4

Un cuadrado m´agico es una tabla con el mismo n´ umero de casillas verticales (columnas) que horizontales (filas), donde se ubican n´ umeros, de manera que la suma de las filas es la misma que la de las diagonales y la de las columnas. 1

˜ MUNOZ, J. M., Imposible duplicar el cubo. ¿Sabe usted por qu´e?, VII Coloquio Distrital de Matem´aticas y Estad´ıstica, Universidad Nacional, Bogot´ a, Dic. 1990. 2 ´ HERSTEIN, I. N., Algebra moderna, Trillas, M´exico, 1976, p.249. 3 ´ FRALEIGH, J. B.; Algebra abstracta, Addison Wesley Iberoamericana, M´exico,1988, p. 415. 4 Los cuadrados m´ agicos son de origen oriental.

151

´ ´ ACTIVIDADES MATEMATICAS PARA EL DESARROLLO DE PROCESOS LOGICOS

Coloque5 en cada casilla vac´ıa un n´ umero entre 1 y 9, de manera que la suma de los n´ umeros en cada fila, columna o diagonal, sea 15 en base 10. Repita el ejercicio en base 11 y en base 16.

5 9 ¿Cu´antas maneras hay de hacerlo?,¿la suma debe ser 15 o puede ser otro n´ umero?, ¿puede ser cualquier n´ umero?, ¿se puede con multiplicaciones?, ¿con m´ınimo com´ un m´ ultiplo? umeros entre 1 y 16 en un cuadrado 4× 4, Escriba6 una tabla similar con los n´ de manera que su suma sea 34. Repita el ejercicio en base 11 y en base 16. Busque regularidades, explore posibilidades.

5.4.2.

Observando secuencias

1. Estudie las siguientes secuencias de n´ umeros en base 10: 1

16 17 8 9 4 5 2 3 1 3 5 6

18 19 10 11 6 7 6 7 5 7

8 4 2 1 20 12 12 10 9

9 5 3 3 21 13 13 11 11

2 1 4 5 2 3 1 3 10 11 6 7 6 7 5 7 22 23 14 15 14 15 14 15 13 15

3 3 6 6 5 12 12 10 9 24 24 20 18 17

7 7 7 13 13 11 11 25 25 21 19 19

14 14 14 13 26 26 22 22 21

15 15 15 15 27 27 23 23 23

28 28 28 26 25

29 29 29 27 27

30 30 30 30 29

Los chinos resolvieron este ejercicio hace m´as de 6000 a˜ nos. El pintor Alberto Durero present´ o este cuadrado en uno de sus cuadros, en 1514.

152

31 31 31 31 31

´ ´ LOS NUMEROS NATURALES EN OTRAS RAMAS DE LA MATEMATICA

D´e una explicaci´on de su formaci´ on. Puede ayudarle, pensando por ejemplo, en: a) La secuencia de los primeros n´ umeros de cada tabla. b) Si hay relaci´on entre el primero y el u ´ltimo n´ umero de cada tabla. c) La forma como se rompe la secuencia en cada fila y el n´ umero de veces que esto ocurre. ¿Cu´antos n´ umeros aparecen en una sola fila en cada tabla?, ¿cu´ antos n´ umeros aparecen s´ olo en dos filas en cada tabla?, ¿en tres?, ¿en cuatro?...¿encuentra alguna regularidad? Justifique. Sin mirar las tablas, diga cu´ al n´ umero est´a solamente en la segunda, tercera y cuarta filas de la quinta tabla. Construya un ejemplo an´ alogo partiendo de 32 (los n´ umeros est´ an escritos en base 10). ¿Pueden hacerse ordenamientos similares en otras bases? 2. Observe la siguiente secuencia: 28 26 24 22 20

22 21 23 21 22

18 16 14 12 10

21 24 21 22 21

8 6 4 2

23 21 22 21

Encuentre la mayor potencia de 2 que divide a 28! = 1 × 2 × 3 × 4 × 5 × · · · × 25 × 26 × 27 × 28 Cu´al es la mayor potencia de 2 que divide a n! = 1 × 2 × 3 × 4 × 5 × · · · × (n − 2) × (n − 1) × n a) Si n es par b) Si n es impar Repita el ejercicio con la mayor potencia de 3 que divida a n! 153

´ ´ ACTIVIDADES MATEMATICAS PARA EL DESARROLLO DE PROCESOS LOGICOS

3. Considere la suma7 7 + 77 + 777 + · · · + 77777777777777777777 Encuentre el d´ıgito del lugar 100 de la suma. 4. La siguiente secuencia se construye as´ı: La primera l´ınea tiene un 1; eso escribo en la segunda: 1 1; la segunda l´ınea tiene dos unos; eso escribo en la tercera: 2 1; la tercera l´ınea tiene un 2 y un 1; eso escribo en la cuarta; la cuarta l´ınea tiene un 1, un dos y dos unos; eso escribo en la quinta, etc. 1

1 3

1 1 1

1 2 2 1 2

1 1 1 2 2

1 2 1

1 1

¿C´ omo es la l´ınea 12?;¿cu´al es la l´ınea 25?; ¿cu´al es la l´ınea n´ umero k?

5.5.

Sumando sumas

1. El menor n´ umero de paquetes que se pueden colocar en dos sacos, de tal manera que en cada saco quede un n´ umero diferente de paquetes y que cada uno de ellos contenga por lo menos uno, es tres. ¿Cu´antos en tres sacos?, ¿cu´antos en cuatro sacos?, ¿cu´ antos en 7 sacos?, ¿cu´antos en k sacos? Invirtiendo la pregunta, el mayor n´ umero de sacos en los que se pueden repartir 28 paquetes de manera que cada uno tenga por lo menos uno, pero cada uno con un n´ umero distinto de paquetes es 7. ¿Cu´antos sacos con 55 paquetes? 7

Mathematics Teacher. Diciembre 1992 Vol. 85. No 9.

154

´ ´ LOS NUMEROS NATURALES EN OTRAS RAMAS DE LA MATEMATICA

Y si tenemos 25 paquetes, podemos distribuirlos con las condiciones establecidas en m´aximo 6 sacos, ubicando, por ejemplo, 1, 2, 3, 4, 5 y 10 paquetes en cada uno de los sacos. Por supuesto, esta no es la u´nica soluci´on posible. ¿De cu´antas formas distintas se puede hacer esta repartici´ on? Si el n´ umero de sacos es 17, ¿cu´al es el n´ umero m´ınimo de paquetes? Si el n´ umero de paquetes es 123, ¿cu´al es el mayor n´ umero de sacos y de cu´ antas formas diferentes es posible hacerlo? ¿En qu´e casos la soluci´ on es u ´nica? Ahora, si el n´ umero de sacos es 23, ¿cu´ al es el n´ umero mayor de paquetes? Si el n´ umero de paquetes es 157, ¿cu´al es el mayor n´ umero de sacos y de cu´antas formas diferentes es posible hacerlo? Hay casos en que la soluci´on es u ´nica, y sucede cuando el n´ umero de paquetes es igual a la suma de los sacos; ejemplo: 1 + 2 + 3 + 4 = 10 Como el total de los paquetes es 10, el n´ umero de sacos es 4 exactamente. En este proceso no hay otras formas distintas de soluci´on; por eso, ella es u ´nica. 2. Un libro est´a paginado desde la primera p´agina en base 7. ¿Cu´anto suman las cifras que numeran las 6 , 16 , 26 , 36, 46, 56, 66, primeras p´ aginas del libro?. ¿Cu´anto suman las cifras que numeran las 15 , 20, 4 3 primeras p´aginas? Plantee y resuelva un ejercicio similar en base 8. Ensaye una generalizaci´ on para un n´ umero de p´ aginas con tres cifras. ¿Cu´antas p´ aginas tiene un libro si la suma de los d´ıgitos que las numeran (en base 10) suma 381? ¿C´ omo ser´ıa en base 9? 3. Si a un n´ umero en base 2 se agrega un 1 a la derecha, ¿qu´e relaci´ on tiene con el original?, ¿y si se agrega a la izquierda?, ¿qu´e pasa en base 3, 4, · · · , k?, ¿y si en lugar de 1 se le agrega un 2?, ¿y si se le agrega la base? Intente generalizaciones. 155

´ ´ ACTIVIDADES MATEMATICAS PARA EL DESARROLLO DE PROCESOS LOGICOS

5.6.

¿Cu´ al es el n´ umero m´ as grande?

1. Escribir n´ umeros grandes en distintas bases. Queremos estudiar ahora, cu´al es el n´ umero m´as grande que se puede escribir con tres cifras iguales con los convenios establecidos para la escritura de los n´ umeros, pero sin signos de operaciones entre ellos, salvo lo acordado para la notaci´on exponencial. En base 2 s´olo tenemos dos s´ımbolos, 0 y 1. El n´ umero m´as grande que se puede escribir con tres cifras repetidas y sin colocar signos de operaciones entre ellos es, naturalmente, 1 1 1, puesto que:  1 1 1(1 ) = 11 = (1)11 < (11)1 < 111 ¿Es v´alido este ordenamiento con tres unos, en cualquier base? En base tres, la secuencia con tres doses es:  2 2 2(2 ) = 22 < 222 < (22)2 < (2)22 ¿La secuencia es la misma en cualquier base? ¿Cu´al es el orden con tres treses en base 4? 3 3(3 )

 3 3 3

(3)33

(33)3

¿Es el mismo en otra base? Con tres cuatros en base 5, la secuencia es:   4 444 < (44)4 < 44 4 < (4)44 < 4(4 ) puesto que 4 4(4 ) > 444

156

´ ´ LOS NUMEROS NATURALES EN OTRAS RAMAS DE LA MATEMATICA

En base 10 el mayor n´ umero que se puede escribir con tres doses, sin ning´ un 22 signo de operaci´on, es 2 , puesto que:  2 2 2 = 16 ≤ 222 ≤ 222 = 484 ≤ 222 = 4194304 Ahora, con tres treses el mayor n´ umero que podemos escribir es 333, debido a que: 3 33 ≤ 333 = 5, 559060566556 × 1015 9

El mayor n´ umero con 3 nueves es 99 = 9387420489 (m´as grande que el n´ umero de ´atomos de todo el universo). Intentemos una generalizaci´on para el caso de tres cifras iguales a b en base k. Los dos candidatos a ser el m´as grande, seg´ un hemos visto, son: b

b (b )

y

bbb

Como la base es la misma basta comparar los exponentes8 esto es bb

y

kb + b,

Para que el primero sea mayor, es necesario que b(b−1) > (k + 1), donde hemos dividido por b ambas expresiones (suponiendo naturalmente que b no es 0). Si b es 4 la expresi´on es cierta hasta base 62, donde b(b−1) es 43 = 12(64 en base 10) y k + 1 = 11(63 en base 10). La f´ormula considerada en base 10 es cierta para b > 3. 8

Esto se debe a que si x < y entonces ax < ay .

157

´ ´ ACTIVIDADES MATEMATICAS PARA EL DESARROLLO DE PROCESOS LOGICOS

Ensaye un trabajo similar con cuatro cifras iguales; he aqu´ı unas perlitas:

El n´ umero m´as grande que se puede escribir con cuatro unos es: 1111 = 285311670611 Con cuatro doses el m´as grande es: 

(2)2

22

Este n´ umero es mayor que 24000000, que a su vez es mayor que 101200000 y este u ´ltimo que tiene m´as de un mill´on de cifras9. 2. Escriba de menor a mayor 2121

5.7.

955

788

Contar en geometr´ıa

1. Este problema requiere para su soluci´on mezclar un poco de intuici´on en el manejo del espacio, con cierta habilidad de c´alculo. Inicialmente, notemos que con 3 puntos se puede formar una sola fila (de manera que los tres est´en en la misma recta): *

*

*

Si ponemos 4 puntos, con la condici´on adicional de no poner m´as de tres puntos en cada fila, podemos hacer, de nuevo, s´olo 1 fila de 3 puntos. *

* *

*

Pero con 5 puntos, ya podemos hacer, a lo m´as, dos filas de 3: 9

´ PERELMAN, Y., Algebra Recreativa, Mir, ;Mosc´ u, 1989, p. 49.

158

´ ´ LOS NUMEROS NATURALES EN OTRAS RAMAS DE LA MATEMATICA

*

* *

*

*

Se puede verificar, que con 6 puntos se hacen m´aximo 4 filas de 3 7 puntos se hacen m´aximo 6 filas de 3 8 puntos se hacen m´aximo 7 filas de 3 9 puntos se hacen m´aximo 9 filas de 3 10 puntos se hacen m´aximo 11 filas de 3. Ejemplificamos este u ´ltimo caso en la figura 2: 



 







 



Figura 2

Con 20, 21, 22, puntos, ¿cu´ al es el mayor n´ umero de filas de tres puntos que se puede construir?¿Cu´ al es la secuencia para k puntos? 2. La conjetura del punto. Consideremos un conjunto de puntos que est´an conectados todos entre s´ı, por l´ıneas rectas, como se muestra en la figura 3: 159

´ ´ ACTIVIDADES MATEMATICAS PARA EL DESARROLLO DE PROCESOS LOGICOS 









 









Figura 3 La conjetura sostiene que es imposible trazar un diagrama de puntos en el que cada l´ınea contenga al menos tres puntos (sin tener en cuenta el diagrama en el que todos los puntos descansen sobre la misma l´ınea). En efecto, tras experimentar con unos pocos diagramas, la afirmaci´on parece ser cierta. Por ejemplo, el primer diagrama consta de cinco puntos conectados por seis l´ıneas. Cuatro de ellas no contienen tres puntos, as´ı que, claramente, esta disposici´on no cumple el requisito de que todas las l´ıneas posean tres puntos. Si insertamos un punto m´as y su l´ınea correspondiente, como en el segundo diagrama, el n´ umero de l´ıneas que no re´ une tres puntos se reduce a tres. Distribuya el diagrama de manera que todas las l´ıneas contengan al menos tres puntos. ¿Es imposible? Argumente por qu´e. 3. Observemos las siguiente figura:

1

5 Figura 4

14

En la primera aparece solamente un cuadrado de lado 1. En la segunda hay 4 cuadrados de lado 1 y 1 de lado 2 En la tercera hay 9 cuadrados de lado 1, 4 de lado 2 y 1 de lado 3. De esto inducimos que, para un cuadrado de lado 4 deber´ıa haber 16 cuadrados de lado 1, 9 de lado 2, 4 de lado 3 y 1 de lado 4. 160

´ ´ LOS NUMEROS NATURALES EN OTRAS RAMAS DE LA MATEMATICA

Y en un tablero de ajedrez deber´ıa haber: 12 + 22 + 32 + 42 + 52 + 62 + 72 + 82 = 204

cuadrados en total.

Existe, adem´as, solamente una figura regular que nos permite hacer figuras semejantes a partir de ella por repetici´on, como en el caso anterior; es el tri´angulo equil´atero10. Estudie el problema de encontrar el n´ umero de tri´ angulos equil´ ateros que se forman en un tri´angulo equil´atero de lado 1, 2, 3, · · · , 10, k. En la figura 5 se muestra el caso para un tri´angulo equil´atero de lado 4:

Figura 5

5.8.

Contar en topolog´ıa

1. La caracter´ıstica de Euler.Considere un tri´angulo ABC en un plano α y un punto P fuera del plano. Si unimos con segmentos de l´ınea recta el punto P con cada uno de los v´ertices del tri´angulo obtenemos un poliedro de cuatro caras, llamado tetraedro, que tiene cuatro v´ertices y 6 aristas. Note que el n´ umero de v´ertices V m´as el n´ umero de caras C, menos el n´ umero de aristas A es 2. Este n´ umero es llamado la caracter´ıstica de Euler del tetraedro. Sustituya el tri´ angulo por un pol´ıgono de 4, 5, 6 . . . lados en α (con esto obtenemos pir´ amides), cuente sus caras, sus v´ertices y sus aristas. Calcule la caracter´ıstica de Euler de las pir´ amides; es decir, el n´ umero V + C - A. Sustituya ahora el punto P por un pol´ıgono A’B’C’. . . en un plano β, paralelo a β, del mismo n´ umero de lados que el que se toma en el plano α; una los v´ertices 10

La justificaci´ on de este comentario aparece en el cap´ıtulo 6.

161

´ ´ ACTIVIDADES MATEMATICAS PARA EL DESARROLLO DE PROCESOS LOGICOS

correspondientes hasta formar un prisma. Repita las cuentas hechas para las pir´ amides y compare los resultados. Intente con otras figuras. En otros poliedros, como las torres, que est´an formadas por pir´amides sobre prismas, tambi´en se verifica la relaci´on de Euler11. ¿Existe alguna figura donde ella no se verifique? Calcule la caracter´ıstica de Euler para un cubo con un agujero, como el que muestra la figura 6. Haga sus propios ensayos con s´ olidos que tengan dos o m´as huecos.

Figura 5 Una consecuencia asombrosa de que la caracter´ıstica de Euler sea 2 para cualquier poliedro, es que: ❆ ❆ ❆ ❆ ❆ ❆ ❆ ❆ ❆ ❆ ❆ ❆ ❆ ❆ ❆ ❆ ❆ ❆ S´olo existen 5 poliedros regulares ❆ ❆ ❆ ❆ ❆ ❆ ❆ ❆ ❆ ❆ ❆ ❆ ❆ ❆ ❆ ❆ ❆ ❆ Un poliedro se llama regular si su superficie est´a formada por caras poligonales todas congruentes entre s´ı y los a´ngulos en sus v´ertices son tambi´en congruentes. Y s´olo existen cinco poliedros regulares: el tetraedro, el hexaedro o cubo, el octaedro, el dodecaedro y el icosaedro. Mostraremos aqu´ı un razonamiento basado en el teorema 21 del libro XI de los Elementos de Euclides, el cual dice que “todo a´ngulo s´olido es contenido por ´angulos menores que 4 a´ngulos rectos”. Veamos: 11

CAMPOS, A.; El m´ as bello teorema, Memorias VII Encuentro de Geometr´ıa y sus Aplicaciones, Bogot´a, 1996.

162

´ ´ LOS NUMEROS NATURALES EN OTRAS RAMAS DE LA MATEMATICA

Con 3 tri´angulos equil´ateros se construye un a´ngulo s´olido del tetraedro, cuya medida es la suma de las medidas de los a´ngulos de los tri´angulos que forman el v´ertice del ´angulo del tetraedro (esto es 3 × 60 = 180 < 360); con 4 el a´ngulo del octaedro (4×60 = 240 < 360), con 5 el a´ngulo del icosaedro (5×60 = 300 < 360). Con 6 tri´angulos equil´ateros, no se puede formar un a´ngulo s´olido, porque, como el a´ngulo del tri´angulo equil´atero es dos tercios de un a´ngulo recto, los 6 ser´ıan iguales a 4 a´ngulos rectos, lo que es imposible. El ´angulo de un cubo est´a formado por 3 cuadrados y no se puede formar un ´angulo s´olido con 4 cuadrados, porque ser´ıan 4 a´ngulos rectos. El ´angulo del dodecaedro regular est´a formado por 3 pent´agonos equil´ateros, y construir un ´angulo s´olido con 4 pent´agonos es imposible, puesto que el a´ngulo del pent´agono regular es 6/5 de un a´ngulo recto, los 4 a´ngulos ser´ıan mayores que 4 a´ngulos rectos. Imposible.

En conclusi´on, s´olo hay 5 poliedros regulares. El n´ umero de sus v´ertices, aristas y caras est´a en la tabla 6. Poliedro regular

N´ umero de v´ertices de caras

N´ umero de Aristas

N´ umero

Tetraedro

4

6

4

Hexaedro

8

12

6

Octaedro

6

12

8

Dodecaedro

20

30

12

Icosaedro

12

30

20

163

´ ´ ACTIVIDADES MATEMATICAS PARA EL DESARROLLO DE PROCESOS LOGICOS

Para todos ellos la caracter´ıstica de Euler es 2. Una demostraci´on anal´ıtica de este hecho, donde se utiliza la relaci´on de Euler V − A + C = 2 se encuentra en el libro de COURANT, R. y ROBBINS, R.12

5.9.

Contar en teor´ıa de grafos.

1. Cuentas en teor´ıa de Grafos. Algunas de las siguientes figuras se pueden trazar sin levantar el l´apiz y sin repetir l´ınea, ¿cu´ ales?.

Figura 8 Observe la siguiente secuencia de figuras. Todas ellas se pueden dibujar con las condiciones estipuladas. En la figura 9 encontramos el caso m´as simple, en donde el punto de partida (i) coincide con el punto de llegada (f). Notemos que en estos trazos, la forma no es determinante para la soluci´on del problema13 ; ellos son esencialmente los mismos. i,f

i,f

i,f

Figura 9 La figura 10 muestra nuevamente tres trazos esencialmente iguales, en el sentido que tienen la misma soluci´on, aunque su forma es distinta. Busque otras. 12

¿Qu´e es la Matem´ atica?, Aguilar, Madrid, 1971, p.p. 242 - 246. Existen otros problemas en donde la forma de las figuras es irrelevante para su soluci´ on; una de las ramas de la matem´atica que se ocupa de algunos de ellos es la topolog´ıa. 13

164

´ ´ LOS NUMEROS NATURALES EN OTRAS RAMAS DE LA MATEMATICA

f

i

f

i

i

f

Figura 10 El primer trazo de la figura 10 se obtiene mediante la adici´on de una l´ınea interior al primer trazo de la figura 9 y por un proceso similar, podemos obtener nuevas situaciones que tambi´en son equivalentes, como las que se muestran en la figura 11. f i

i f

f

i

f

i

Figura 11 Exprese un criterio seg´ un el cual son equivalentes los trazos en cada una de las figuras anteriores. La figura 12 muestra otros trazos que se pueden hacer, sin repetir l´ınea y sin levantar el l´apiz, que pueden servir para que usted intente sus propias conjeturas, sin importar que logre o no una conclusi´on v´alida, mediante el pegamiento de figuras similares al primer trazo de la figura 10.

Figura 12 165

´ ´ ACTIVIDADES MATEMATICAS PARA EL DESARROLLO DE PROCESOS LOGICOS

Sin embargo, si adicionamos una l´ınea m´as al primer trazo de la figura 11, la construcci´on ya no es posible. Este y otros casos imposibles se muestran en la figura 13. Estudie en las figura que hemos presentado y determine en qu´e condiciones se pueden trazar sin levantar el l´ apiz y sin repetir l´ınea.

Figura 13 En figuras como las que hemos presentado se acostumbra llamar v´ertices a los puntos de los cuales parten las l´ıneas, v´ertice impar si de ´el parte un n´ umero impar de l´ıneas, y par en caso contrario. Por ejemplo, el v´ertice A en la figura 13 es par y el B es impar.

E

A

D

B

C Figura 14 166

´ ´ LOS NUMEROS NATURALES EN OTRAS RAMAS DE LA MATEMATICA

El problema de hacer trazos sin levantar el l´apiz y sin repetir l´ınea est´a relacionado con uno que le fue propuesto al matem´atico Leonhard Euler, sobre un paseo por la ciudad de K¨onigsberg (actualmente Kaliningrado); all´ı hay una isla llamada Kueiphof, el r´ıo que la rodea se divide en dos brazos y sobre ellos, en tiempos de Euler, estaban colocados siete puentes, de la forma como se indica en la figura 15. Los habitantes del lugar intentaban descubrir una trayectoria para sus paseos, de forma tal que pudiesen regresar al punto de partida despu´es de haber cruzado por los siete puentes, pero pasando por cada uno s´olo una vez.

Euler present´o una soluci´on a un problema m´as general: “Dada una configuraci´on cualquiera del r´ıo y de los brazos en que pueda dividirse, as´ı como un n´ umero cualquiera de puentes, determinar si es o no posible cruzar cada puente exactamente una vez”, de la siguiente forma: Euler present´o una soluci´on a un problema m´as general: “Dada una configuraci´on cualquiera del r´ıo y de los brazos en que pueda dividirse, as´ı como un n´ umero cualquiera de puentes, determinar si es o no posible cruzar cada puente exactamente una vez”, de la siguiente forma: Si hay m´as de dos regiones a las que conduce un n´ umero impar de puentes, ninguna ruta se puede hallar que satisfaga las condiciones requeridas. Si, por el contrario, hay s´olo dos regiones con un n´ umero impar de puentes que conduzcan a ellas, el camino pedido se puede realizar, supuesto que comience en una de tales regiones. Si, finalmente, no hay ninguna regi´on a la que conduzca un n´ umero impar de puentes, el camino pedido se puede realizar comenzando en cualquier regi´on. 167

´ ´ ACTIVIDADES MATEMATICAS PARA EL DESARROLLO DE PROCESOS LOGICOS

Una soluci´on propuesta en teor´ıa de grafos para el problema de trazar figuras sin levantar el l´apiz y sin repetir l´ınea se muestra a continuaci´on14: 1. Las figuras donde todos sus v´ertices son pares se pueden dibujar con un trazo continuo partiendo de un v´ertice cualquiera

Figura 16 La estrella pentagonal (figura 16), s´ımbolo que empleaban los pitag´oricos para reconocerse, y la firma de Mahoma, formada por dos medias lunas opuestas (figura 17), y que seg´ un la tradici´on, trazaba el Profeta con la punta de su cimitarra, son dos de estos casos.

Figura 17 2. Cuando una figura tiene dos v´ertices impares, puede describirse con trazo continuo partiendo de uno de dichos v´ertices. As´ı, por ejemplo, la figura 18 puede describirse en cualquier sentido con tal de que se parta de una de sus v´ertices impares B o D. 14

TAHAN, M., El Hombre que Calculaba, Ed. Panamericana, Bogot´ a, 1997, p. 236.

168

´ ´ LOS NUMEROS NATURALES EN OTRAS RAMAS DE LA MATEMATICA

C B

D E A

F Figura 17

En iguales condiciones se halla la figura 19.

Figura 19

Las figuras que tienen m´as de dos v´ertices impares, no pueden describirse con un trazo continuo.

Compare esta soluci´on con la dada por Euler al problema de los puentes. ¿A qu´e corresponde una regi´ on en el lenguaje de los trazos?, ¿a qu´e corresponde un puente?; dibuje el trazo15 correspondiente. 15

La palabra trazo que utilizamos aqu´ı, corresponde a grafo en los libros dedicados a sus estudio, por ejemplo: JOHNSON, D., JOHNSON J., Graph Theory with Engineering applications, Ronald Press, 1972.

169

´ ´ ACTIVIDADES MATEMATICAS PARA EL DESARROLLO DE PROCESOS LOGICOS

5.10.

Contar en teor´ıa de probabilidades

Los elementos que aportan estos ejercicios son utilizados en la formulaci´on de los conceptos b´asicos de la teor´ıa de probabilidades16: 1. Estudiemos de cu´antas formas se pueden colocar en un rengl´on 1, 2, 3 y 4 letras respectivamente. En el primer caso s´olo tenemos una forma. En el segundo caso, si las letras son a y b, ´estas las podemos colocar de dos formas diferentes ab y ba Para tres letras, por ejemplo a,b y c, las podemos colocar de 6 formas distintas a saber: a b c,

a c b,

b a c,

b c a,

c a b,

cba

notemos que 2 empiezan con a, dos con b y dos con c, en total 3 × 2 = 6; tres formas de empezar con dos posibilidades cada una. Si son 4 letras, digamos a, b,c y d, tenemos 4 maneras de empezar, y por el ejercicio anterior, 6 posibilidades en cada una, por ejemplo: a b c d,

a b d c,

a c b d,

a c d b,

a d b c,

adcb

Ahora, colocando la b en la primera posici´on nos resultar´an otras seis posibilidades y si continuamos con cada una de las letras vamos a obtener seis posibles formas por cada letra; lo que nos da 4 × 3 × 2 = 24 maneras diferentes para las cuatros letras(con cuatro letras cada una). ¿Cu´antas formas posibles existen para colocar 5 letras? ¿Cu´ antos posibles arreglos se pueden encontrar con 10 elementos diferentes?; ¿con 25?, ¿con k letras? 16

KREYSZIG, E., Introducci´ on a la estad´ıstica matem´ atica, principios y m´etodos. Limusa, 1978.

170

´ ´ LOS NUMEROS NATURALES EN OTRAS RAMAS DE LA MATEMATICA

2. Si tenemos 5 objetos que no se puedan distinguir por el tacto, por ejemplo, bolas de igual volumen y peso, pero distinto color en una bolsa, hay exactamente 5 posibilidades de sacar una bola de un color determinado. Si deseamos sacar dos bolas, hay 5 posibilidades de sacar la primera y 4 posibilidades de sacar la segunda, pues ya hay una menos en la bolsa. Es decir, 5 × 4 en total. Para sacar la tercera bola hay 5 × 4 × 3 posibilidades. Si sacamos 4 bolas de 6 posibles, ¿cu´antas posibilidades hay?; ¿si sacamos k bolas de n posibles? 3. Complicando el problema anterior un poco m´as, deseamos saber (y esto es fundamental en cualquier estudio) de cu´antas formas se puede escoger 2 bolas de 5 posibles, pero en esta ocasi´on no distinguimos entre sacar la bola marcada con 1 y luego la marcada con 2, y la elecci´on en orden contrario; es decir, sacar primero la bola 2 y luego la bola 1. En este caso, las posibilidades se reducen a la mitad; es decir, dividimos la respuesta del problema anterior, entre 2. Si en lugar de 2, sacamos 3, 4, bolas, sin distinguir el orden de aparici´ on, ¿cu´antas formas son posibles?; ¿si sacamos k bolas de n posibles? 4. ¿De cu´antas formas pueden 7 personas estar sentadas en una silla de una cafeter´ıa con capacidad para 3 personas? 5. Se quieren sentar 3 hombres y 2 mujeres en una fila de modo que las mujeres ocupen los sitios pares. ¿De cu´antas formas pueden sentarse?, ¿y si son 5 hombres y 4 mujeres? 6. ¿De cu´antas formas pueden sentarse 4 personas alrededor de una mesa de 3 puestos, si pueden sentarse de cualquier forma? 7. ¿De cu´antas formas pueden sentarse 5 personas alrededor de una mesa de 3 puestos, si dos personas determinadas no deben sentarse una al lado de la otra? 8. Con 3 mujeres y 5 hombres quiere formarse un comit´e de 2 mujeres y 3 hombres. ¿Cu´ antos comit´es distintos pueden formarse si: 171

´ ´ ACTIVIDADES MATEMATICAS PARA EL DESARROLLO DE PROCESOS LOGICOS

i. No se impone ninguna restricci´ on? ii. Dos mujeres determinadas deben estar en el comit´e? iii. Un determinado hombre no debe estar en el comit´e? 9. En dos cajas hay encerrados un mango y una papaya; si sabemos que en cada caja hay una sola fruta, es obvio que sacando la fruta que hay en una de ellas sabremos el contenido de la otra.

Similarmente es posible saber el contenido de tres cajas, donde hay tres mangos y tres papayas, sacando s´olo una fruta de cada caja y sabiendo que en cada caja hay dos frutas y que no hay en ellas la combinaci´on de frutas que se indica en la figura. Encuentre la soluci´ on. Enuncie y resuelva una generalizaci´ on de este problema para 4 cajas, 6 mangos y 6 papayas.

MM

PM

PP

Figura 19

10. ¿Con una balanza es posible distinguir una moneda falsa entre tres, sabiendo que la falsa pesa menos y haciendo s´olo una pesada?; ¿y si son 6, 8, o 9 monedas, se puede en dos pesadas?

¿Y si la falsa pesa distinto de las otras dos, pero no sabemos si m´ as o menos, se puede distinguir una de tres haciendo dos pesadas? ¿Una de 9 o de 12 en tres pesadas? ¿Hay una regla general para este problema? 11. Observe la tabla siguiente y complete los sitios que faltan. 172

´ ´ LOS NUMEROS NATURALES EN OTRAS RAMAS DE LA MATEMATICA

1

1

1

1

1

1

1

2

3

4

5

6

7

8

3

6

10

15

21

28

36

4

10

20

35

56

84

120

5

15

35

70

126

230

350

1

6 ¿C´ omo es la fila k? Esta tabla tiene una curiosidad hist´orica: la utiliz´o Pascal para sus estudios en teor´ıa de juegos. Adem´as, si se lee, no por filas y columnas sino por diagonales y columnas, nos indica el n´ umero de grupos que pueden hacerse con el n´ umero de elementos que indica la columna seleccionada disminuida en uno, a partir de un conjunto de elementos cuyo n´ umero queda indicado por la diagonal seleccionada. Por ejemplo, el elemento de la cuarta diagonal y la tercera columna nos dice el n´ umero de combinaciones posibles de cuatro elementos tomados de 2 en 2.

5.11.

Contar en teor´ıa de conjuntos

La teor´ıa de conjuntos es una rama de la matem´atica que trata de fundamentar a ´esta, utilizando como punto de partida los conceptos de conjunto y de pertenencia de un elemento a un conjunto y con unos axiomas que relacionan los conceptos b´asicos. En teor´ıa de conjuntos, como en casi todas las ramas de la matem´atica aparece el conteo de forma natural. Asumiremos de momento que podemos contar los elementos de conjuntos particulares y nos ponemos por tarea contar los elementos de conjuntos formados a partir de ellos.

5.11.1.

Subconjuntos y el conjunto de partes

Un conjunto X es subconjunto de otro conjunto Y si y s´olo si todo elemento de X es elemento de Y. El conjunto de todos los subconjuntos de un conjunto X es el conjunto de partes de X y se denota ℘(X). 173

´ ´ ACTIVIDADES MATEMATICAS PARA EL DESARROLLO DE PROCESOS LOGICOS

Si un conjunto X tiene n elementos, ¿cu´antos subconjuntos tiene X? Es decir, ¿cu´antos elementos tiene su conjunto de partes ℘(X)? Empecemos, como de costumbre, haciendo una lista: Si X es vac´ıo naturalmente tiene un solo subconjunto. Si X tiene 1 elemento, tiene 2 subconjuntos, etc. Encuentre todos los subconjuntos en conjuntos con 3, 4, 5, 6 elementos y complete la siguiente tabla: Elementos

Subconjuntos

0

1

1

2

2 3 4 5 6 Escriba una f´ ormula para el caso en que el conjunto tenga n elementos. Observe en el proceso de formaci´ on de la tabla. ¿Cu´antos conjuntos se agregan en cada paso? D´e una explicaci´on para ello.

5.11.2.

El producto cartesiano de conjuntos

Dados dos conjuntos, A y B, el producto cartesiano de A y B notado A × B, es el conjunto formado por las parejas ordenadas (a, b) de manera que el primer elemento pertenece a A y el segundo a B; es decir: A × B = {(a, b) : a ∈ A y b ∈ B} Si queremos averiguar el n´ umero de elementos de un producto conociendo el n´ umero de elementos de cada uno de sus factores, procedemos por partes: 174

´ ´ LOS NUMEROS NATURALES EN OTRAS RAMAS DE LA MATEMATICA

Primero, tomemos el conjunto A con un elemento; por ejemplo, A = {a} y el conjunto B, tambi´en con 1 elemento, digamos B = {x}; el n´ umero de parejas ordenadas que se puede formar teniendo como primera componente a elementos de A y como segunda componente a elementos de B, es solamente: A × B = {(a, x)} Si A tiene 1 elemento y B tiene 2 elementos el producto naturalmente tiene 2 parejas, as´ı sucesivamente; luego tomamos un conjunto A con 2 elementos y variamos el n´ umero de elementos de B. Complete la siguiente tabla:

5.11.3.

N´ umero de

N´ umero de

N´ umero de

elementos de A

elementos de B

elementos de A × B

1

1

1

1

2

2

1

3

3

1

k

k

2

1

2

2

2

4

2

3

6

2

k

2k

·

·

·

·

·

·

n

k

?

Relaciones

A cualquier subconjunto del producto cartesiano A × B lo llamamos una relaci´ on de A en B. Si el n´ umero de elementos de A en n y el de B es m, ¿cu´antas relaciones hay de A en B? 175

´ ´ ACTIVIDADES MATEMATICAS PARA EL DESARROLLO DE PROCESOS LOGICOS

5.11.4.

Funciones

Una funci´on f de un conjunto A en un conjunto B es una relaci´on de A en B, tal que todo elemento de A est´a como primera componente en alguna pareja ordenada de f, pero solamente en una. Es decir, que si la pareja (x, y) ∈ f y la pareja (x, z) ∈ f, debe tenerse que y = z. El conjunto de todas las funciones que se pueden definir de un conjunto A en otro conjunto B lo notamos B A . Nuestro prop´osito es averiguar cu´antos elementos tiene B A , si A tiene n elementos y B tiene m elementos. De nuevo recurramos a las tablas por casos: Supongamos que A tiene s´olo un elemento y B tiene un elemento; por ejemplo: A = {0} y B = {a} Una funci´on de A en B debe tener por lo menos una pareja que tenga como primera componente a 0, y el u ´nico elemento para combinar en B es a, luego la u ´nica funci´on de A en B es: f = {(0, a)} Si A tiene s´olo un elemento y B tiene dos elementos, por ejemplo: A = {0} y B = {a, b} Una funci´on de A en B debe tener por lo menos una pareja que tenga como primera componente a 0, y pero ya hay dos posibilidades para combinar en B, una con a y otra con b, luego hay dos funciones de A en B, ellas son: f = {(0, a)} y g = (0, b) As´ı, es f´acil intuir que si A tiene un elemento y B tiene n elementos, en total hay n funciones de A en B; una por cada elemento de B como segunda componente. Si A tiene n elementos y B s´olo uno, la situaci´on es distinta, pues s´olo hay una funci´on de A en B. ¿Por qu´e? Complete la siguiente tabla: 176

´ ´ LOS NUMEROS NATURALES EN OTRAS RAMAS DE LA MATEMATICA

5.11.4.1.

#A

#B

#BA

1

n

n

n

1

1

2

2

4

2

3

9

2

n

3

2

8

3

3

27

3

n

n

4

n

m

Funciones inyectivas

Una funci´on f de un conjunto A en un conjunto B es una funci´on inyectiva o uno a uno si todo elemento de B que est´a en alguna pareja de la funci´on, est´a s´olo una vez; es decir, que si la pareja (x, y) ∈ f y la pareja (z, y) ∈ f debe tenerse que x = z. Dicho de otra manera: si para todo x, y elementos de A se cumple que f(x) = f(y), entonces debe tenerse que x = y. Para contar cu´antas funciones inyectivas hay de un conjunto A con n elementos en un conjunto B con m elementos debemos distinguir dos casos: i. Si m ≤ n. ii. Si n ≤ m. En el primero, no hay funciones inyectivas de A en B. ¿Por qu´e? Para determinar el n´ umero de funciones inyectivas de A en B, en el segundo caso, haga una tabla similar a las anteriores y establezca una f´ ormula. 177

´ ´ ACTIVIDADES MATEMATICAS PARA EL DESARROLLO DE PROCESOS LOGICOS

5.11.4.2.

Funciones sobreyectivas

Una funci´on f de un conjunto A en un conjunto B es una funci´on sobreyectiva o simplemente sobre si todo elemento de B est´a en alguna pareja de la funci´on; es decir, que para todo elemento y de B exista un elemento x de A tal que la pareja (x, y) ∈ f. Para contar cu´antas funciones sobreyectivas hay de un conjunto A con n elementos en un conjunto B con m elementos, tambi´en debemos distinguir dos casos: i. Si n ≤ m. ii. Si m ≤ n. En el primer caso, no hay funciones sobreyectivas de A en B. ¿Por qu´e?. Para el segundo, haga una tabla similar a las anteriores y establezca una f´ormula para el n´ umero de funciones sobreyectivas de A en B. 5.11.4.3.

Funciones biyectivas

Una funci´on f de un conjunto A en un conjunto B es una funci´on biyectiva si es inyectiva y sobreyectiva. De las dos secciones anteriores se desprende que la u ´nica manera de que hayan funciones biyectivas entre dos conjuntos A y B es que ambos tengan el mismo n´ umero de elementos. ¿Por qu´e? ¿Cu´antas funciones biyectivas hay entre dos conjuntos que tengan n elementos? 5.11.4.4.

Operaciones

Una operaci´on binaria interna en un conjunto A es una funci´on ∗:A×A→A ¿Cu´antas operaciones binarias internas son posibles en un conjunto de n elementos?

178

CAP´ITULO 6 ´ EN MATEMATICAS ´ LA INDUCCION Introducci´ on Uno de los procesos l´ogicos que m´as han desarrollado las personas que se dedican a estudiar matem´aticas, es el proceso de encontrar regularidades en una sucesi´on de resultados a partir de algunos casos particulares, esto es, encontrar una manera general de decir todos los casos, hallar f´ormulas que resuman en una sola expresi´on todas las caracter´ısticas de un grupo de informaciones; este proceso lo llamaremos inducir 1 . Este proceso, como casi todos, no se desarrolla de manera espont´anea a partir de una explicaci´on, sino, por el contrario, va surgiendo, se va formando lentamente, saliendo del ejercicio cotidiano, de la insistencia, de la continua repetici´on. Es fruto de muchos ensayos y errores, de volver a empezar, pero, sobre todo, de la persistencia en el trabajo. En el ejercicio del proceso de inducir, es frecuente enfrentarse a dificultades adicionales, como, por ejemplo, la sospecha de que una f´ormula que es v´alida en muchos casos lo es en general; dificultad que s´olo superaremos cuando estudiemos el principio de inducci´ on matem´ atica 2. 1

El l´ ogico Charles S. Pierce llama abducci´ on a este proceso. La expresi´on inducci´ on matem´ atica tiene un significado particular en matem´ aticas como m´etodo de demostraci´on de afirmaciones que se hacen sobre todos los n´ umeros naturales; este uso lo precisaremos en el cap´ıtulo 7. 2

179

´ ´ ACTIVIDADES MATEMATICAS PARA EL DESARROLLO DE PROCESOS LOGICOS

Otra dificultad radica en la tendencia a creer que todo proceso que tiene alg´ un tipo de regularidad es susceptible de ser modelado mediante una f´ormula, situaci´on que en muchos casos no se tiene; sin embargo, la ganancia obtenida en el intento es suficiente retribuci´on por el esfuerzo. Iniciaremos recurriendo a im´agenes que nos permitan “afinar el olfato”para percibir relaciones entre los n´ umeros, que nos lleven a establecer conjeturas y verificarlas en casos particulares.

6.1.

Los n´ umeros y el espacio

Los n´ umeros naturales tienen una gran simplicidad para su comprensi´on, lo que facilita la familiarizaci´on con ellos de manera relativamente r´apida, pero encontrar y sistematizar relaciones entre ellos puede llegar a complicarse mucho, aunque en la mayor´ıa de los casos pueden ser enunciadas de manera simple. Esta aparente dificultad les da una ventaja pedag´ogica, pues con pocos conocimientos podemos plantear y resolver problemas cuyo enunciado es de f´acil comprensi´on, permitiendo, adem´as, distintos niveles de profundidad en las soluciones propuestas. Comenzaremos por estudiar algunas curiosidades de los n´ umeros que forman 3 parte del legado matem´atico griego , las cuales, por su cercan´ıa con representaciones geom´etricas sencillas, nos ayudan a desarrollar la intuici´on y con ella la capacidad de abstracci´on y generalizaci´on.

6.1.1.

N´ umeros pares e impares

Cuando estudiamos los n´ umeros naturales, una primera observaci´on permite diferenciar dos tipos de n´ umeros, que aparecen cuando contamos de dos en dos, a partir de 0 o a partir de 1. Los primeros, conocidos como n´ umeros pares son: 0, 2, 4, 6, 8, 10, 12, 14, 16, 18, 20, . . . 3

Los griegos, hacia el siglo IV a.C. desarrollaron los conceptos b´ asicos de lo que hoy conocemos con el nombre teor´ıa de n´ umeros.

180

´ EN MATEMATICAS ´ LA INDUCCION

Los segundos, llamados n´ umeros impares, son: 1, 3, 5, 7, 9, 11, 13, 15, 17, 19, 21, . . . Iniciemos tratando de determinar cu´al es el par n´ umero 100. Un camino es notar la regularidad en la sucesi´on de n´ umero pares y establecer una f´ormula. Como la diferencia entre cada uno de ellos es 2, al cabo de n pasos, obtenemos 2n si partimos de 0. Para los n´ umeros impares el mismo proceso nos da 2n − 1 si empezamos a contar en 1. Empecemos a formular preguntas, por ejemplo: ¿la suma de dos pares es par?; ¿la de dos impares?, ¿el producto? Ejercicios 1. Sea p un impar y n un n´ umero natural cualquiera, entonces p2 + np a) ¿es siempre impar? b) ¿es siempre par? c) ¿es par si n es par? d ) ¿es impar si n es impar? e) ¿es impar si n es par? Justifique su respuesta. 2. Encuentre el promedio4 de los 7777 primeros n´ umeros impares. 3. Si en lugar de contar de 2 en 2, partiendo de 0, cont´ aramos de 3 en 3, obtendr´ıamos la siguiente lista: 0, 3, 6, 9, 12, 15, 18, 21, 24, 27, . . . ¿Comparte ella algunas propiedades de los n´ umeros pares? 4

El promedio de un conjunto de n´ umeros es el resultado de dividir su suma entre el n´ umero de elementos del conjunto.

181

´ ´ ACTIVIDADES MATEMATICAS PARA EL DESARROLLO DE PROCESOS LOGICOS

¿Si partimos de 1, cu´al es la lista?, ¿si partimos de 2?, ¿si partimos de 3?, etc.; ¿cu´antas listas hay que no tengan elementos en com´ un? Contemos ahora de 4 en 4, a partir de 0, de 1, de 2, etc. Enuncie conclusiones generales cuando contamos de k en k, donde k es cualquier n´ umero natural.

6.1.2.

Los n´ umeros triangulares

Los disc´ıpulos de Pit´agoras5 estudiaban los n´ umeros absolutos (aritm´etica), los n´ umeros aplicados (la m´ usica), las magnitudes en reposo (geometr´ıa) y las magnitudes en movimiento (astronom´ıa). La ciencia de los n´ umeros absolutos estudiaba los n´ umeros poligonales, en especial triangulares y cuadrados, las razones y proporciones, los divisores num´ericos que se relacionan con los n´ umeros 6 umeros por primos, perfectos, etc., y las progresiones . Ellos representaban los n´ medio de agujeros en la arena o por medio de piedras. As´ı, por ejemplo, los n´ umeros triangulares los formaban de acuerdo al anterior, a˜ nadiendo l´ıneas de piedras conformando tri´angulos equil´ateros, como se muestra en la figura 1:

• 1

• • • 3



• • • • • 6

• • • • • • • • • • 10

Figura 1 Los primeros n´ umeros triangulares son: 1, 3, 6, 10, 15, 21, 28, 36, 45, . . . 5

Para los pitag´ oricos los n´ umeros lo eran todo, no s´ olo eran el instrumento para comprender y explicar todos los fen´ omenos del universo, sino que, adem´as, eran de naturaleza divina. Le atribu´ıan cualidades morales a los n´ umeros; en la matem´atica griega se encuentran n´ umeros perfectos, n´ umeros amigables, y otras muchas curiosidades. El n´ umero 1 representaba la dignidad, el 2 representaba el mal, el 3 la armon´ıa, el 5 el matrimonio, el 7 a la diosa Atenea. 6 CARO V., Los n´ umeros: Su historia, sus propiedades, sus mentiras y verdades, Minerva, Bogot´a, 1936, pp. 13-14.

182

´ EN MATEMATICAS ´ LA INDUCCION

Ejercicio o

¿Cu´ales son los cuatro siguientes n´ umeros triangulares? ¿Cu´al es el 35 n´ umero triangular?

Una manera de resolver este problema es hacer los dibujos que contin´ uen la secuencia presentada y contar los puntos. Por supuesto, este proceso no es pr´actico. Sin necesidad de recurrir al dibujo, podemos encontrar una manera para saber cu´antos puntos hay en un tri´angulo equil´atero, como los que aparecen en la figura 1, pero esta vez de lado: 9, 10, 15, 157, etc. La mente ve donde los ojos no alcanzan. Se hace necesario detectar las regularidades que nos permitan saberlo. Cuando lo logramos, esperamos que esa regularidad sea v´alida para todos los casos, pero no podemos escribirla y verificarla para cada uno de ellos; tenemos que hacer una representaci´on simb´olica de los elementos involucrados en las relaciones del problema. En el ejemplo que nos ocupa, al relacionar los n´ umeros triangulares con los lados de los tri´angulos formados, podemos empezar haciendo una lista de los primeros quince n´ umeros triangulares: n 1 2 3 4 5 6 7 8 9 10 11 12 13 14 15 183

Tn 1 3 6 10 15 21 28 36 45 55 66 78 91 105 120

´ ´ ACTIVIDADES MATEMATICAS PARA EL DESARROLLO DE PROCESOS LOGICOS

Ejercicios 1. Observe y enuncie regularidades de acuerdo a la tabla 8. 2. Si la letra n representa a un n´ umero natural cualquiera y el n-´esimo n´ umero ormula para Tn . triangular lo notamos Tn , escriba una f´ 3. Una idea que puede ser de utilidad para sumar los primeros 100 n´ umeros naturales (en base 10) es usar el hecho de que 100 + 1 = 99 + 2 = 98 + 3, etc. ¿Es v´alida esta afirmaci´ on en base 8?¿en otras bases? 4. Sume n´ umeros triangulares consecutivos y escriba una f´ ormula para la suma del n-´esimo n´ umero triangular con el siguiente. 5. ¿Siempre dos n´ umeros triangulares consecutivos tienen un factor en com´ un? Justifique su respuesta. 6. En 1640, Pierre de Fermat encontr´o que todo n´ umero natural o es triangular, o es suma de dos o tres triangulares. En 1796, Carlos Federico Gauss demostr´o que todo n´ umero natural es la suma de tres n´ umeros triangulares (incluyendo el 0 como n´ umero triangular); exprese 954, 2500, 10000 y otros n´ umeros en esta forma. 7. En la vida cotidiana hemos aprendido que s´olo se pueden sumar cosas con id´enticas caracter´ısticas; por ejemplo, si sumamos 3 papas con 5 papas, el resultado es 8 papas; si abstraemos las cosas y dejamos s´olo los n´ umeros, al sumar dos de ellos obtenemos uno de los mismos. Tenemos entonces derecho a la pregunta, ¿la suma de dos n´ umeros triangulares es un n´ umero triangular? Hay casos en que as´ı es7 ; por ejemplo, la tabla 8 nos permite observar que: T3 + T5 T4 + T9 T5 + T14 T6 + T20 7

= T6 = T10 = T15 = T21

´ L., MORENO, H., LUQUE C., Un an´ PINZON, alogo al teorema de Pit´ agoras para n´ umeros triangulares, Memorias IX Encuentro de Geometr´ıa y sus Aplicaciones, Bogot´a, 1998, p. 97.

184

´ EN MATEMATICAS ´ LA INDUCCION

Enuncie una f´ormula que unifique estas observaciones. En los casos mencionados hay una particularidad entre la segunda y tercera columnas; los sub´ındices respectivos difieren en una unidad, pero ´estos no son todos los casos; por ejemplo, se tiene que: T18 + T25 = T31 Por lo tanto, es necesario obtener una generalizaci´on que contenga todas las diferencias entre los sub´ındices. Para ello, organicemos diversas tr´ıadas pitag´ oricas triangulares a partir de la diferencia entre los sub´ındices, como se aprecia en la tabla 9:

1

2

3

4

5

T1 + T0 = T1

T2 + T0 = T2

T3 + T0 = T3

T4 + T0 = T4

T5 + T0 = T5

T2 + T2 = T3

T5 + T6 = T8

T8 + T10 = T13

T11 + T14 = T18

T14 + T18 = T23

T3 + T5 = T6

T6 + T9 = T11

T9 + T13 = T16

T12 + T17 = T21

T15 + T21 = T26

T4 + T9 = T10

T9 + T21 = T23

T11 + T20 = T23

T19 + T45 = T49

T19 + T35 = T40

T5 + T14 = T15

T10 + T26 = T28

T12 + T24 = T27

T20 + T50 = T54

T20 + T39 = T44

···

Observando la secuencia en la primera columna, podemos sugerir: Tn + T Tn −1  = T Tn −1  1

1

+1

Para la segunda columna, podemos escribir: Tn + T Tn −3  = T Tn −3 

+2

Tn + T Tn −6  = T Tn −6 

+3

2

2

Para la tercera: 3

3

En la cuarta: Tn + T Tn −10  = T Tn −10  4

4

+4

Proponga una f´ormula general y verifique algunos casos particulares. 185

k Tk + T0 = Tk

´ ´ ACTIVIDADES MATEMATICAS PARA EL DESARROLLO DE PROCESOS LOGICOS

8. Sume los dos primeros n´ umeros triangulares, los primeros 3, los primeros 4, los primeros 5; ¿ve algo? 9. Si queremos hallar la suma de los n primeros n´ umeros triangulares, la figura 2 puede ayudarnos; m´ırela, cont´emplela y trate de ver m´ as all´a. Estudie la f´ormula para la suma de los n primeros n´ umeros triangulares.

Si tiene problemas para visualizar objetos de tres dimensiones a partir de su representaci´on en dos dimensiones, construya un modelo en madera o cualquier otro material que le facilite evidenciar las relaciones. 10. El menor n´ umero de paquetes que se pueden colocar en dos sacos, de tal manera que en cada saco quede un n´ umero diferente de paquetes y que cada uno de ellos contenga por lo menos uno, es tres. ¿Cu´antos en tres sacos?, ¿cu´antos en cuatro sacos?, ¿cu´ antos en 7 sacos? 11. Invirtiendo la pregunta, el mayor n´ umero de sacos en los que se pueden repartir 28 paquetes de manera que cada uno tenga por lo menos uno, pero cada uno con un n´ umero distinto de paquetes es 7. ¿Cu´antos sacos con 55 paquetes? 186

´ EN MATEMATICAS ´ LA INDUCCION

12.

Y si tenemos 25 paquetes, podemos distribuirlos con las condiciones establecidas en m´aximo 6 sacos, ubicando, por ejemplo, 1, 2, 3, 4, 5 y 10 paquetes en cada uno de los sacos. Por supuesto, ´esta no es la u ´nica soluci´on posible. ¿De cu´antas formas distintas se puede hacer esta repartici´ on?

13.

Si el n´ umero de sacos es 17, ¿cu´al es el n´ umero m´ınimo de paquetes? Si el n´ umero de paquetes es 123,¿cu´al es el mayor n´ umero de sacos y de cu´ antas formas diferentes es posible hacerlo?¿En qu´e casos la soluci´ on es u ´nica?

14.

La base de una pila de naranjas est´a formada por un tri´angulo equil´atero de lado n, la segunda capa hacia arriba est´a formada por un tri´angulo equil´atero de lado n−1, la siguiente de lado n−2, y as´ı sucesivamente hasta terminar con una naranja en la cima, formando una pir´amide. ¿Cu´antas naranjas tiene la pir´ amide? Para cada n hay un n´ umero de ´estos; ellos son 8 conocidos como n´ umeros tri´ angulo-piramidales . Haga un estudio de estos n´ umeros similar al hecho para los n´ umeros triangulares. Los primeros son: 1, 4, 10, 20, 35, 56, . . .

15.

El n-´esimo n´ umero triangular es la suma de los n primeros n´ umeros naturales, el n-´esimo n´ umero factorial, que lo notamos n! es el producto de los primeros n n´ umeros naturales: n! = 1,2,3,4,5. . . . .n Intente encontrar regularidades para estos n´ umeros. Ensaye a definir n´ umeros an´alogos a ´estos con otras operaciones.

6.1.3.

Los n´ umeros cuadrados

Los n´ umeros cuadrados son los que corresponden al n´ umero de puntos que se pueden colocar formando un cuadrado. Cada uno de ellos puede ser construido 8

RODR´IGUEZ, Rafael, op. cit., p. 34

187

´ ´ ACTIVIDADES MATEMATICAS PARA EL DESARROLLO DE PROCESOS LOGICOS

a partir del anterior a˜ nadi´endole un borde de la forma borde ten´ıa especial significado y se le llam´o gnomon 9 . Los primeros n´ umeros cuadrados son:

. Para los griegos este

1, 4, 9, 16, 25, 36, 49, . . . Su expresi´on general es n2 . Los gnomones nos permiten descubrir una relaci´on entre n´ umeros impares y n´ umeros cuadrados: •

• • • •

• • • • • • • • •

• • • •

• • • •

• • • •

• • • •

Figura 3 Ejercicios 1. Observe la figura 3. Formule alguna hip´otesis acerca de la suma de los n primeros n´ umeros impares. 2. Para establecer una f´ormula que nos diga cu´anto suman los n primeros n´ umeros pares, podr´ıamos sugerir dos caminos: a) En la f´ormula 1+ 2 + 3+ 4 + ··· + n =

n(n + 1) 2

multiplicamos por 2 a ambos lados de la igualdad y obtenemos: 2 + 4 + 6 + 8 + · · · + 2n = n(n + 1) 9

La palabra gnomon signific´ o en Babilonia, una varilla vertical cuya sombra marcaba la hora. En la ´epoca de Pit´ agoras significaba una escuadra de carpintero y esta es la forma del gnomon que muestra la figura 3. Tambi´en signific´ o lo que queda de un cuadrado al cortar otro cuadrado m´ as peque˜ no en una de sus esquinas. M´ as tarde, con Euclides, su significado se ampli´ o a lo que queda de un paralelogramo al cortar otro m´ as peque˜ no en una de sus esquinas, siempre que ´este fuera semejante al primero.

188

´ EN MATEMATICAS ´ LA INDUCCION

b) Como en la suma de los n primeros n´ umeros naturales, observamos que el primer t´ermino y el u ´ltimo suman lo mismo que el segundo y el pen´ ultimo, y as´ı sucesivamente, todas las parejas suman 2n + 2 al n sumar n´ umeros pares. Si n es un n´ umero par hay parejas, la suma 2 debe ser n (2n + 2) = n(n + 1). 2 Proponga otra forma. Analice el caso de n impar. 3. Hemos visto que los n´ umeros cuadrados y los triangulares est´an relacionados, si miramos la figura 3, un n´ umero cuadrado puede expresarse como la suma de la cantidad de puntos que se encuentran sobre rectas paralelas a una diagonal. Con esto obtenemos la secuencia: 12 =1 22 =1 + 2 + 1 32 =1 + 2 + 3 + 2 + 1 Escriba una f´ ormula que describa esta situaci´ on. 4. Seguramente ya ha notado que en algunos casos particulares la suma de dos n´ umeros triangulares consecutivos es un n´ umero cuadrado. Sume el n´esimo n´ umero triangular con el siguiente para probar que esta afirmaci´ on es v´alida en general. 5. Fermat demostr´o10 que todo n´ umero natural o es un n´ umero cuadrado o es la suma de 2 o 3 o 4 n´ umeros cuadrados. Exprese 954, 2500, 10000 y otros n´ umeros en esta forma umero de la forma 8n + 3 es la suma de tres 6. Gauss encontr´o11 que todo n´ cuadrados de n´ umeros impares. Por ejemplo: 3 = 12 + 12 + 12 11 = 12 + 12 + 32 19 = 12 + 32 + 32 . Exprese 711, 1219, 2515 y otros n´ umeros en esta forma. 10

´ APOSTOL, T., Introducci´ on a la teor´ıa anal´ıtica de n´ umeros, Revert´e, Barcelona, 1984, p. 2. 11 NEWMAN, J.; op. cit., Vol. I., p. 231.

189

´ ´ ACTIVIDADES MATEMATICAS PARA EL DESARROLLO DE PROCESOS LOGICOS

7. Esta afirmaci´on es equivalente12 a que todo n´ umero natural es suma de tres 13 on de este hecho. n´ umeros triangulares . Busque alguna justificaci´ 8. Lagrange ide´o un procedimiento para dilucidar cu´ando un n´ umero de la forma nx + 1 es un cuadrado. Estudie este problema, comenzando en casos particulares con n = 2, 3, 4, etc. 9. Otra conjetura de Fermat, debida en realidad a Bachet, afirma que todo n´ umero, o es un cuadrado, o es la suma de dos, tres, o cuatro cuadrados. En efecto tenemos que: 1 = 12 2 = 12 + 12 3 = 12 + 12 + 12 4 = 22 5 = 12 + 22 6 = 12 + 12 + 22 7 = 12 + 12 + 12 + 22 8 = 22 + 22 9 = 12 + 22 + 22

10 = 12 + 32 20 = 22 + 42 30 = 12 + 22 + 52 40 = 22 + 62 50 = 52 + 52 60 = 12 + 12 + 32 + 72 70 = 12 + 12 + 22 + 82 80 = 42 + 82 90 = 32 + 92

Intente una descomposici´on semejante para n´ umeros un poco m´ as grandes y trate de encontrar alguna regularidad. 6.1.3.1.

Tr´ıadas Pitag´ oricas

Al igual que en los n´ umeros triangulares, hay n´ umeros cuadrados que se pueden escribir como la suma de otros dos n´ umeros cuadrados. Por ejemplo, el quinto n´ umero cuadrado (que es 25), se puede escribir como la suma del tercero y el cuarto n´ umeros cuadrados: 9 + 16; esto lo expresamos m´as brevemente as´ı: 32 + 42 = 52 Una vez que se tiene una terna, por ejemplo, la formada por los cuadrados de 3, 4, 5 respectivamente, pueden formarse infinitas de ellas tomando sus m´ ultiplos, puesto que si k es un n´ umero natural cualquiera, entonces (3k)2 + (4k)2 = (5k)2 12

Al suponer que una afirmaci´ on es cierta, la otra se deduce de ella y viceversa. GAUSS, C. F., Disquisitiones Arithmeticae, Academia Colombiana de Ciencias, Bogot´ a, 1995. 13

190

´ EN MATEMATICAS ´ LA INDUCCION

Pero este tipo de soluciones no es esencialmente nuevo y se deriva f´acilmente de 3, 4, 5. Nos interesan entonces las soluciones reducidas, que como 3, 4, 5 no tienen divisores comunes (Cuando sucede esto, se dice que 3, 4, 5 o los n´ umeros correspondientes son primos entre s´ı). Otras tr´ıadas que comparten esta propiedad, conocidas como tr´ıadas pitag´ oricas, est´an formadas por los cuadrados de los n´ umeros: 5, 12 7, 24 9, 40 11, 60

y y y y

13; 25; 41; 61;

32 + 42 52 + 122 72 + 242 92 + 402 112 + 602

= 52 = 132 = 252 = 412 = 612

Observemos la secuencia de formaci´on:

Los n´ umeros que inician las tr´ıadas forman la secuencia de los cuadrados de los n´ umeros impares; es decir, la n-´esima tr´ıada de esta secuencia tiene como primer umeros, elemento (2n + 1)2 . Notamos tambi´en que los segundos y los terceros n´ cuyos cuadrados conforman las tr´ıadas, son consecutivos. Debemos encontrar la regularidad de la segunda columna. Entre los n´ umeros de ella hay un factor com´ un; si lo extrae, encontrar´a una secuencia conocida. Encuentre una expresi´ on para la n-´esima tr´ıada pitag´ orica de esta secuencia. Ejercicios 1. Encuentre otras tr´ıadas pitag´ oricas 14 Estudie, por ejemplo, tr´ıadas de la forma x = 2n + 1, y = 2n(n + 1), z = 2n(n + 1) + 1 (Estas fueron estudiadas por los pitag´oricos 600 a˜ nos 14

Los babilonios conoc´ıan muchas ternas pitag´ oricas relacionadas con estas f´ormulas para k = 1: KLEIN, M.; El pensamiento matem´ atico, de la antig¨ uedad a nuestros d´ıas, Alianza, Madrid, 1994, p. 29

191

´ ´ ACTIVIDADES MATEMATICAS PARA EL DESARROLLO DE PROCESOS LOGICOS

antes de Cristo, o las estudiadas por Plat´on (430 a.C.), de la forma x = 4n, umero natural). y = 4n2 − 1, z = 4n2 + 1, donde n es cualquier n´

Para encontrar tr´ıadas pitag´oricas debemos notar que los tres n´ umeros no pueden ser todos impares, porque el cuadrado de un n´ umero impar es siempre impar y la suma de dos impares es par.

Tampoco puede ser que dos de ellos sean pares y el otro impar. S´olo pueden ser uno de ellos par y los otros impares. Con un poco m´as de trabajo puede demostrarse que una terna cualquiera reducida, est´a completamente determinada por un par de n´ umeros m y n, tales que uno es impar y el otro par y no tienen divisores comunes15.

2. Existen otras relaciones entre algunas de las tr´ıadas pitag´oricas; por ejemplo si a, b y c son m´ ultiplos de 3, 4 y 5 respectivamente, entonces se cumple que c = b + (b − a) = 2b − a. Si a, b y c son m´ ultiplos de 5, 12 y 13, se cumple que: c = (3a + 2b)/3. Enuncie un resultado similar para otras tr´ıadas pitag´ oricas. ¿Existe una regla general?

3. ¿Cu´al es el u ´nico n´ umero menor que nueve, tal que aumentado en uno da el doble de un cuadrado y cuyo cuadrado aumentado en uno da otra vez el doble de un cuadrado?

4. El gnomon que mencionamos en los n´ umeros cuadrados puede utilizarse de manera diversa para encontrar otras regularidades. Observe, por ejemplo, el siguiente esquema: 15

En el libro X de los Elementos de Euclides aparece la f´ ormula x = k(n2 − m2 ), y = 2kmn, 2 2 z = k(n + m ) donde k, m, n son n´ umeros naturales, con n > m, uno par y el otro impar y que no tengan factores primos comunes.

192

´ EN MATEMATICAS ´ LA INDUCCION

12

=

1

1

1

1

1

22

=

1

2

2

2

2

32

=

1

2

3

3

3

42

=

1

2

3

4

4

52

=

1

2

3

4

5

¿Podemos inducir de aqu´ı una f´ormula para la suma de los n primeros n´ umeros cuadrados16? 5. En analog´ıa a lo hecho con los n´ umeros tri´angulo-piramidales, podemos construir, n´ umeros cuadrado-piramidales, haciendo una pir´amide, pero esta vez de base cuadrada colocando n2 naranjas en ella; en la siguiente capa de encima colocamos (n − 1)2 naranjas, y as´ı sucesivamente hasta terminar con una naranja en la c´ uspide. ¿Cu´al es el n´ umero total de naranjas en la pir´ amide? 6. Escriba los primeros 7 n´ umeros cuadrado-piramidales. Haga un estudio de ellos.

6.1.4.

Los n´ umeros poligonales

An´alogamente a lo hecho hasta ahora, se construyen con otros pol´ıgonos regulares n´ umeros con las formas de pent´agonos, hex´agonos, etc. 16

Estudie por ejemplo la relaci´on: 1 2 + 2 2 + 3 2 + 4 2 + · · · + n2 =

n

k=1

Tk +

n−1

Tk = n(n + 1)(2n + 1)/6.

k=1

Los babilonios conoc´ıan expresiones concretas de la f´ormula: 12 + 22 + 32 + 42 + · · · + n2 = (1/3 + 2n/3)(1 + 2 + 3 + · · · + n). Ver: KLEIN, M., op. cit., p. 28

193

´ ´ ACTIVIDADES MATEMATICAS PARA EL DESARROLLO DE PROCESOS LOGICOS

El primer n´ umero pentagonal es el uno; el segundo, cuyos puntos forman los v´ertices de un pent´agono, es el 5; el tercero es 1+ 4+ 7 = 12, y as´ı sucesivamente, 22, 35, 51, 70, son n´ umeros pentagonales. En la figura 7 se encuentra la secuencia de formaci´on de estos n´ umeros y a partir de ella usted puede verificar, por ejemplo, que el n-´esimo n´ umero pentagonal se puede obtener mediante la f´ormula 3n2 − n 2 An´alogamente, los n´ umeros hexagonales son 1, 6, 15, 28, ... y en general, el n-´esimo n´ umero hexagonal est´a dado por la f´ormula: 2n2 − n Ejercicio 1. Copiando lo hecho hasta ahora para n´ umeros triangulares y cuadrados, estudie los n´ umeros pentagonales, hexagonales, etc. Inicialmente, con base en dibujos, como los presentados en la figura 4, intente hallar regularidades por usted mismo. Luego, busque informaci´on sobre el tema.

194

´ EN MATEMATICAS ´ LA INDUCCION

2. Encuentre una f´ ormula para el n-´esimo n´ umero poligonal asociado a un 17 pol´ıgono regular de p lados . 3. Gauss demostr´o que todo n´ umero se puede escribir como la suma de a lo m´as tres n´ umeros triangulares y a lo m´as cuatro n´ umeros cuadrados. 18 umero se puede Cauchy generaliz´o las soluciones demostrando que todo n´ escribir como la suma de a lo m´as p n´ umeros poligonales, donde p es el n´ umero de lados del pol´ıgono considerado. Exprese 954, 2500, 10000 y otros n´ umeros en esta forma. 1 1 1 1 1

2 3 4 5 6

3 5 7 9 11

4 7 10 13 16

5 9 13 17 21

6 11 16 21 26

7 13 19 25 31

8 15 22 29 36

9 17 25 33 41

10 19 28 37 46

Construya una nueva tabla cuya primera fila y primera columna son las mismas de la anterior, obteniendo cada elemento restante de la segunda tabla, sumando el elemento anterior en la fila de la nueva tabla con el de la fila anterior y la misma columna en la tabla inicial. ¿Le sugiere la nueva tabla alguna relaci´ on con los n´ umeros poligonales?

6.1.5.

N´ umeros c´ ubicos

De la misma forma como se ubicaron en el plano puntos formando v´ertices de pol´ıgonos regulares para visualizar los n´ umeros poligonales, pueden ´estos distribuirse en el espacio formando v´ertices de s´olidos conocidos. Un primer ejemplo de ello se encuentra en los n´ umeros c´ ubicos. Estos son los n´ umeros que representan a la cantidad de puntos que pueden disponerse en una red c´ ubica, como se muestra en la figura 5. Los primeros son: 1, 8, 27, 64 . . . , n3 17

Estudie, por ejemplo, la f´ ormula n +

n(n − 1)b para distintos valores de b , o expresiones 2

n(n(p − 2) + (4 − p)) . 2 18 Investigaci´on y Ciencia, Grandes Matem´ aticos, Temas 1, Prensa Cient´ıfica S.A., Barcelona, 1995, p. 73. de la forma

195

´ ´ ACTIVIDADES MATEMATICAS PARA EL DESARROLLO DE PROCESOS LOGICOS

• • •



• • •



• • • • •

• •

• • • • •

• • • •

• • • •

• • •

• •





Figura 5

Ejercicios 1. Observe: 8= 3+5 27 = 7 + 9 + 11 64 = 13 + 15 + 17 + 19 Escriba 125 como una suma similar y enuncie una regla general. 2. Otra curiosidad de los n´ umeros c´ ubicos la observamos en la siguiente secuencia: 8= 4+2+2 27 = 9 + 3 + 3 + 3 + 9 64 = 16 + 4 + 4 + 4 + 4 + 16 + 16 125 = 25 + 5 + 5 + 5 + 5 + 5 + 25 + 25 + 25 Escriba los siguientes tres n´ umeros c´ ubicos de esta forma y enuncie su regularidad. 196

´ EN MATEMATICAS ´ LA INDUCCION

3. Una manera de relacionar los n´ umeros cuadrados con los n´ umeros c´ ubicos se encuentra en la siguiente secuencia:

12 = 13 22 = 23 − 22 32 = 33 − 32 − 32 42 = 43 − 42 − 42 − 42

Exprese 72 , 152 y otros n´ umeros de esta forma. Proponga una explicaci´ on.

4. Una relaci´on entre los n´ umeros triangulares y los n´ umeros c´ ubicos se colige de:

13 = 1 13 + 23 = 32 13 + 23 + 33 = 62 13 + 23 + 33 + 43 = 102

Escriba los siguientes dos renglones de la secuencia.

5. Georg Schrage19 nos muestra una idea para comprender la relaci´on anterior en la figura 6:

19

SCHRAGE, George, “Sums of integers and sums of cubes”, en NELSEN, R., Proofs without words, The Mathematical Association of America, 1993, p. 90.

197

´ ´ ACTIVIDADES MATEMATICAS PARA EL DESARROLLO DE PROCESOS LOGICOS

1 1 + 2 + · · · + n = (n + 1)n 2 2 1 n(n + 1) 13 + 23 + · · · + n3 = 2 1 2

1 × 12 2 × 22 3 × 32 .. . n × n2

3 .. . n

Figura 6

Estudie la figura y exprese sus propias conclusiones. 6. El gnomon tambi´en puede ense˜ narnos algo sobre la suma de los primeros n n´ umeros c´ ubicos. Observe y conjeture alguna relaci´ on.

13

=

1

2

3

4

5

23

=

2

4

6

8

10

33

=

3

6

9

12

15

43

=

4

8

12

16

20

53

=

5

10

15

20

25

198

´ EN MATEMATICAS ´ LA INDUCCION

6.1.5.1.

Anotaciones sobre el u ´ltimo20 teorema de Fermat

umeros c´ ubicos: El n´ umero21 1729 se puede expresar como la suma de dos n´ 1729 = 123 + 13 encuentre otro par de n´ umeros a, b tal que 1729 = a3 + b3 Sin embargo, 1729 no es un n´ umero c´ ubico. Naturalmente, podemos sumar un par de n´ umeros c´ ubicos cualesquiera intentando conseguir otro n´ umero c´ ubico; pero se ha demostrado que esta tarea es infructuosa. umeros enUn teorema famoso22 , debido a Fermat, asegura que no existen n´ teros a, b, c, que satisfagan: c3 = a 3 + b 3 Explore con algunos n´ umeros. Este teorema es uno de los m´as famosos de las matem´aticas; Fermat lo escribi´o en el margen del libro Aritm´etica, de Diofanto, sin demostraci´on. Pero sugiri´o una prueba23 para el caso espec´ıfico de n = 4, en un problema totalmente distinto usando un m´etodo de prueba por contradicci´on inventado por ´el, conocido como m´etodo de descenso infinito. Comenz´o por suponer que exist´ıa una soluci´on, digamos: x = x1, y = y1 , z = z1. Estudiando las propiedades de esta soluci´on demostr´o que si esta soluci´on umeros m´as fuera cierta deb´ıa existir otra soluci´on; x2, y2, z2 formada por n´ peque˜ nos; aplicando de nuevo el procedimiento debe existir otra soluci´on, x3, 20

Fermat, siendo abogado, acostumbraba escribir afirmaciones matem´ aticas sin dar las pruebas, como una manera de retar a los matem´aticos; con el tiempo, todas fueron demostradas; este teorema se le denomina el “´ ultimo”porque fue la u ´ltima de las afirmaciones que qued´ o por demostrar. Ver: SINGH, S.,op. cit. 21 El matem´ atico hind´ u, S. Ramanujan, descubri´ o que este n´ umero es el m´as peque˜ no que se puede escribir como la suma de dos cubos, de dos maneras diferentes. 22 El teorema de Fermat, en realidad, afirma que no es posible encontrar tres n´ umeros enteros x, y, z que satisfagan la igualdad xn + yn = z n , para n mayor que 2. 23 SINGH, S., El enigma de Fermat, Planeta, Barcelona, 1998, pp. 95-96.0

199

´ ´ ACTIVIDADES MATEMATICAS PARA EL DESARROLLO DE PROCESOS LOGICOS

y3 , z3 a´ un menor, y as´ı sucesivamente hasta el infinito. Pero como los n´ umeros naturales tienen un elemento que es el m´as peque˜ no de todos, el proceso no puede seguir indefinidamente, lo que implica que la soluci´on inicial no puede ser cierta. Ejercicios 1. La demostraci´on, para el caso de n = 4, prueba tambi´en los casos para n = 8, 12, 16, 20, etc. ¿Por qu´e? 2. En 1753, Euler aplic´o el m´etodo de descenso infinito de Fermat para demostrar el teorema en el caso n = 3, utilizando el n´ umero imaginario i; intent´o con otros valores de n, pero fracas´o. Si la prueba es v´alida para n = 3, tambi´en es v´alida para n = 6, 9, 12, 15, etc. Este caso es m´as importante puesto que el 3 es un n´ umero primo. Realmente, para probar el u ´ltimo teorema de Fermat con todos los valores de n, basta probarlo para los casos en que n sea un n´ umero primo. ¿Por qu´e? Sophie Germain, nacida el 1 de abril de 1776, plante´o a Gauss un m´etodo general de aproximaci´on al problema; su objetivo inmediato no consist´ıa en probar un caso concreto, sino en afirmar algo sobre muchos casos a la vez de un n´ umero primo p, tal que 2p + 1 tambi´en fuera primo. Para los valores de n equivalentes a estos primos de Germain, utiliz´o en argumento notable con el fin de mostrar que era muy probable que no hubiera soluciones a la ecuaci´on xn + y n = z n . Con “probable”, Germain quer´ıa decir que la existencia de soluciones era inveros´ımil porque, si las hubiera, x, y o z tendr´ıan que ser m´ ultiplos de n, y eso impondr´ıa fuertes restricciones a las soluciones posibles. En 1825, el m´etodo de Germain alcanz´o su primer ´exito pleno gracias a Gustav Lejeune-Dirichlet y a Adrien-Marie Legendre, para resolver el caso n = 5. En 1993, Andrew Wiles present´o una demostraci´on del u ´ltimo teorema de Fermat, que luego de ser examinada por los matem´aticos, present´o un error. Sin embargo, Wiles logr´o corregirlo y presentar la soluci´on definitiva, en 1995. 200

´ EN MATEMATICAS ´ LA INDUCCION

3. A pesar de que esta tarea no tiene soluci´on24, hay otra parecida que s´ı la tiene; es posible encontrar tres n´ umeros c´ ubicos cuya suma sea un n´ umero 25 c´ ubico , por ejemplo: 33 + 43 + 53 = 63 73 + 143 + 173 = 203 293 + 343 + 443 = 533 Escriba otros ejemplos. 4. Tambi´en 5 cubos sumados pueden dar un cubo, por ejemplo: 83 + 53 + 43 + 33 + 13 = 93 Escriba otros ejemplos. 5. Tambi´en 5 cubos sumados pueden dar un cubo, por ejemplo: 83 + 53 + 43 + 33 + 13 = 93 Escriba otros ejemplos. 6. Tambi´en hay en matem´aticas conjeturas famosas que luego de mucho tiempo han resultado falsas; tal vez el m´as c´elebre ejemplo sea la Conjetura de Euler, parecida al teorema de Fermat: “No existen soluciones enteras para la ecuaci´on x4 + y 4 + z 4 = w4 ”. 24

En los primeros intentos de demostraci´on, Leibniz resolvi´ o el caso n = 4 en 1678. El caso n = 3 fue resuelto por Euler y conten´ıa el germen de la teor´ıa de los ideales que Kummer aplic´ o en 1840. El caso n = 5 fue tratado por Legendre y Dirichlet, en 1825. El caso n = 7 fue demostrado por Lam´e y Lebesgue. 25 En general, si se tienen dos soluciones al problema x3 + y3 + z 3 = t3 y a3 + b3 + c3 = d3 , podemos encontrar infinitas, puesto que podemos encontrar una condici´ on sobre un n´ umero k para que (a + kx)3 + (b + ky)3 + (c + kz)3 = (d + kt)3 sea tambi´en una soluci´ on; para ello basta que: k = (t2 d − y2 b − z 2 c − x2 a)/(xa2 + yb2 + zc2 − td2 ) Este proceso, como muchos otros en matem´ aticas, se hace en reversa: se supone que (a + kx)3 + (b + ky)3 + (c + kz)3 = (d + kt)3 se cumple para alg´ un k y se despeja k.

201

´ ´ ACTIVIDADES MATEMATICAS PARA EL DESARROLLO DE PROCESOS LOGICOS

La conjetura dur´o 200 a˜ nos sin ser probada ni refutada, hasta que en 1988, Noam Elkies, de la Universidad de Harvard, la refut´o con el siguiente contraejemplo: 26824404 + 153656394 + 1879604 = 206156734 Adem´as, demostr´o que existen infinitas soluciones. Encuentre otra. ¿Se le ocurren algunas preguntas? ❆ ❆ ❆ ❆ ❆ ❆ ❆ ❆ ❆ ❆ ❆ ❆ ❆ ❆ ❆ ❆ ❆ Ya va siendo hora de desarrollar iniciativas propias. ❆ ❆ ❆ ❆ ❆ ❆ ❆ ❆ ❆ ❆ ❆ ❆ ❆ ❆ ❆ ❆ ❆

202

CAP´ITULO 7 ´ ´ EL METODO DE DEMOSTRACION ´ MATEMATICA ´ POR INDUCCION Introducci´ on El proceso intuitivo de inducci´on, que hemos presentado en el cap´ıtulo anterior y que es de com´ un uso en las ciencias de la naturaleza y en la l´ogica, consiste en la obtenci´on de una f´ormula o ley general a partir de algunos casos particulares; en ´el se razona desde lo particular hasta lo general. La base de este proceso es la suposici´on de que si algo es cierto en algunas ocasiones, tambi´en lo es en situaciones similares, aunque ´estas no se hayan observado. Las afirmaciones obtenidas mediante la inducci´on intuitiva no necesariamente son v´alidas para todos los casos; sin embargo, la probabilidad de acierto aumenta cuando el n´ umero de fen´omenos observados es mayor. Existen herramientas para estudiar la veracidad de las afirmaciones obtenidas por esta manera de inducci´on. Un ejemplo de ellas es el uso de m´etodos estad´ısticos para la selecci´on de muestras, que es el objeto de estudio de la estad´ıstica inductiva. Como en la mayor parte de los casos no se puede tener certeza absoluta, el estudio deber´a hacerse con base en la teor´ıa de probabilidades1 . 1

SPIEGEL, M., Teor´ıa y Problemas de Estad´ıstica., Mc Graw Hill., 1961.

203

´ ´ ACTIVIDADES MATEMATICAS PARA EL DESARROLLO DE PROCESOS LOGICOS

Un ejemplo com´ un de inducci´on intuitiva aparece en la expresi´on: “todos los hombres son iguales”, la cual es producto de algunas experiencias con unos pocos hombres (si tenemos en cuenta que todos los hombres pueden ser alrededor de tres mil millones). En general, las respuestas dadas por un peque˜ no grupo de personas en una encuesta de opini´on, se pueden proyectar para grupos m´as grandes usando los m´etodos de la estad´ıstica. El razonamiento inductivo fue desarrollado por varios fil´osofos, desde Francis Bacon hasta David Hume, John Stuart Mill y Charles Sanders Peirce (Studies in Philosophy). El proceso contrario, conocido como deducci´on, parte de una afirmaci´on general aceptada como v´alida para inferir la veracidad de un caso particular, y es el que m´as se utiliza en matem´aticas para demostrar teoremas. En los cap´ıtulos anteriores hemos tenido oportunidad de descubrir algunas relaciones entre los n´ umeros, usando el proceso intuitivo de inducci´on; ahora estamos interesados en demostrar la veracidad de tales conjeturas. Muchas veces, en el proceso del descubrimiento de tales conexiones, hemos utilizado gr´aficos, como en el caso de los n´ umeros poligonales, para construir ciertas secuencias o simplemente para examinar un problema concreto. Otras veces usamos listas o tablas de n´ umeros que nos permiten intuir alguna regularidad. Una vez obtenida la conjetura, que involucra un enunciado que sospechamos v´alido para cualquier escogencia de un n´ umero natural, ni los gr´aficos, ni las verificaciones para gran cantidad de casos, ni las mismas argumentaciones que surgieron al examinar el problema dan una prueba de que el enunciado es v´alido. La invalidez de los gr´aficos, como argumentos para demostrar una afirmaci´on, est´a basada en las limitaciones de nuestros sentidos y de los instrumentos de medida que utilizamos para describir una situaci´on. Para ilustrar este tipo de dificultades, observemos la figura 1, donde aparentemente disposiciones diferentes de los mismos objetos geom´etricos, numerados del 1 al 4, reportan a´reas diferentes. 204

´ ´ POR INDUCCION ´ MATEMATICA ´ EL METODO DE DEMOSTRACION

4

3

1

2

4

2

1 3 Figura 1 Tampoco las verificaciones, para un n´ umero grande de casos, nos garantiza la veracidad de una afirmaci´on intuida, pues ya vimos en el cap´ıtulo anterior que la f´ormula p = n2 − 79n + 1601 es v´alida para calcular n´ umeros primos, si n toma valores menores que 80, pero para valores de n mayores ya no es v´alida. La dificultad radica en que una afirmaci´on en la que est´en involucrados los n´ umeros naturales, debe ser cierta para todos ellos y puesto que ´estos son infinitos, no podemos comprobarla en todos los casos. Las argumentaciones que surgieron al examinar el problema, tampoco son aceptadas como prueba de una afirmaci´on donde est´en involucrados todos los n´ umeros naturales. Por ejemplo, cuando nos preguntamos por el n´ umero de cuadrados que se pueden distinguir en un cuadrado de lado n (figura 4, secci´on 5.7), observamos que: 1. Podemos ubicar un cuadrado de lado n. 2. De manera similar a la anterior podemos encontrar 4 cuadrados de lado n − 1, pues s´olo hay cuatro posiciones distintas posibles de un cuadrado de lado n − 1, dentro de un cuadrado de lado n. 3. Tambi´en hay 9 maneras distintas de colocar un cuadrado de lado n − 2 dentro de un cuadrado de lado n, y ya estamos listos para dar el salto: “Existen en total 12 + · · · + n2 cuadrados en un cuadrado de lado n” 205

´ ´ ACTIVIDADES MATEMATICAS PARA EL DESARROLLO DE PROCESOS LOGICOS

Razonamientos como ´este, nos pueden convencer de la veracidad de la afirmaci´on, pero no son aceptados como pruebas en matem´aticas, porque no sabemos c´omo sean los cuadrados infinitamente grandes, o posiblemente, ni siquiera son cuadrados; adem´as, tampoco estamos de acuerdo con lo que queremos decir con la palabra infinito. En matem´aticas, para garantizar la veracidad de tales afirmaciones que involucran a todos los n´ umeros naturales, debe recurrirse a un m´etodo conocido como inducci´ on matem´ atica. Este m´etodo requiere una clara concepci´on de la idea de infinito y ella nos es un poco esquiva y dif´ıcil; por ello, necesitamos precisarla un poco, para mostrar enseguida en qu´e consiste la inducci´on matem´atica

7.1.

¿Qu´ e significa infinito?

Las concepciones que tenemos los seres humanos acerca del infinito son, en algunos casos, inocentes, intimidantes en otros, pero en el com´ un de las personas est´a asociada con una cantidad suficientemente grande. En particular, en la escuela primaria se discute si el n´ umero de gotas de agua que hay en el mar, el n´ umero de granos de arena de todas las playas del mundo, el n´ umero de estrellas en el cielo y, por supuesto, el n´ umero de a´tomos del universo son infinitos. Podr´ıamos pensar que el n´ umero de a´tomos del universo es el m´as grande de todos los que representen cosas que existen en la realidad; sin embargo, ´este no ser´ıa el n´ umero m´as grande para nuestro pensamiento; por ejemplo, a este n´ umero se le puede sumar otro tanto y queda m´as grande, lo podemos multiplicar por mil billones y el resultado es a´ un m´as grande; seguramente no exista en nuestro universo alg´ un conjunto que tenga ese n´ umero de elementos, pero en nuestra cabeza podemos seguir inventando n´ umeros cada vez m´as grandes. El n´ umero 10100 (un 1 seguido por 100 ceros) conocido como un googol, es un n´ umero realmente grande, pero 10googol , llamado un googol−plex es desmesuradamente m´as grande que un googol (recordemos los efectos de la potenciaci´on sobre los n´ umeros grandes); este n´ umero es un 1 seguido por un googol de ceros. 206

´ ´ POR INDUCCION ´ MATEMATICA ´ EL METODO DE DEMOSTRACION

No alcanzar´ıan los a´tomos del universo para escribir un cero de este n´ umero en cada uno de ellos. Pero a´ un podemos seguir, si elevamos un googol − plex al exponente de un googol − plex y repetimos el proceso un googol − plex de veces, obtendr´ıamos un n´ umero fant´asticamente m´as grande que toda cosa conocida o imaginable. Y aunque podemos seguir invent´andonos n´ umeros grandes, ninguno de ellos puede capturar la noci´on de infinito. En el tratamiento que hicimos de los n´ umeros grandes nos percatamos, que aunque esos n´ umeros s´ı son grandes, hay n´ umeros a´ un mayores y si la idea de infinito es “un n´ umero suficientemente grande”, los que corresponden a estas cantidades no pueden ser infinitos. El primer contacto de los antiguos con el infinito se dio estudiando el proceso para extraer ra´ıces cuadradas -atribuido inicialmente a Arquitas (428 - 365 a. C.), a Her´on de Alejandr´ıa 100 d. C. y posteriormente a Newton, pero que aparece en una tabla babil´onica2 -. Este es un proceso que, si se aplica para obtener la ra´ız cuadrada de 2, no tiene un u ´ltimo paso; pueden conseguirse mejores aproximaciones de manera indefinida, es un proceso infinito. Otro estudio famoso sobre el infinito en la antig¨ uedad lo presenta Zen´on de Elea3, en sus paradojas. Un ejemplo de ellas es: “Antes de que un objeto en movimiento pueda recorrer una distancia dada, debe recorrer la mitad de esa distancia, pero a´ un antes de que recorra ´esta, debe recorrer la mitad de esta o sea un cuarto de la inicial y antes el primer octavo y as´ı sucesivamente a trav´es de una sucesi´on infinita de subdivisiones. Un corredor que quiere iniciar su carrera debe realizar un numero infinito de etapas, sin ninguna primera en un tiempo finito, pero es imposible agotar una colecci´on infinita y por lo tanto, el comienzo del movimiento es imposible”. M´as tarde, Gauss afirmaba que “en matem´aticas la magnitud infinita nunca puede usarse como algo que realmente exista, sino que el infinito tan s´olo es una forma de hablar”. En teolog´ıa, por el contrario, “el infinito”no s´olo tiene existencia propia sino que tiene parte activa en la naturaleza; en ella se atribuye a Dios ser “infinitamente”bueno, justo y poderoso; el car´acter infinito y eterno de Dios no es s´olo 2 3

BOYER, C., Historia de la Matem´ atica, Alianza Universidad, Madrid, 1968, p. 52. Ibid., p. 109.

207

´ ´ ACTIVIDADES MATEMATICAS PARA EL DESARROLLO DE PROCESOS LOGICOS

una forma de hablar. Por supuesto, esta concepci´on implica que el infinito es u ´nico. El primer matem´atico que trat´o el infinito como un ente con existencia real fue George Cantor, quien, al generalizar la noci´on de n´ umero, desarroll´o el concepto de n´ umeros transfinitos y defini´o para ellos una aritm´etica. Prob´o que no s´olo existe un infinito, sino que hay infinitos tipos de infinitos4 . En la teor´ıa cantoriana del infinito, el n´ umero de elementos de un conjunto se llama cardinal del conjunto. Un conjunto A es finito si existe alg´ un n´ umero natural n, de manera que se pueda establecer una correspondencia biyectiva5 entre A y un conjunto de la forma: {1, 2, 3, . . . , n} En este caso se dice que el cardinal de A es n. De lo contrario, el conjunto es infinito. Si A es el conjunto de los n´ umeros naturales, al cardinal de A se le da el nombre de Aleph0 y se representa con el s´ımbolo ℵ0 . Este n´ umero es de otro mundo, de un mundo conocido como el de los n´ umeros transfinitos; es realmente el m´as peque˜ no de los n´ umeros transfinitos, pero para el sentido com´ un puede parecer gigantesco, ya que si lo dividimos a la mitad queda exactamente igual al original, pues a cada natural le podemos hacer corresponder un n´ umero par multiplic´andolo por 2, y esta es una correspondencia biyectiva. Esto significa que el n´ umero de n´ umeros naturales es el mismo que el de n´ umeros pares. ¡Qui´en lo creyera! A´ un m´as, si dividimos ℵ0 en 10.000 pedazos cada uno de ellos, es id´entico al original y no importa el n´ umero de pedazos en que lo dividamos; en cualquier caso seguir´ıa siendo igual al original. Cualquier n´ umero finito, por grande que sea, es demasiado peque˜ no frente al menor de los n´ umeros transfinitos ℵ0 . 4

˜ OZ, ´ J. M., Introducci´ MUN on a la teor´ıa de conjuntos, Departamento de Matem´ aticas y Estad´ıstica. Universidad Nacional, Bogot´ a, 1983, p.281. 5 A cada elemento del primer conjunto le corresponde un u ´nico elemento del segundo y cada elemento del segundo proviene mediante la correspondencia de un u ´nico elemento del primero.

208

´ ´ POR INDUCCION ´ MATEMATICA ´ EL METODO DE DEMOSTRACION

ℵ0 tampoco se inmuta por multiplicaciones con un n´ umero finito, pues multiplicarlo por cualquier n´ umero m es equivalente a considerar el n´ umero asociado a un conjunto que contenga m copias de los naturales, pero al hacerlo podemos definir una funci´on biyectiva entre el conjunto as´ı formado y el de los naturales, como lo ilustra la figura 2, en donde, por ejemplo, el punto identificado por 32 corresponde al n´ umero natural 3 en la segunda copia realizada. La funci´on asigna umero 1, a 12 el n´ umero 2 y en general al punto identificado por al punto 11 el n´ umero (p − 1)m + q. pq el n´

• m1   

• m2   







  

  

  

••• •••

• mn   





  

  

••• •••

• • • 31 • 21 • 11

• • • 32 • 22 • 12

• • • 33 • 23 • 13

• • • • •

• • • • •

••• ••• ••• ••• •••

• • • 3n • 2n • 1n

• • • • •

• • • • •

••• ••• ••• ••• •••

1

2

3





•••

n





•••

 

 

 

Figura 2

Incluso, si multiplicamos ℵ0 por s´ı mismo; es decir, reproducimos por cada n´ umero natural una copia del conjunto de los n´ umeros naturales y los pintamos en un arreglo rectangular, como lo muestra la figura 3, en donde sobre cada natural, por ejemplo, el 3, colocamos una copia de los n´ umeros naturales indicados por los puntos 13 , 23 , 33 , . . . n3 , . . . , el cardinal de este nuevo conjunto es nuevamente ℵ0 , por cuanto podemos definir una correspondencia biyectiva entre ellos y los n´ umeros naturales.

La figura 3 da una idea intuitiva de la manera como podr´ıa definirse esta umero 1, a 21 el 2, a correspondencia, asignando al punto identificado con 11 el n´ 12 el 3, a 13 el 4, y as´ı sucesivamente. 209

´ ´ ACTIVIDADES MATEMATICAS PARA EL DESARROLLO DE PROCESOS LOGICOS

  

  

• m1 • •   

• m2 • •   

  

  

  

• • •

• • •

• • •

  

  



  

••• ••• ••• •••

  

• • • mn   

  

  

• • •

• • •

  

  

••• ••• •••

• • • • 31 • 21 • 11

• • • • 32 • 22 • 12

• • • • 33 • 23 • 13

• • • • • •

• • • • • •

••• ••• ••• ••• ••• •••

• • • • 3n • 2n • 1n

• • • • • •

• • • • • •

••• ••• ••• ••• ••• •••

1

2

3





•••

•n





•••

  

  

  

Figura 3 Podemos resumir las anteriores relaciones entre ℵ0 y cualquier cardinal n de un conjuntos finito, as´ı: n + ℵ0 = ℵ0 ℵ0 − n = ℵ0 ℵ0 /n = ℵ0 ℵ0 + ℵ0 = ℵ0 n · ℵ0 = ℵ0 ℵ0 · ℵ0 = ℵ0 (ℵ0 )n = ℵ0 La operaci´on equivalente a la resta de ℵ0 consigo mismo presenta un comportamiento especial. Veamos: Si de los n´ umeros naturales eliminamos el subconjunto de los pares, el cardinal del conjunto resultante es tambi´en ℵ0 . Pero si de los naturales eliminamos todos los mayores que un cierto n´ umero n, el cardinal del conjunto resultante es n; por lo tanto, no hay manera de establecer un u ´nico resultado para ℵ0 − ℵ0 ; luego esta expresi´on no est´a definida. Si queremos hacer una analog´ıa para interpretar una divisi´on entre ℵ0 consigo mismo, consideremos disponibles tantas cajas como n´ umeros naturales. Podemos 210

´ ´ POR INDUCCION ´ MATEMATICA ´ EL METODO DE DEMOSTRACION

imaginar todas estas cajas como no vac´ıas colocando un n´ umero natural en cada caja, pero tambi´en podr´ıa hacerlo depositando los dos primeros en la primera caja, los dos siguientes en la segunda y as´ı sucesivamente, quedando exactamente dos n´ umeros en cada caja. Por supuesto, podr´ıa hacer una repartici´on parecida depositando k n´ umeros naturales en cada caja. Incluso mediante una distribuci´on adecuada podr´ıan asociarse a cada caja infinitos n´ umeros naturales. No podemos, por tanto, asignar un u´nico valor al posible resultado de efectuar ℵ0 /ℵ0 . Esta expresi´on tampoco est´a definida. La potenciaci´on, en cambio, s´ı provoca un cambio sustancial a Aleph0 , ¡otra vez la potenciaci´on!, si elevamos ℵ0 al exponente ℵ0 (bastar´ıa incluso elevar 2, o umero infinito llamado cualquier n´ umero finito al exponente ℵ0 ), obtenemos otro n´ umero de puntos que Aleph1 representado por el s´ımbolo ℵ1 . Este representa el n´ hay en un segmento de l´ınea recta o curva; es el mismo n´ umero de puntos de toda la recta. Adem´as, corresponde al n´ umero de puntos de un cuadrado de cualquier tama˜ no y al n´ umero de puntos de todo el espacio. Esto u ´ltimo es un golpe fuerte a la intuici´on, ¿no le parece? ¡Pero ya deber´ıamos irnos acostumbrando a perder la verg¨ uenza! Si reincidimos y elevamos Aleph1 al exponente Aleph1, obtenemos Aleph2 , que corresponde al n´ umero de curvas6 que se pueden trazar en un plano. As´ı sucesivamente podemos construir infinitos n´ umeros infinitos y con ellos hacer una Aritm´etica parecida a la que conocemos, que se conoce como La Aritm´etica ordinal transfinita de Georg Cantor7 .

7.2.

El m´ etodo de demostraci´ on por Inducci´ on Matem´ atica

Cuando escribimos una f´ormula donde aparecen n´ umeros naturales, como por ejemplo: n(n + 1) 1+ 2 + 3+ ··· + n = 2 6 7

NAVARRO, J., La nueva matem´ atica, Salvat, Barcelona, 1973. SUPPES,P., Teor´ıa axiom´ atica de conjuntos, Norma, Cali, 1976, pp. 123-149.

211

´ ´ ACTIVIDADES MATEMATICAS PARA EL DESARROLLO DE PROCESOS LOGICOS

quiz´a no nos damos cuenta del profundo significado que ella tiene. A un humano corriente, no interesado por las mieles intelectuales del razonamiento matem´atico, le ocupar´a bastante tiempo, en caso de serle estrictamente necesario, sumar los primeros 200 n´ umeros naturales; a nosotros, en cambio, que ya descubrimos que basta multiplicar 201 por 100, o sea 20100, nos llevar´ıa 15 segundos a lo m´as. ¡Hemos ganado! Si nos retan, podr´ıamos hasta sumar el primer googol de n´ umeros naturales en muy poco tiempo; pero la f´ormula dice m´as, ella debe valer para cualquier n´ umero natural n y no podemos comprobarla para todos. Hay veces, sin embargo, en que las f´ormulas se cumplen s´olo para algunos valores de n y no para otros, como ya lo hemos visto para los n´ umeros primos. De nuevo la pregunta es: ¿c´omo aseguramos que una f´ormula se vale para todo n´ umero natural si no podemos ensayar con todos? Los matem´aticos han asumido un principio llamado principio de inducci´on matem´ atica, que act´ ua como garante en este caso y que est´a basado en el contagio propio de una hilera de fichas de domin´o, colocadas cada una a corta distancia de la otra, de manera que cuando se empuja una de ellas, ella afecta a la siguiente y ´esta, a su vez, a la siguiente, y as´ı sucesivamente. Fueron Pascal y Fermat, quienes desarrollaron primero el razonamiento por inducci´on. En 1654, Pascal dio una exposici´on clara y precisa del m´etodo de inducci´on completa, aunque ya hab´ıa ciertas indicaciones en la obra de Maurolico. El nombre de inducci´on matem´atica tuvo su origen m´as tarde, en el art´ıculo de De Morgan, en la Penny Cyclopaedia de 1838. En 1889, Guiseppe Peano escogi´o el principio de inducci´on como uno de los axiomas para fundamentar la aritm´etica con un simbolismo formalizado (con s´ımbolos formales y no con el lenguaje cotidiano) en su obra Arithmetices Principia nova methodo exposita. El principio de inducci´on matem´atica establece que: Una proposici´on matem´atica p(k) donde est´e involucrado un n´ umero natural k es v´alida para todo n´ umero natural n si: 212

´ ´ POR INDUCCION ´ MATEMATICA ´ EL METODO DE DEMOSTRACION

i. Se cumple para el n´ umero 0. ii. Si se cumple para n debe cumplirse para el siguiente de n, es decir (n + 1) La condici´on (ii) afirma que la verdad de p para el valor (n + 1) depende de la verdad de p para el valor n. Esto basta para asegurar la verdad de p para cualquier valor de n si se cumple la condici´on (i), pues si p es cierta para el valor n = 1, entonces es cierta para n = 2; y como es cierta para n = 2 entonces, es cierta para n = 3, y as´ı sucesivamente. Naturalmente, todo n´ umero ser´a alcanzado alguna vez mediante una serie de etapas de esta clase, de manera que p ser´a verdad para todos los n´ umero n. En el caso de la f´ormula para la suma de los n primeros n´ umeros naturales, obviamente se tiene para 1, y si suponemos que se cumple para un determinado k; es decir, suponemos que:

1+ 2 + 3+ ··· + k =

k(k + 1) 2

sumando (k + 1) en ambos lados de la igualdad obtenemos: 1 + 2 + 3 + · · · + k + (k + 1) =

=

k(k + 1) + (k + 1) 2

(k + 1)(k + 2) 2

que es la f´ormula correspondiente para (k + 1), de donde podemos concluir que es v´alida para todo n´ umero natural n. Observemos que una demostraci´on que utiliza el principio de inducci´on matem´atica no tiene relaci´on expl´ıcita con el proceso intuitivo de inducci´on mediante el cual se descubre la f´ormula. El principio de inducci´on matem´atica tiene varias formas equivalentes, por ejemplo: 213

´ ´ ACTIVIDADES MATEMATICAS PARA EL DESARROLLO DE PROCESOS LOGICOS

1. Si A es una colecci´on (conjunto) de n´ umeros naturales y i. 1 pertenece a A, ii. k + 1 pertenece a A siempre que k pertenezca a A Entonces, A es el conjunto de todos los n´ umeros naturales8 . 2. A primera vista parece cierto que si A es una colecci´on cualquiera de n´ umeros naturales, en A debe haber un elemento m´as peque˜ no que todos los dem´as. Si A es el conjunto vac´ıo (notado ∅) esta afirmaci´on no es cierta, puesto que el conjunto vac´ıo es una colecci´on de n´ umeros naturales que no tiene elementos y por lo tanto no tiene elemento m´ınimo. Pero si A es un conjunto no vac´ıo de n´ umeros naturales, entonces A tiene un elemento m´ınimo. Esta afirmaci´on se conoce como principio de buena ordenaci´ on. Este principio se puede demostrar por inducci´on, como sigue: Sup´ongase que el conjunto A es no vac´ıo y no tiene elemento m´ınimo. Sea B el conjunto de los n´ umeros naturales n tales que 1, 2, 3, 4, 5, . . . , n no est´e ninguno en A. Por supuesto que 1 est´a en B, pues si 1 estuviese en A entonces A tendr´ıa a 1 como elemento m´ınimo. Adem´as, si 1, 2, 3, 4, 5, . . . , k no est´an en A, evidentemente k+1 no est´a en A (si no, k+1 ser´ıa un elemento m´ınimo de A), de manera que 1, 2, 3, 4, 5, . . . , k+ 1 no est´an en A. Esto demuestra que si k est´a en B, entonces k + 1 est´a en B. Entonces todo n´ umero n est´a en B; es decir, los n´ umeros 1, 2, 3, 4, 5, . . . , n no est´an en A cualquiera que sea el n´ umero natural n. As´ı pues, A = ∅, pero esto no puede ser porque hab´ıamos supuesto lo contrario, con lo que se concluye la demostraci´on. 8

SPIVAK, M.; C´ alculo infinitesimal, Revert´e, Barcelona, 1978, p. 29.

214

´ ´ POR INDUCCION ´ MATEMATICA ´ EL METODO DE DEMOSTRACION

Ejercicio Consulte y estudie9 una demostraci´on del principio de inducci´ on matem´ atica tomando como cierto el principio de buena ordenaci´ on. 3. Ocurre a veces que para demostrar que una proposici´on p se cumple para el valor k + 1, debemos suponer que p no s´olo es v´alida para el valor k, sino para todos los valores menores o iguales a k. Este principio se conoce como el principio de inducci´on completa, que establece: Si A es un conjunto de n´ umeros naturales y (1) 1 est´a en A (2) k + 1 est´a en A si 1, 2, 3, 4, . . . , k est´an en A entonces A es el conjunto de todos los n´ umeros naturales. Aunque el principio de inducci´on completa puede parecer m´as fuerte que el principio de inducci´on ordinario, en realidad no es sino una consecuencia de este u ´ltimo. Ejemplos Presentamos enseguida otras demostraciones por inducci´on, que no pretenden mostrar todos los casos, sino mejorar la percepci´on sobre este m´etodo de razonamiento. Comenzamos con algunas de las afirmaciones que hicimos en los cap´ıtulos anteriores y que requieren nuestra justificaci´on. 1. Cuando estudiamos la divisibilidad por 2, afirmamos que 3k − 1 es par para toda escogencia de k dentro de los n´ umeros naturales. Para demostrar esta afirmaci´on, debemos verificar que se cumple al reemplazar k por 1, como en efecto sucede. Enseguida, debemos suponer que la afirmaci´on es cierta para el valor k, y haciendo uso de esto, construimos la afirmaci´on para el valor k + 1. 9

Ap´ ostol, T., Calculus., Revert´e, Vol. I., 1988, p. 42.

215

´ ´ ACTIVIDADES MATEMATICAS PARA EL DESARROLLO DE PROCESOS LOGICOS

Comenzamos, sin embargo, construyendo la afirmaci´on para el valor k + 1, esto es: 3k+1 − 1

es par

pero 3k+1 − 1 = 3 · 3k − 1 = 2 · 3k + (3k − 1) El primer sumando es par, pues es m´ ultiplo de 2 y el segundo es par, porque as´ı lo hemos supuesto; esta es la hip´otesis de inducci´on. Con esto queda demostrado el teorema. umero natural n. 2. 5k − 2k es divisible por 3, para todo n´ Si n = 1 la igualdad se cumple ya que 51 − 21 = 3, que es divisible por 3. Supongamos que la igualdad es verdadera para n = k , esto es, existe un n´ umero q tal que 5k − 2k = 3q y probemos que se tiene para n = k + 1 5k+1 − 2k+1 = 5 · 5k − 2 · 2k = (3 + 2) · 5k − 2 · 2k = 3 · 5k + 2 · 5k − 2 · 2k = 3 · 5k + 2 · (5k − 2k ) = 3 · 5k + 2 · (3q) Notemos que en este paso hemos usado la hip´otesis de inducci´on, finalmente: = 3 · (5k + 2q) lo que implica que 5k+1 − 2k+1 es un m´ ultiplo de 3 y con esto el teorema queda demostrado. ultiplo de 3 y que 10k − 4 es An´alogamente, se prueba que 4k − 1 es m´ divisible por seis, para todo n´ umero natural k. 216

´ ´ POR INDUCCION ´ MATEMATICA ´ EL METODO DE DEMOSTRACION

❆ ❆ ❆ ❆ ❆ ❆ ❆ ❆ ❆ ❆ ❆ ❆ ❆ ❆ ❆ ❆ ❆ Por supuesto, usted ya no espera ´ordenes. ❆ ❆ ❆ ❆ ❆ ❆ ❆ ❆ ❆ ❆ ❆ ❆ ❆ ❆ ❆ ❆ ❆ 3. Si miramos los ejemplos anteriores, podemos sospechar (inducci´on intuitiva) que si10 a > b entonces an −bn es divisible por a−b para todo valor de n natural. Haremos enseguida una demostraci´on por inducci´on matem´atica: Si n = 1 se cumple ya que a1 − b1 = a − b. Supongamos que es verdad para n = k , esto es, existe un n´ umero p tal que ak − bk = (a − b)p y probemos que se tiene para n = k + 1 ak+1 − bk+1 = a · ak − bbk Como a > b entonces existe un n´ umero natural c tal que a = c + b; por tanto: ak+1 − bk+1 = (c + b)ak − bbk = cak + bak − bbk = cak + b(ak − bk ) = cak + b(a − b)p Pero sabemos que c = a − b, luego: ak+1 − bk+1 = (a − b)ak + b(a − b)p = (a − b)(ak + bp) = (a − b)q donde q = ak + bp, y con esto se concluye la demostraci´on. 10

Esta condici´on s´ olo es para garantizar que estamos tratando con n´ umeros naturales; sin embargo, la proposici´ on es v´alida en conjuntos de n´ umeros m´as generales.

217

´ ´ ACTIVIDADES MATEMATICAS PARA EL DESARROLLO DE PROCESOS LOGICOS

4. El n´ umero de subconjuntos de un conjunto de n elementos es 2n . F´acilmente se verifica que para un conjunto con 1 elemento, el n´ umero de sus subconjuntos es 2, el conjunto vac´ıo y el mismo conjunto. Si suponemos cierta la afirmaci´on para n elementos, debemos probar que es cierta para n + 1 elementos. Si un conjunto tiene n+1 elementos, tiene un elemento m´as que el conjunto de n elementos y como este tiene 2n subconjuntos, podemos formar 2n nuevos subconjuntos agregando el nuevo elemento a cada uno de los que ya estaban formados; es decir, en total tenemos 2n + 2n elementos, o sea 2n+1 elementos. umeros naturales11 n y 5. La desigualdad (1 + x)n ≥ 1 + nx para cualesquier n´ 12 x, conocida como desigualdad de Bernoulli , tiene la siguiente prueba por inducci´on sobre n: Claramente se tiene para n = 1 (1 + x)1 ≥ 1 + 1x Si suponemos que la f´ormula es v´alida para n = k, cuando n = k + 1, tenemos que: (1 + x)k+1 = (1 + x)k (1 + x) Utilizando la hip´otesis de inducci´on (1 + x)k+1 ≥ (1 + kx)(1 + x) = 1 + kx + x + kx2 = 1 + (k + 1)x + kx2 11

La desigualdad tambi´en es v´alida s´ı x no es un n´ umero natural, si est´ a en el campo m´as amplio de los n´ umeros reales. 12 Esta desigualdad fue publicada por ´el en 1689, pero ya se encontraba en una publicaci´ on de Isaac Barrow, maestro de Newton, en 1670.

218

´ ´ POR INDUCCION ´ MATEMATICA ´ EL METODO DE DEMOSTRACION

y como kx2 es un n´ umero natural, entonces (1 + x)k+1 ≥ 1 + (k + 1)x porque si a un n´ umero natural escrito como una suma (1 + (k + 1)x + kx2 ) no. Y con le quitamos uno de los sumandos kx2 , el resultado es m´as peque˜ esto finaliza la demostraci´on. 6. En este ejemplo hacemos uso del principio de inducci´on completa para demostrar que “todo n´ umero natural mayor o igual que 2 o es primo o se puede escribir como un producto de n´ umeros primos”. Si n = 2, la proposici´on se tiene. Si n = k, supongamos que la afirmaci´on es v´alida para todo n´ umero natural menor o igual que k y mayor o igual que 2. Si n = k + 1, entonces: a) Si k + 1 es primo, la afirmaci´on ser´ıa v´alida. b) Si k + 1 no es primo, k + 1 tiene un divisor m diferente de 1 y de s´ı mismo, es decir existe un n´ umero z tal que k + 1 = mz donde 1 < m ≤ k y 1 < z ≤ k. Ahora, aplicando la hip´otesis de inducci´on a los n´ umeros m y z, ambos son producto de n´ umeros primos y por tanto k + 1 es producto de n´ umeros primos. 7. El teorema del binomio de Newton13 afirma que:     n n−0 0 n n−1 1 n n n−n n n n−(n−1) n−1 a b + a b +· · ·+ a b + a b (a+b) = 0 1 n−1 n Su demostraci´on por inducci´on tiene a primera vista un aspecto impresionante, por lo que su lectura requiere la paciencia que es natural en los matem´aticos. 13

Teniendo como base que

n! = (n − k)!k!

 n k

219

´ ´ ACTIVIDADES MATEMATICAS PARA EL DESARROLLO DE PROCESOS LOGICOS

❆ ❆ ❆ ❆ ❆ ❆ ❆ ❆ ❆ ❆ ❆ ❆ ❆ ❆ ❆ ❆ ❆ Si a´ un no la ha desarrollado suficientemente tenga paciencia, que si se esfuerza, ya vendr´a. ❆ ❆ ❆ ❆ ❆ ❆ ❆ ❆ ❆ ❆ ❆ ❆ ❆ ❆ ❆ ❆ ❆   1 1 Para el caso n = 1, la f´ormula se cumple puesto que a + b = a+ b. 0 1 Supongamos que es v´alida para n = k. Si n = k + 1: (a + b)k+1 = (a + b)(a + b)k = a(a + b)k + b(a + b)k =     k k−1 1 k k−2 2 k k k−0 0 a b + a b + ··· + ak−(k−1) bk−1 a a b + 1 2 k−1 0  k k−k k + a b k     k k−0 0 k k−1 1 k k−2 2 k b a b + a b + a b + ··· + ak−(k−1) bk−1 0 1 2 k−1  k k−k k + a b = k     k k+1−1 1 k k+1−2 2 k k k+1−0 0 b + a b + a b + ··· + ak+1−(k−1) bk−1 a 1 2 k−1 0     k k+1−k k k k−1 1+1 k k−2 2+1 k k−0 0 + a b + a b + a b a b + k 1 2 0   k k k−k 1+k k−(k−1) k+1−1 + ··· + a b + a b k−1 k

Teniendo en cuenta que: 

  k+1 n k = + k k−1 k

(La anterior igualdad queda como ejercicio para el lector) Agrupando los t´erminos que tengan los mismos exponentes para a y para 220

´ ´ POR INDUCCION ´ MATEMATICA ´ EL METODO DE DEMOSTRACION

b, concluimos que:    k + 1 k+1−0 0 k + 1 k+1−1 1 k + 1 k+1−2 2 k+1 = a b + a b + a b + ··· (a + b) 0 1 2   k + 1 0 k+1 k + 1 k−(k−1) k+1−1 b + ab + a k+1 k y por lo tanto la f´ormula es v´alida para todo n´ umero natural n. 8. Consideremos ahora expresiones en las que aparecen dos o m´as n´ umeros naturales y en las que se afirma que son v´alidas para todo n´ umero natural; en ellas debemos hacer una demostraci´on por inducci´on sobre cada variable. Por ejemplo, cuando estudiamos la divisibilidad por 2 en cualquier base, afirmamos que un n´ umero escrito en base impar es divisible por 2 si la suma de sus cifras es par, esta afirmaci´on es cierta si lo es la siguiente: (2n + 1)k − 1

es par.

El problema es equivalente a probar que (2n + 1)k es impar para cualquier escogencia de los n´ umeros n y k. Para probar esta afirmaci´on por inducci´on, consideramos primero el caso n = 1 y en ´el debemos probar que para todo n´ umero natural k, 3k es un n´ umero impar. Pero esta tarea ya fue realizada en el ejemplo 1 de esta secci´on. Suponemos ahora que la afirmaci´on es v´alida para un valor fijo de n = s. Esto es, para todo n´ umero natural k, se cumple que (2s + 1)k es un n´ umero impar y debemos probar que para n = s + 1 se cumple que [2(s + 1) + 1]k = umero impar para cualquier valor del n´ umero natural k y (2s + 3)k es un n´ para ello nuevamente necesitamos hacer inducci´on sobre la variable k. Para k = 1, la proposici´on es v´alida. Suponemos que para k = t, (2s + 3)t es un n´ umero impar y como (2s + umeros impares concluimos 3)t+1 = (2s + 3)t (2s + 3), es el producto de dos n´ que es tambi´en un n´ umero impar. 221

´ ´ ACTIVIDADES MATEMATICAS PARA EL DESARROLLO DE PROCESOS LOGICOS

Estas demostraciones, conocidas como inducci´ on doble 14 , admiten una manera alternativa que consiste en fijar primero una variable y hacer inducci´on sobre la otra y viceversa. Primero hacemos inducci´on sobre k, luego sobre n. Consideremos para ello n un n´ umero natural fijo. Observemos inicialmente que para k = 1 se tiene que (2n + 1)k = 2n + 1 es un n´ umero impar. Supongamos que la afirmaci´on se tiene para k = s; es decir, existe Q tal que (2n + 1)s = 2Q + 1 y probemos que se tiene para k = s + 1. (2n + 1)s+1 = (2n + 1)(2n + 1)s = (2n + 1)(2Q + 1) = 4Qn + 2Q + 2n + 1 = 2(2Qn + Q + n) + 1 = 2q + 1 tomando como q = 2Qn + p + n. Luego, habiendo fijado n, la expresi´on es v´alida para todo n´ umero natural k. Ahora debemos hacer inducci´on sobre n, dejando fijo el exponente k, y esta tarea se la dejamos al lector. Un asunto que debemos resaltar es que no todas las f´ormulas que incluyen n´ umeros naturales se prueban por inducci´on; es necesario que la f´ormula se refiera a todos los n´ umeros naturales. Por ejemplo, la afirmaci´on “Un n´ umero natural es perfecto si es de la forma 2n−1 (2n − 1) umero primo cuando n es primo” donde 2n − 1 es un n´ no es demostrable por inducci´on matem´atica, pues a pesar de que el n´ umero n es natural, en esta afirmaci´on debe ser primo y no todos los naturales lo son. 14

HALMOS, P.; Naive Set Theory, Van Nostrand, Nueva York, 1960, p. 46.

222

´ ´ POR INDUCCION ´ MATEMATICA ´ EL METODO DE DEMOSTRACION

7.3.

Definiciones por recurrencia

Otra de las ventajas que ofrece el principio de inducci´on matem´atica se encuentra en la posibilidad de utilizarla en la obtenci´on de una nueva forma para definir objetos matem´aticos. Estas definiciones, conocidas como definiciones por recurrencia, se encuentran de manera abundante en las matem´aticas y consisten en distinguir un elemento de un conjunto e indicar la manera de conseguir un nuevo objeto a partir de otros previamente construidos. En ella se est´a contagiando la esencia misma de los n´ umeros naturales a otros conjuntos; ´esta es que cada elemento tiene un sucesor. Veamos algunos ejemplos: 1. La definici´on del factorial de un n´ umero n, el cual se denota n, se describe por recurrencia, as´ı: 0! = 1 (n + 1)! = n! · (n + 1) 2. La potenciaci´on tambi´en puede ser definida por recurrencia as´ı: y a0 = 1 n+1 n =a ·a a 3. No necesariamente se define un nuevo objeto a partir del anterior, sino a la manera de lo presentado en la inducci´on completa. Puede presentarse a partir de varios o todos los anteriores. Por ejemplo, una sucesi´on relacionada con la reproducci´on de los conejos, con la forma de distribuci´on de las ramas en los a´rboles, y de las semillas en la flor del girasol, conocida como sucesi´ on de Fibonacci tiene su presentaci´on por recurrencia, como: a1 = 1 a2 = 1 an+1 = an + an−1 . 223

´ ´ ACTIVIDADES MATEMATICAS PARA EL DESARROLLO DE PROCESOS LOGICOS

Ejercicio Demuestre por inducci´ on que las f´ormulas que le presentamos a continuaci´on son v´alidas para todo n´ umero natural n: 1. La suma de los primeros n n´ umeros pares es: 2 + 4 + 6 + · · · + 2n = n(n + 1) La suma de los primeros n n´ umeros que se obtienen a partir de uno y contando de tres en tres, es: 1 + 4 + 7 + · · · + (3n − 2) =

n (3n − 1) 2

Las dos f´ormulas anteriores son casos particulares de la siguiente relaci´on, donde partiendo de a se cuenta de x en x. a + (a + x) + (a + 2x) + · · · + (a + (n − 1)x) =

n (2a + (n − 1)x) 2

2. Las sumas de los primeros n n´ umeros cuadrados, los primeros n n´ umeros cuadrados impares y los n primeros n´ umeros c´ ubicos impares, est´an dadas respectivamente por: n(n + 1)(2n + 1) 6 n 2 2 2 2 1 + 3 + 5 + · · · + (2n − 1) = (4n2 − 1) 3 13 + 33 + 53 + · · · + (2n − 1)3 = n2 (2n2 − 1) 12 + 22 + 32 + · · · + n2 =

3. Las dos f´ormulas siguientes establecen una relaci´on entre la suma de los primeros n n´ umeros c´ ubicos y el n-´esimo n´ umero triangular: n2 1 + 2 + 3 + · · · + n = (n + 1)2 4 2 n (1 + 2 + 3 + · · · + n)2 = (n + 1)2 4 3

3

3

3

224

´ ´ POR INDUCCION ´ MATEMATICA ´ EL METODO DE DEMOSTRACION

4. Los tres siguientes ejercicios tienen algo en com´ un: 2 + 22 + 23 + · · · + 2n = 2(2n − 1) 3 3 + 32 + 33 + · · · + 3n = (3n − 1) 2 5 5 + 52 + 53 + · · · + 5n = (5n − 1) 4 Encuentre una f´ ormula general que los describa y pru´ebela tambi´en. 5. La siguiente suma aparece al hacer la interpretaci´on moderna de la paradoja de Zen´on: 1+

1 1 1 1 + 2 + · · · + n−1 = 2 − + n−1 2 2 2 2

y es un caso particular de: a + ar + ar2 + · · · + arn−1 =

a(1 − rn ) , r = 1 1−r

6. Hay f´ormulas que no son v´alidas para todo n´ umero natural, pero s´ı para todo natural mayor que un n´ umero dado. Estas f´ormulas pueden escribirse como v´alidas para todo n´ umero natural introduciendo una variable distinta. Por ejemplo, 2n ≥ n2 es una desigualdad v´alida a partir de 4, y es equivalente a garantizar que si n = 3 + m la expresi´on 2m+3 ≥ (m + 3)2 es v´alida para todo n´ umero natural m. Sin embargo, sin necesidad de hacer el cambio de variable, la prueba por inducci´on se reduce a garantizar que la f´ormula dada es v´alida para n = 5 y continuar el proceso de manera similar al anterior. Demuestre que: 2n ≥ n2 para n > 3. 2n ≤ n! para n > 3. 4n ≥ n 3n para n > 6. 225

´ ´ ACTIVIDADES MATEMATICAS PARA EL DESARROLLO DE PROCESOS LOGICOS

7. Use el m´etodo de inducci´on matem´atica para demostrar que los coeficientes  n binomiales son n´ umeros naturales para todo k ≥ n. ¿Se requiere usar k inducci´ on completa?;¿inducci´on doble? Justifique. 8. Las siguientes proposiciones requieren de inducci´on doble: an+m = an am (an )m = anm Toda potencia de un n´ umero par es par. Realice las demostraciones.

7.4.

Presentaci´ on axiom´ atica de los n´ umeros naturales

Hemos insistido en que las intuiciones que resultan de la experiencia directa usando nuestros sentidos, o las obtenidas de observar regularidades, por m´as que ´estas se presenten muchas veces, no son suficientes para dar validez a una afirmaci´on matem´atica; esto hace necesario que fijemos un criterio para decidir sobre la validez de ellas. Existe un m´etodo regularmente aceptado entre los matem´aticos desde hace muchos a˜ nos, conocido como m´etodo axiom´atico. Euclides, en el a˜ no 300 a.C, e incluso antes Eudoxo, presentaron la geometr´ıa como un conjunto de proposiciones que se deducen de unas fundamentales que llamamos axiomas, mediante una forma preestablecida de razonar. Arqu´ımedes hizo lo mismo con la mec´anica te´orica que luego perfeccionan Newton en su Principia Matem´atica de 1686, y Lagrange, en su Mec´ anica anal´ıtica de 1788. La presentaci´on axiom´atica de la geometr´ıa en su forma moderna se debe a Hilbert en su libro Grundlagen der Geometrie de 1899. En estas presentaciones axiom´aticas se parte de unos t´erminos no definidos, se enuncian unas relaciones entre ellos, que aceptamos como ciertas (los Axiomas), se presume una forma correcta de razonar, usualmente la l´ogica bivalente cl´asica, y con esto se deducen otras afirmaciones que llamamos teoremas. Los Teoremas 226

´ ´ POR INDUCCION ´ MATEMATICA ´ EL METODO DE DEMOSTRACION

son ciertos en la medida de que los axiomas lo sean y que los razonamientos sean correctos. En el sistema axiom´atico, para los n´ umeros naturales, presentado por el italiano Guiusseppe Peano, en 1889, figuran como t´erminos primitivos el n´ umero 0 y el concepto de sucesor y como axiomas: 1. 0 es un n´ umero natural15 2. El sucesor de cualquier n´ umero natural (n) es un n´ umero natural (n+ ) 3. Dos n´ umeros naturales diferentes no tienen nunca el mismo sucesor; es decir, que si k = n entonces k + = n+ . 4. 0 no es el sucesor de alg´ un n´ umero. (0 es el primer n´ umero natural16 ) 5. Si P es una propiedad tal que: a) 0 tiene la propiedad P . b) Siempre que un n´ umero n tiene la propiedad P implica que su sucesor umero natural tiene n+ tambi´en tiene la propiedad P , entonces todo n´ la propiedad P . El axioma 5 es el que da sustento l´ogico al m´etodo de inducci´on matem´atica que hemos utilizado para probar las regularidades encontradas al trabajar con los n´ umeros naturales.

7.4.1.

La adici´ on de n´ umeros naturales

La operaci´on adici´on de n´ umeros naturales se define por recurrencia 17 de la siguiente forma: i. n + 0 = n ii. n + k + = (n + k)+ . 15

En la presentaci´ on inicial de Peano, el primer n´ umero es el 1, pero esto no cambia sustancialmente el el sistema. 16 Hay versiones axiom´aticas de los n´ umeros naturales donde el primer n´ umero natural es el 1. 17 NEWMAN, J.; op. cit., Vol. V., pp. 7- 22.

227

´ ´ ACTIVIDADES MATEMATICAS PARA EL DESARROLLO DE PROCESOS LOGICOS

Para cualesquier n´ umero naturales n y k. Con esto y la definici´on de multiplicaci´on, que presentamos un poco m´as adelante, tenemos las bases para deducir todas las propiedades que rigen al sistema de los n´ umeros naturales. Un tratamiento completo puede encontrarse en un libro de teor´ıa de conjunumeros19 . tos , o en un libro de teor´ıa de n´ 18

Veamos, a manera de ejemplo, la demostraci´on de las propiedades fundamentales de los n´ umeros naturales. ❆ ❆ ❆ ❆ ❆ ❆ ❆ ❆ ❆ ❆ ❆ ❆ ❆ ❆ ❆ ❆ ❆ ❆ Observe la diferencia en la manera de pensar con respecto a lo hecho anteriormente. ❆ ❆ ❆ ❆ ❆ ❆ ❆ ❆ ❆ ❆ ❆ ❆ ❆ ❆ ❆ ❆ ❆ ❆ 7.4.1.1.

Propiedad modulativa de la adici´ on

Para todo n´ umero natural se cumple que n+0= 0+n = n Prueba: De la definici´on de suma se tiene que n+0 = n. Demostraremos por inducci´on que para todo n, se tiene que 0 + n = n. i. Para n = 0, tenemos que 0 + 0 = 0, por la definici´on. ii. Suponemos v´alido que 0 + k = k, siendo k un n´ umero natural y debe+ mos demostrar que se cumple para su sucesor k . Pero esto tambi´en es inmediato de la segunda parte de la definici´on de la suma puesto que 0 + k + = (0 + k)+ = k + . Por lo tanto, la afirmaci´on es v´alida para todo n´ umero natural n. 18

˜ MUNOZ, J., Introducci´ on a la teor´ıa de conjuntos, Universidad Nacional, Bogot´ a, 1983, p.p. 141-185. 19 ´ RUBIANO, G., GORDILLO, J., JIMENEZ, R., Teor´ıa de n´ umeros para principiantes, Universidad Nacional, Bogot´ a, 1999, pp. 1-9.

228

´ ´ POR INDUCCION ´ MATEMATICA ´ EL METODO DE DEMOSTRACION

7.4.1.2.

Propiedad conmutativa de la adici´ on

Para hacer una demostraci´on de la propiedad conmutativa de la adici´on, probaremos primero que para todo n y k n´ umeros naturales se cumple que: k + + n = (k + n)+ Prueba: Hacemos inducci´on sobre n. Para n = 0 k + + 0 = (k + 0)+ por la propiedad modulativa de la adici´on. Supongamos que la igualdad es v´alida para n = m y demostr´emosla para m+ ; es decir, debemos probar que k + + m+ = (k + m+ )+ . Partamos de: k + + m+ = (k + + m)+ = ((k + m)+ )+ = (k + m+ )+

por la definici´on de adici´on por la hip´otesis de inducci´on por la definici´on de adici´on

que es lo que deb´ıamos demostrar. Ahora s´ı probaremos la propiedad conmutativa de la adici´on que afirma: para todo m, n n´ umeros naturales se tiene que m + n = n + m. Prueba: A pesar de que hay dos variables, basta hacer inducci´on sobre una de ellas (¿por qu´e?), elijamos m: i. Para m = 0, por ser 0 el m´odulo de la adici´on, tenemos que 0 + n = n = n + 0. 229

´ ´ ACTIVIDADES MATEMATICAS PARA EL DESARROLLO DE PROCESOS LOGICOS

ii. Suponemos que para m = k se tiene que k + n = n + k, debemos probar que k + + n = n + k + , pero esto es cierto puesto que:

k + + n = (k + n)+ = (n + k)+ = n + k+

por el teorema anterior por la hip´otesis de inducci´on por la definici´on de adici´on

La u ´ltima igualdad termina la prueba. 7.4.1.3.

Propiedad asociativa de la adici´ on

Para todo m, n, k n´ umeros naturales se cumple que: (m + n) + k = n + (m + k). Prueba: Tambi´en aqu´ı basta hacer inducci´on sobre una variable, digamos k: i. Para k = 0, aplicamos dos veces la propiedad modulativa y obtenemos que: (m + n) + 0 = m + n = m + (n + 0) ii. Suponemos que para k = t se tiene que (m + n) + t = m + (n + t) para todo m, n n´ umeros naturales, debemos probar que (m + n) + t+ = m + (n + t+ ) Y esto se deduce de: (m + n) + t+ = ((m + n) + t)+ = (m + (n + t))+ = (m + (n + t)+ ) = (m + (n + t+ )). La u ´ltima expresi´on es el resultado deseado. 230

por la definici´on de suma. por la hip´otesis de inducci´on por la definici´on de suma

´ ´ POR INDUCCION ´ MATEMATICA ´ EL METODO DE DEMOSTRACION

7.4.2.

La multiplicaci´ on de N´ umeros Naturales

La multiplicaci´on tambi´en admite una definici´on por recurrencia de la siguiente forma: i. n · 0 = 0 ii. n · k + = n · k + n. Para cualesquier n´ umero naturales n y k. 7.4.2.1.

Propiedad modulativa de la multiplicaci´ on

Para todo n´ umero natural n se cumple que n·1= 1·n = n donde hemos definido 1 = 0+ . Prueba:

i. Para n = 0, por la primera parte de la definici´on de multiplicaci´on se tiene que 1 · 0 = 0. Y de la segunda parte, 0 · 1 = 0 · 0+ = 0·0+0 = 0+0= 0

por la definici´on de 1 por la definici´on de multiplicaci´on por la propiedad modulativa de la adici´on

ii. Suponemos v´alido para n = k que k · 1 = 1 · k = k, y debemos demostrar que se cumple para su sucesor k + ; es decir, que k + · 1 = 1 · k + = k + . 1 · k+ = 1 · k + 1 =k+1 = k + 0+ = (k + 0)+ = k+

por por por por por

la definici´on de multiplicaci´on hip´otesis de inducci´on la definici´on de 1 la definici´on de suma la propiedad modulativa de la suma.

231

´ ´ ACTIVIDADES MATEMATICAS PARA EL DESARROLLO DE PROCESOS LOGICOS

Falta probar que: k+ · 1 = k+. Pero esto es inmediato, puesto que k + · 1 = k + · 0+ = k+ · 0 + k+ = 0 + k+ = k+

por por por por

la la la la

definici´on de 1 definici´on de multiplicaci´on definici´on de multiplicaci´on propiedad modulativa de la suma.

Por lo tanto, la afirmaci´on es v´alida para todo n´ umero natural n. 7.4.2.2.

Para todo n´ umero natural n se tiene que 0·n=0

En la definici´on de multiplicaci´on se afirma que n · 0 = 0, probaremos ahora que 0 · n = 0. Prueba: Hacemos inducci´on sobre n. i. Para n = 0, se tiene que 0 · 0 = 0 por la definici´on de multiplicaci´on. ii. Supongamos que para n = k , 0 · k = 0, probemos que 0 · k + = 0.

0 · k+ = 0 · k + 0 = 0+0 =0

7.4.2.3.

por la definici´on de multiplicaci´on por hip´otesis de inducci´on por la propiedad modulativa de la suma.

Propiedad distributiva de la multiplicaci´ on con respecto a la adici´ on

Para todo m, n, k, n´ umeros naturales se cumple que (n + k) · m = n · m + k · m. 232

´ ´ POR INDUCCION ´ MATEMATICA ´ EL METODO DE DEMOSTRACION

Prueba: Debemos hacer doble inducci´on, inicialmente hagamos inducci´on sobre m: i. Para m = 0, tenemos que (n + k) · 0 = 0 =n·0+k·0

por el teorema anterior por la propiedad modulativa de la adici´on

ii. Suponemos que se cumple que (n + k) · m = n · m + k · m, para todo m y n n´ umeros naturales y debemos demostrar que (n + k) · m+ = n · m+ + k · m+ Pero, (n + k) · m+ = (n + k) · m + (n + k) por la definici´on de multiplicaci´on = (n · m + k · m) + (n + k) por la hip´otesis de inducci´on = (n · m + n) + (k · m + k) por las propiedades conmutativa y asociativa de la adici´on. + + por la definici´on de multiplicaci´on =n·m +k·m que es lo que quer´ıamos demostrar. 7.4.2.4.

Propiedad conmutativa de la multiplicaci´ on

Para todo m, n, n´ umeros naturales se cumple que m · n = n · m. Prueba: Hacemos inducci´on sobre m: i. Para m = 0, tenemos que 0 · n = 0 = n · 0 por el teorema anterior. ii. Suponemos que para m = k se tiene que k · n = n · k, debemos probar que k+ · n = n · k+. 233

´ ´ ACTIVIDADES MATEMATICAS PARA EL DESARROLLO DE PROCESOS LOGICOS

Tenemos que: n · k+ = n · k + n =k·n+n = k·n+1·n = (k + 1) · n = (k + 0+ ) · n = (k + 0)+ · n = k+ · n

por por por por por por por

la la la la la la la

definici´on de multiplicaci´on hip´otesis de inducci´on propiedad modulativa de la multiplicaci´on propiedad distributiva definici´on de 1 definici´on de adici´on propiedad modulativa de la adici´on

Ejercicios 1. Complete la prueba de la propiedad distributiva haciendo inducci´on sobre n o sobre k. 2. Demuestre la propiedad asociativa de la multiplicaci´on de n´ umeros naturales; es decir, que para todo m, n, k n´ umeros naturales se cumple que: (m · n) · k = n · (m · k). 3. Demuestre las leyes cancelativas de la adici´on y la multiplicaci´on; es decir, que: a) Para todo m, n, k, n´ umero natural se cumple que, si m + n = m + k entonces n = k. b)

7.5.

Para todo m, n, k, n´ umero natural, m = 0, se tiene que, si m·n = m·k entonces n = k

El orden en los n´ umeros naturales

Entre los n´ umeros naturales hemos establecido una jerarqu´ıa que nos permite decir cuando uno de ellos es mayor que otro; las propiedades de esta relaci´on, conocida como una relaci´ on de orden, tambi´en puede demostrarse usando las propiedades de los n´ umeros naturales que hemos demostrado en este cap´ıtulo. 234

´ ´ POR INDUCCION ´ MATEMATICA ´ EL METODO DE DEMOSTRACION

Decimos que entre dos n´ umeros naturales a y b, a es menor o igual que b, o tambi´en que b es mayor o igual que a y se nota a ≤ b si y s´olo si existe un n´ umero natural c tal que a + c = b. Esta relaci´on cumple las siguientes propiedades: 1. Reflexiva: para todo n´ umero natural a se cumple que a ≤ a. Prueba: El primer axioma de Peano garantiza que el 0 es un n´ umero natural y la propiedad modulativa de la suma afirma que para todo n´ umero natural a, se tiene que a + 0 = a, por lo tanto a ≤ a para todo n´ umero natural a. 2. Antisim´etrica: dados dos n´ umeros naturales cualesquiera a y b, si se cumple que a ≤ b y tambi´en se cumple que b ≤ a entonces debe cumplirse que a = b. Prueba: Si se cumple que a ≤ b entonces existe un n´ umero natural c tal que a+c = b, y si adem´as se cumple que b ≤ a, existe tambi´en un n´ umero natural d tal que b + d = a. Si reemplazamos b en la segunda igualdad, obtenemos : (a + c) + d = a aplicando las propiedades asociativa y cancelativa de la adici´on, la igualdad se convierte en c+d =0 lo que implica que c = d = 0, porque si no fuera as´ı y uno de ellos, por ejemplo d fuera diferente de 0, existir´ıa un n´ umero natural p tal que p+ = d (¿por qu´e?) y entonces c + d = c + p+ = (c + p)+ = 0 lo que significar´ıa que 0 es sucesor de (c + p), lo que contradice el 4 axioma de Peano. 235

´ ´ ACTIVIDADES MATEMATICAS PARA EL DESARROLLO DE PROCESOS LOGICOS

3. Transitiva: dados n´ umeros naturales cualesquiera a, byc, si se cumple que a ≤ b y tambi´en se cumple que b ≤ c entonces debe cumplirse que a ≤ c. Prueba: Si se cumple que a ≤ b entonces existe un n´ umero natural k tal que a + k = b, y si adem´as se cumple que b ≤ c, existe tambi´en un n´ umero natural d tal que b + d = c. Si reemplazamos b en la segunda igualdad, obtenemos: (a + k) + d = c aplicando la propiedad asociativa de la adici´on, la igualdad se convierte en a + (k + d) = c lo que significa que a ≤ c, puesto que (k + d) es un n´ umero natural. 4. Monoton´ıa de la adici´ on: dados n´ umeros naturales cualesquiera a, b, c y d, si a ≤ b y c ≤ d entonces a + c ≤ b + d Prueba: Si a ≤ b entonces existe un n´ umero natural k tal que a + k = b, y si adem´as c ≤ d, existe tambi´en un n´ umero natural n tal que c + n = d. Si sumamos las dos igualdades, obtenemos: (a + k) + (c + n) = b + d aplicando las propiedades conmutativa y asociativa de la adici´on, la anterior igualdad toma la forma (a + c) + (k + n) = b + d lo que significa que a + c ≤ b + d, puesto que (k + n) es un n´ umero natural.

236

´ ´ POR INDUCCION ´ MATEMATICA ´ EL METODO DE DEMOSTRACION

Ejercicio 1. Demuestre la propiedad de monoton´ıa para la multiplicaci´ on, esto es que: dados n´ umeros naturales cualesquiera a, b y c, si a ≤ b entonces a · c ≤ b · c 2. Demuestre que si a, b, c son n´ umeros naturales y a · c < b · c entonces a < b. 3. Si definimos por recurrencia la potenciaci´on de n´ umeros naturales por las f´ormulas: i. a0 = 1 +

ii. an = an · a demuestre que an+m = an am (an )m = an·m 4. En el cap´ıtulo 1 definimos una operaci´on que reitera la potenciaci´on; esta se puede definir por recurrencia con las f´ormulas: i. 1 a = a ii.

n+

a = (n a)a

Estudie sus propiedades y demu´estrelas por inducci´on. Notemos la diferencia de los procedimientos y los razonamientos que hacemos en este cap´ıtulo con lo hecho en los cap´ıtulos anteriores, aqu´ı lo que importa son los axiomas y sus conexiones l´ogicas sin importar las representaciones que de las palabras “n´ umero natural”podamos hacer. Por ejemplo, si llamamos 1 = 0+ , 2 = 1+ , 3 = 2+ , 4 = 3+ , · · · , etc. Podemos demostrar que 3+1= 4

(¿No le parece ex´otico, demostrar esto?)

Veamos: 3 + 1 = 1 + 3 = 1 + 2+ = (1 + 2)+ = (1 + 1+ )+ = ((1 + 1)+ )+ = ((1 + 0+ )+ )+ = (((1 + 0)+ )+ )+ = ((1+ )+ )+ = (2+ )+ = 3+ = 4. Esta es una de las maneras oficiales de presentar los n´ umeros naturales; usted puede averiguar otras. 237

´ ´ ACTIVIDADES MATEMATICAS PARA EL DESARROLLO DE PROCESOS LOGICOS

❆ ❆ ❆ ❆ ❆ ❆ ❆ ❆ ❆ ❆ ❆ ❆ ❆ ❆ ❆ ❆ ❆ ❆ El estudio aqu´ı apenas comienza. ❆ ❆ ❆ ❆ ❆ ❆ ❆ ❆ ❆ ❆ ❆ ❆ ❆ ❆ ❆ ❆ ❆ ❆

238

BIBLIOGRAF´IA

[1] AABOE, A., Matem´aticas: Episodios hist´ oricos, Ed. Norma, Cali, Colombia, 1964. ´ [2] APOSTOL, T., Calculus, Vol. I., Ed. Revert´e, Barcelona, 1988. ´ [3] APOSTOL, T., Introducci´ on a la teor´ıa anal´ıtica de n´ umeros, Ed. Revert´e, Barcelona, 1984. [4] BALDOR, A., Aritm´etica, Ed. Cultural Colombiana, Bogot´a, 1972. [5] BELL, E. T., Historia de las matem´ aticas, Ed. Fondo de Cultura Econ´omica, M´exico, 1995. [6] BOYER, Carl., Historia de la matem´ atica, Ed. Alianza, Madrid. 1968. [7] BURNS, W., “La tabla de c´alculo de los incas”, en Bolet´ın de Lima, Lima. [8] BURTON, Jones., Teor´ıa de los n´ umeros, Ed. Trillas, M´exico, 1969. [9] CAMPOS, A., “El m´as bello teorema”, en Memorias VII Encuentro de Gemetr´ıa y sus aplicaciones, Universidad Pedag´ogica Nacional, Bogot´a, 1996. [10] CARO, V., Los n´ umeros: su historia, sus propiedades, sus mentiras y verdades, Ed. Minerva, Bogot´a, 1936. [11] CHURCHILL, Eileen M., Contando y midiendo, Ed. Uteha, M´exico, 1965. 239

´ ´ ACTIVIDADES MATEMATICAS PARA EL DESARROLLO DE PROCESOS LOGICOS

[12] COSSIU, del Pomar Felipe., El mundo de los incas, Ed. Fondo de Cultura Econ´omica, M´exico, 1969. [13] COURANT, R. y ROBBINS, R., ¿Qu´e es la Matem´ atica?, Ed. Aguilar, Madrid, 1971. ´ [14] DONADO, A., LUQUE, C. y PAEZ, J., El proceso de inducir, Universidad Pedag´ogica Nacional, Bogot´a, 1998. ´ [15] FRALEIGH, J. B., Algebra abstracta., Ed. Adisson Wesley Iberoamericana, M´exico, 1988. [16] GARDNER, M., Rosquillas Anudadas, Ed. Labor, Barcelona, 1987. [17] GAUSS, C. F., Disqisitiones Arithmeticae, Academia Colombiana de Ciencias, Bogot´a, 1995. ´ [18] GERDES, Paulus y CHERINDA, Marcos., Contar en Africa, Par´ıs, Vol. 46., noviembre de 1993. [19] GOLDSTEIN C., En: Mundo cient´ıfico, Vol. 15, No161, octubre de 1995. [20] GORDILLO, J., JIMENEZ, R. y RUBIANO, G., Teor´ıa de n´ umeros para principiantes, Universidad Nacional de Colombia, Bogot´a, Bogot´a, 1999. [21] Grandes matem´ aticos. En: Investigaci´on y Ciencia. Barcelona. 1995. [22] HALMOS, P., Naive Set theory, Ed. Van Nostrand, Nueva York, 1960. [23] HAMMOND, N., “Unearthing the Oldest Known Maya”, en National Geographic, Vol. 162, No 1, julio de 1982. ´ [24] HERSTEIN, I. N., Algebra moderna, Ed. Trillas, M´exico, 1976. [25] JOHNSON, D. y JOHNSON J., Graph Theory with Engineering applications, Ed. Ronald Press, Nueva York, 1972. [26] KLEIN, M., El pensamiento matem´atico: de la antig¨ uedad a nuestros d´ıas, Ed. Alianza, Madrid, 1994. [27] KREYSZIG, E., Introducci´ on a la estad´ıstica matem´ atica, principios y m´etodos, Ed. Limusa. 1978. 240

BIBLIOGRAF´IA

´ [28] LUQUE C., MORENO H. y PINZON, L., “Un an´alogo al teorema de Pit´agoras para n´ umeros triangulares”, en Memorias IX Encuentro de Geometr´ıa y sus Aplicaciones, Universidad Pedag´ogica Nacional, Bogot´a, 1998. [29] LUQUE, C., MORA, L. y VALERO, N., La yupana como herramienta pedag´ogica, en: Memorias X Encuentro de Geometr´ıa y sus Aplicaciones. Universidad Pedag´ogica Nacional, Bogot´a, 1999. [30] Mathematics Teacher. Vol. 85. No 9. Diciembre 1992. [31] MESERVE, B. y SOBEL, M., Introducci´ on a las matem´aticas, Ed. Revert´e, Barcelona, 1967. [32] MOSTER´IN, J., “The Natural Numbers as a Universal Library”, en Philosophy of Mathematics Today, Ed. Kluwer Academic Publishers, 1997. ˜ [33] MUNOZ, J. M., Introducci´ on a la teor´ıa de conjuntos, Departamento de Matem´aticas y Estad´ıstica, Universidad Nacional de Colombia, Bogot´a, 1993. ˜ [34] MUNOZ, J. M., “Imposible duplicar el cubo. ¿Sabe usted por qu´e?”, en Memorias VII Coloquio Distrital de Matem´ aticas y Estad´ıstica, Universidad Nacional, Bogot´a, diciembre de 1990. [35] NAVARRO, J. La nueva matem´atica, Salvat, Barcelona, 1973. [36] NELSEN, R., Proofs without words, The Mathematical Association of America, Washington, 1993. [37] NEWMAN, J., Sigma: el mundo de las matem´ aticas, Vol. I, IV, V. Ed. Grijalbo, Barcelona, 1994. [38] PAREJA, D., Arithmetical Algorithms of the Incas, Universidad del Quind´ıo, Armenia, 1986. [39] PAREJA, D., Instrumentos prehisp´ anicos de c´ alculo: el quipu y la yupana, Instituto de Investigaciones y posgrados, Universidad del Quind´ıo, Armenia, 1986. ´ [40] PERELMAN, Y., Algebra recreativa. Ed. Mir, Mosc´ u, 1989. [41] RODR´IGUEZ, R., Diversiones matem´aticas, Ed. Revert´e, Barcelona, 1987. 241

´ ´ ACTIVIDADES MATEMATICAS PARA EL DESARROLLO DE PROCESOS LOGICOS

[42] SINGH, S., El enigma de Fermat, Ed. Planeta, Barcelona, 1998. [43] SPIEGEL, M., Teor´ıa y problemas de estad´ıstica, Ed. Mc Graw Hill, 1991. [44] SPIVAK, M., C´alculo infinitesimal, Ed. Revert´e, Barcelona, 1974. [45] SUPESS, P., Teor´ıa axiom´ atica de Conjuntos, Ed. Norma, Cali, 1976. [46] TAHAN, M., El hombre que calculaba, Ed. Panamericana, Bogot´a, 1994. ´ [47] VAN DER WAERDEN, Bartel Leenert., A History of Algebra, Ed. SpringerVerlag, Germany, 1985. [48] VON HAGEN,V.W., El mundo de los mayas, Ed. Diana, M´exico, 1960. [49]

, Los incas, Colecci´on culturas b´asicas del mundo, M´exico, 1964.

[50] WASSEN, H., El antiguo a´baco peruano, seg´ un el manuscrito de Guam´an Poma, G¨otemburgo, 1940.

242